sat practice test 5.pdf

144
Help | Profile | My Organizer | My Bookmarks | Logout Answers and Explanations Test Sections Section 1 Section 2 Section 3 Section 4 Section 5 Section 6 Section 8 Section 9 Section 10 Back to Score Report Essay Online - Practice Test #5 These sample essays were originally handwritten by students but are shown typed here for ease of reading. The essays are displayed exactly as students wrote them, without any corrections to spelling, punctuation, or syntax. One handwritten sample essay is provided to illustrate the need for legible and clear handwriting. Exemplars: Essay Prompt Think carefully about the information presented in the following excerpt and the assignment below. Many people believe that closed doors make us creative.These people argue that obstacles and restrictions are necessary, for without them we would never be forced to come up with new solutions. But closed doors,either in the form of specific obstacles or a lack of opportunities, often prevent people from reaching their full creative potential. Do closed doors make us creative? Plan and write an essay in which you develop your point of view on this issue. Support your position with reasoning and examples taken from your reading, studies, experience, or observations. Back to Score Report Copyright © 2006 The College Board. All rights reserved. Privacy Policy Terms of Use Contact Us 页码1/1 The Official SAT Online Course 2006-11-12 file://E:\新建文件\e2.htm UnRegistered

Upload: sohaylla-musad

Post on 08-Nov-2014

472 views

Category:

Documents


5 download

DESCRIPTION

Online course SAT test # 5

TRANSCRIPT

Page 1: SAT practice test 5.pdf

严禁用于商业用途!

Help | Profile | My Organizer | My Bookmarks | Logout

Answers and Explanations

Test Sections

Section 1

Section 2

Section 3

Section 4

Section 5

Section 6

Section 8

Section 9

Section 10

Back to Score Report  

Essay     Online - Practice Test #5

These sample essays were originally handwritten by students but are shown typed here for ease of reading. The essays are displayed exactly as students wrote them, without any corrections to spelling, punctuation, or syntax. One handwritten sample essay is provided to illustrate the need for legible and clear handwriting.

Exemplars:

Essay Prompt

Think carefully about the information presented in the following excerpt and the assignment below.

Many people believe that “closed doors make us creative.” These people argue that obstacles and restrictions are necessary, for without them we would never be forced to come up with new solutions. But “closed doors,” either in the form of specific obstacles or a lack of opportunities, often prevent people from reaching their full creative potential.

Do closed doors make us creative? Plan and write an essay in which you develop your point of view on this issue. Support your position with reasoning and examples taken from your reading, studies, experience, or observations.

Back to Score Report  

Copyright © 2006 The College Board. All rights reserved. Privacy Policy Terms of Use Contact Us

 

页码,1/1The Official SAT Online Course

2006-11-12file://E:\新建文件夹\e2.htm

UnRe

gistered

Page 2: SAT practice test 5.pdf

严禁用于商业用途!

Help | Profile | My Organizer | My Bookmarks | Logout

Answers and Explanations

Test Sections

Section 1

Section 2

Section 3

Section 4

Section 5

Section 6

Section 8

Section 9

Section 10

Back to Score Report  

View Answers and Explanations     Online - Practice Test #5

1 The garden that had remained ------- for months was now pleasantly enlivened by the budding shoots of its perennial flowers.

ANSWERS AND EXPLANATIONS Explanation for Correct Answer B :  Choice (B) is correct. “Dormant” means inactive or not growing. If one were to insert this term into the text, the sentence would read “The garden that had remained dormant for months was now pleasantly enlivened by the budding shoots of its perennial flowers.” A “dormant,” or inactive, garden would become active as its perennial flowers began to grow.

Explanation for Incorrect Answer A :  Choice (A) is incorrect. “Redolent” means aromatic or fragrant. If one were to insert this term into the text, the sentence would read “The garden that had remained redolent for months was now pleasantly enlivened by the budding shoots of its perennial flowers.” A garden in full bloom can be “redolent,” but the phrase “now pleasantly enlivened” suggests that the garden was inactive before the flowers began to grow. An inactive garden is most likely not fragrant.

Explanation for Incorrect Answer C :  Choice (C) is incorrect. “Exuberant” means plentiful or producing in abundance. If one were to insert this term into the text, the sentence would read “The garden that had remained exuberant for months was now pleasantly enlivened by the budding shoots of its perennial flowers.” The phrase “now pleasantly enlivened” suggests that the garden was inactive before the flowers began to grow. An inactive garden would most likely not be described as “exuberant,” or plentiful.

Explanation for Incorrect Answer D :  Choice (D) is incorrect. “Compliant” means willing to conform. If one were to insert this term into the text, the sentence would read “The garden that had remained compliant for months was now pleasantly enlivened by the budding shoots of its perennial flowers.” It is illogical to suggest that a garden could be “compliant,” or willing to conform.

Explanation for Incorrect Answer E :  Choice (E) is incorrect. “Trenchant” means keen or sharply perceptive. If one were to insert this term into the text, the sentence would read “The garden that had remained trenchant for months was now pleasantly enlivened by the budding shoots of its perennial flowers.” It is illogical to suggest that a garden could be “trenchant,” or sharply perceptive.

(A) redolent

(B) dormant

(C) exuberant

(D) compliant

(E) trenchant

2 After several months of training, the ------- young spaniel was finally ------- enough to be walked safely without a leash.

(A) eager . . unruly

页码,1/23The Official SAT Online Course

2006-11-12file://E:\新建文件夹\e3.htm

UnRe

gistered

Page 3: SAT practice test 5.pdf

严禁用于商业用途!

ANSWERS AND EXPLANATIONS Explanation for Correct Answer D :  Choice (D) is correct. “Boisterous” means rowdy and undisciplined. “Docile” means easily managed. If one were to insert these terms into the text, the sentence would read “After several months of training, the boisterous young spaniel was finally docile enough to be walked safely without a leash.” The word “finally” indicates that after several months of training, the spaniel’s behavior changed. It is logical to suggest that a “boisterous” dog could become “docile” after several months of training. A dog that is docile, or easily managed, could likely be walked safely without a leash.

Explanation for Incorrect Answer A :  Choice (A) is incorrect. “Eager” means enthusiastic or impatient. “Unruly” means undisciplined and willful. If one were to insert these terms into the text, the sentence would read “After several months of training, the eager young spaniel was finally unruly enough to be walked safely without a leash.” Although a dog can certainly be “eager,” it is unlikely that an “unruly” dog could be walked safely without a leash. Furthermore, one would not expect training to cause a dog to be unruly, or undisciplined.

Explanation for Incorrect Answer B :  Choice (B) is incorrect. “Placid” means calm and undisturbed. “Defiant” means bold and disobedient. If one were to insert these terms into the text, the sentence would read “After several months of training, the placid young spaniel was finally defiant enough to be walked safely without a leash.” Although a dog can certainly be “placid,” or calm, it is unlikely that a “defiant” dog could be walked safely without a leash. Furthermore, one would not expect training to cause a dog to be defiant, or disobedient.

Explanation for Incorrect Answer C :  Choice (C) is incorrect. “Clever” means smart and quick-witted. “Helpful” means providing assistance. If one were to insert these terms into the text, the sentence would read “After several months of training, the clever young spaniel was finally helpful enough to be walked safely without a leash.” Although the first term fits within the context of the sentence, the second term does not. A dog can certainly be “clever,” but it does not make logical sense to say that a dog is providing assistance by walking without a leash.

Explanation for Incorrect Answer E :  Choice (E) is incorrect. “Vigilant” means watchful and alert. “Convinced” means certain or persuaded. If one were to insert these terms into the text, the sentence would read “After several months of training, the vigilant young spaniel was finally convinced enough to be walked safely without a leash.” Although a dog can be “vigilant,” it does not make sense in this context to say that the dog was “convinced enough” to be walked without a leash. It is not clear what the dog has been convinced of or why convincing was necessary in order to walk the dog without a leash—or, indeed, that dogs can be “convinced,” a term usually applied to humans.

(B) placid . . defiant

(C) clever . . helpful

(D) boisterous . . docile

(E) vigilant . . convinced

3

------- as Mario’s misdeed was, his grandmother, always blind to his faults, pretended to be unaware of it.

(A) Accidental

(B) Apt

(C) Random

(D) Flagrant

(E) Covert

页码,2/23The Official SAT Online Course

2006-11-12file://E:\新建文件夹\e3.htm

UnRe

gistered

Page 4: SAT practice test 5.pdf

严禁用于商业用途!

ANSWERS AND EXPLANATIONS Explanation for Correct Answer D :  Choice (D) is correct. “Flagrant” means conspicuously or obviously offensive. If one were to insert this term into the text, the sentence would read “Flagrant as Mario’s misdeed was, his grandmother, always blind to his faults, pretended to be unaware of it.” The structure of the sentence indicates that Mario’s grandmother was certainly aware of Mario’s misdeed, even though she pretended to be unaware. The missing term should describe a misdeed that is very obvious. The term “flagrant” accurately describes an action that is obviously offensive.

Explanation for Incorrect Answer A :  Choice (A) is incorrect. “Accidental” means occurring unexpectedly or by chance. If one were to insert this term into the text, the sentence would read “Accidental as Mario’s misdeed was, his grandmother, always blind to his faults, pretended to be unaware of it.” The structure of the sentence indicates that Mario’s grandmother was certainly aware of Mario’s misdeed, even though she pretended to be unaware. The missing term should describe a misdeed that is very obvious. An action that is “accidental” would not necessarily be obvious.

Explanation for Incorrect Answer B :  Choice (B) is incorrect. “Apt” means appropriate or suitable. If one were to insert this term into the text, the sentence would read “Apt as Mario’s misdeed was, his grandmother, always blind to his faults, pretended to be unaware of it.” A “misdeed” is inappropriate behavior and would not likely be described as “apt.”

Explanation for Incorrect Answer C :  Choice (C) is incorrect. “Random” means lacking a definite pattern or plan. If one were to insert this term into the text, the sentence would read “Random as Mario’s misdeed was, his grandmother, always blind to his faults, pretended to be unaware of it.” The structure of the sentence indicates that Mario’s grandmother was certainly aware of Mario’s misdeed, even though she pretended to be unaware. The missing term should describe a misdeed that is very obvious. An action that is “random,” or does not follow a pattern or plan, would not necessarily be obvious.

Explanation for Incorrect Answer E :  Choice (E) is incorrect. “Covert” means secret or not openly shown. If one were to insert this term into the text, the sentence would read “Covert as Mario’s misdeed was, his grandmother, always blind to his faults, pretended to be unaware of it.” The structure of the sentence indicates that Mario’s grandmother was certainly aware of Mario’s misdeed, even though she pretended to be unaware. The missing term should describe a misdeed that is very obvious. A “covert,” or secret, action is the opposite of an obvious action. Mario’s grandmother would most likely not be aware of a “covert” misdeed.

4 Despite his ------- desire to show off, he remained at heart a very ------- person.

ANSWERS AND EXPLANATIONS Explanation for Correct Answer C :  Choice (C) is correct. “Occasional” means happening only sometimes. “Reticent” means quiet and reserved. If one were to insert these terms into the text, the sentence would read “Despite his occasional desire to show off, he remained at heart a very reticent person.” The word “Despite” indicates that the missing terms will describe contrasting behaviors. Someone who is “reticent,” or quiet and reserved, would not frequently show off but might do so from time to time.

(A) uncharacteristic. . demonstrative

(B) inexplicable. . hedonistic

(C) occasional. . reticent

(D) continual. . transparent

(E) blatant. . exhibitionistic

页码,3/23The Official SAT Online Course

2006-11-12file://E:\新建文件夹\e3.htm

UnRe

gistered

Page 5: SAT practice test 5.pdf

严禁用于商业用途!

Explanation for Incorrect Answer A :  Choice (A) is incorrect. “Uncharacteristic” means not typical. “Demonstrative” means displaying feelings openly. If one were to insert these terms into the text, the sentence would read “Despite his uncharacteristic desire to show off, he remained at heart a very demonstrative person.” The word “Despite” indicates that the missing terms will describe contrasting behaviors. There is no inherent contrast between the terms “uncharacteristic” and “demonstrative,” because showing off is not necessarily uncharacteristic behavior for someone who displays his feelings openly.

Explanation for Incorrect Answer B :  Choice (B) is incorrect. “Inexplicable” means incapable of being explained. “Hedonistic” means concerned primarily with happiness and pleasure. If one were to insert these terms into the text, the sentence would read “Despite his inexplicable desire to show off, he remained at heart a very hedonistic person.” The word “Despite” indicates that the missing terms will describe contrasting behaviors. Even if the desire to show off is “inexplicable,” or incapable of being explained, there is no inherent contrast between having a desire to show off and being “hedonistic.” Showing off is not necessarily related to being primarily concerned with happiness and pleasure.

Explanation for Incorrect Answer D :  Choice (D) is incorrect. “Continual” means going on without interruption. “Transparent” means obvious and free from pretense. If one were to insert these terms into the text, the sentence would read “Despite his continual desire to show off, he remained at heart a very transparent person.” The word “Despite” indicates that the missing terms will describe contrasting behaviors. There is no inherent contrast between having a “continual” desire to show off and being a “transparent” person, and a person could be obvious in his desire to show off.

Explanation for Incorrect Answer E :  Choice (E) is incorrect. “Blatant” means completely obvious. “Exhibitionistic” means acting in a way that will attract attention. If one were to insert these terms into the text, the sentence would read “Despite his blatant desire to show off, he remained at heart a very exhibitionistic person.” The word “Despite” indicates that the missing terms will describe contrasting behaviors. There is no contrast between having a “blatant” desire to show off and being an “exhibitionistic” person. On the contrary, someone who wants to attract attention would probably be likely to show off.

5 The employer blamed the staff member's lack of productivity on ------- rather than incompetence, claiming that the man knew how to do his job but was too lazy to apply himself.

ANSWERS AND EXPLANATIONS Explanation for Correct Answer D :  Choice (D) is correct. “Indolence” is a tendency to be lazy. If one were to insert this term into the text, the sentence would read “The employer blamed the staff member’s lack of productivity on indolence rather than incompetence, claiming that the man knew how to do his job but was too lazy to apply himself.” The employer’s claim that the staff member was competent but “too lazy to apply himself” indicates that the staff member’s lack of productivity was the result of laziness. The term “indolence” precisely describes the laziness that the employer claimed was responsible for the staff member’s lack of productivity.

Explanation for Incorrect Answer A :  Choice (A) is incorrect. “Infatuation” is a foolish or extravagant love or admiration for something. If one were to insert this term into the text, the sentence would read “The employer blamed the staff member’s lack of productivity on

(A) infatuation

(B) tension

(C) indigence

(D) indolence

(E) ineptitude

页码,4/23The Official SAT Online Course

2006-11-12file://E:\新建文件夹\e3.htm

UnRe

gistered

Page 6: SAT practice test 5.pdf

严禁用于商业用途!

infatuation rather than incompetence, claiming that the man knew how to do his job but was too lazy to apply himself.” The employer’s claim that the staff member was competent but “too lazy to apply himself” indicates that the staff member’s lack of productivity was the result of laziness. The missing term should describe this laziness. “Infatuation,” or a foolish love, does not describe laziness.

Explanation for Incorrect Answer B :  Choice (B) is incorrect. “Tension” is stress. If one were to insert this term into the text, the sentence would read “The employer blamed the staff member’s lack of productivity on tension rather than incompetence, claiming that the man knew how to do his job but was too lazy to apply himself.” The employer’s claim that the staff member was competent but “too lazy to apply himself” indicates that the staff member’s lack of productivity was the result of laziness. The missing term should describe this laziness. “Tension,” or stress, does not describe laziness.

Explanation for Incorrect Answer C :  Choice (C) is incorrect. “Indigence” is extreme poverty. If one were to insert this term into the text, the sentence would read “The employer blamed the staff member’s lack of productivity on indigence rather than incompetence, claiming that the man knew how to do his job but was too lazy to apply himself.” The employer’s claim that the staff member was competent but “too lazy to apply himself” indicates that the staff member’s lack of productivity was the result of laziness. The missing term should describe this laziness. “Indigence,” or extreme poverty, does not describe laziness.

Explanation for Incorrect Answer E :  Choice (E) is incorrect. “Ineptitude” is incompetence and a lack of sense. If one were to insert this term into the text, the sentence would read “The employer blamed the staff member’s lack of productivity on ineptitude rather than incompetence, claiming that the man knew how to do his job but was too lazy to apply himself.” The term “ineptitude” is not logical in this context. “Ineptitude” means incompetence. The employer did not blame the staff member’s lack of productivity on incompetence but, rather, claimed that the staff member “knew how to do his job.”

6 The audience recognized the officer’s characteristic ------- when he attributed his achievements to ------- rather than bravery.

ANSWERS AND EXPLANATIONS Explanation for Correct Answer C :  Choice (C) is correct. “Humility” is the quality of being humble and not overly proud. “Fortune” means luck or chance. If one were to insert these terms into the text, the sentence would read “The audience recognized the officer’s characteristic humility when he attributed his achievements to fortune rather than bravery.” The sentence structure indicates that the first missing term will describe the behavior demonstrated in the officer’s discussion of his achievements. Attributing achievements to “fortune,” or luck, instead of bravery is an example of “humility.”

Explanation for Incorrect Answer A :  Choice (A) is incorrect. “Pedantry” means to pay extreme attention to rules. “Chance” means luck or an unpredictable advantage. If one were to insert these terms into the text, the sentence would read “The audience recognized the officer’s characteristic pedantry when he attributed his achievements to chance rather than bravery.” The sentence structure indicates that the first missing term will describe the behavior demonstrated in the officer’s discussion of his achievements. Attributing achievements to “chance,” or luck, is not an example of “pedantry,” or extreme attention to rules.

Explanation for Incorrect Answer B : 

(A) pedantry. . chance

(B) gallantry. . whimsy

(C) humility. . fortune

(D) bravado. . accident

(E) effrontery. . discretion

页码,5/23The Official SAT Online Course

2006-11-12file://E:\新建文件夹\e3.htm

UnRe

gistered

Page 7: SAT practice test 5.pdf

严禁用于商业用途!

Choice (B) is incorrect. “Gallantry” is very noble and brave behavior. “Whimsy” is a whim or a sudden impulse. If one were to insert these terms into the text, the sentence would read “The audience recognized the officer’s characteristic gallantry when he attributed his achievements to whimsy rather than bravery.” The sentence structure indicates that the first missing term will describe the behavior demonstrated by the officer’s discussion of his achievements. There is not necessarily anything noble or brave in the officer’s claim that his achievements are due to “whimsy,” or sudden impulse.

Explanation for Incorrect Answer D :  Choice (D) is incorrect. “Bravado” means pretending to be brave. “Accident” is an unplanned and unexpected event. If one were to insert these terms into the text, the sentence would read “The audience recognized the officer’s characteristic bravado when he attributed his achievements to accident rather than bravery.” Attributing his achievements to “accident” instead of bravery is not an example of the officer pretending to be brave. Someone demonstrating “bravado” would probably attribute his achievements to bravery.

Explanation for Incorrect Answer E :  Choice (E) is incorrect. “Effrontery” means offensive boldness. “Discretion” is the ability to make responsible decisions. If one were to insert these terms into the text, the sentence would read “The audience recognized the officer’s characteristic effrontery when he attributed his achievements to discretion rather than bravery.” There is no reason to believe that the audience would consider the officer to be offensively bold when attributing his achievements to “discretion,” or responsible choices.

7The strong ------- the professor was able to establish with his students made him ------- confidant for those on campus seeking advice beyond the purely academic.

ANSWERS AND EXPLANATIONS Explanation for Correct Answer B :  Choice (B) is correct. “Rapport” means a relationship of trust. “Respected” means thought highly of and appreciated. If one were to insert these terms into the text, the sentence would read “The strong rapport the professor was able to establish with his students made him a respected confidant for those on campus seeking advice beyond the purely academic.” A “confidant” is someone one trusts with secrets or private matters. If the professor established a “rapport,” or relationship of trust, with his students, it makes sense that the students would respect his advice on personal matters.

Explanation for Incorrect Answer A :  Choice (A) is incorrect. “Program” means an academic plan or curriculum. “Occasional” means happening only sometimes. If one were to insert these terms into the text, the sentence would read “The strong program the professor was able to establish with his students made him an occasional confidant for those on campus seeking advice beyond the purely academic.” A “confidant” is someone one trusts with secrets or private matters. While the professor could establish a strong “program” with the help of his students, there is not necessarily any reason to believe that the professor’s program would cause his students to seek him out to discuss personal matters.

Explanation for Incorrect Answer C :  Choice (C) is incorrect. “Confidence” is trust in a person or thing. “Unappreciated” means not recognized or not valued. If one were to insert these terms into the text, the sentence would read “The strong confidence the professor was able to establish with his students made him an unappreciated confidant for those on campus seeking advice beyond the purely academic.” A “confidant” is someone one trusts with secrets or private matters. If the professor was able to establish “confidence,” or trust, with his students, it is reasonable to suggest that they would appreciate his advice on personal matters.

(A) program. . an occasional

(B) rapport. . a respected

(C) confidence. . an unappreciated

(D) community. . an unusual

(E) ambition. . a valued

页码,6/23The Official SAT Online Course

2006-11-12file://E:\新建文件夹\e3.htm

UnRe

gistered

Page 8: SAT practice test 5.pdf

严禁用于商业用途!

Explanation for Incorrect Answer D :  Choice (D) is incorrect. “Community” means a group of people with a connection or something in common. “Unusual” means not ordinary. If one were to insert these terms into the text, the sentence would read “The strong community the professor was able to establish with his students made him an unusual confidant for those on campus seeking advice beyond the purely academic.” A “confidant” is someone one trusts with secrets or private matters. If the professor had established a strong “community,” it would not be “unusual” for his students to seek his advice on personal matters.

Explanation for Incorrect Answer E :  Choice (E) is incorrect. “Ambition” is the desire to achieve a specific thing. “Valued” means appreciated and regarded highly. If one were to insert these terms into the text, the sentence would read “The strong ambition the professor was able to establish with his students made him a valued confidant for those on campus seeking advice beyond the purely academic.” While the second term fits the meaning of the sentence, the first term does not. A professor could be a “valued” confidant, or person trusted with private matters, but it does not make logical sense to say that the professor established “ambition” with his students.

8 After David left him waiting for the third consecutive time, Kirk realized that the same behavior he had initially valued as spontaneous and carefree was, in fact, simply -------.

ANSWERS AND EXPLANATIONS Explanation for Correct Answer A :  Choice (A) is correct. “Capricious” means irresponsibly unpredictable. If one were to insert this term into the text, the sentence would read “After David left him waiting for the third consecutive time, Kirk realized that the same behavior he had initially valued as spontaneous and carefree was, in fact, simply capricious.” The phrase “initially valued” suggests that after being left waiting three times in a row, Kirk changed his opinion of David’s behavior. It would make sense to say that Kirk would no longer value David’s behavior if he came to see it as “capricious,” or irresponsibly unpredictable.

Explanation for Incorrect Answer B :  Choice (B) is incorrect. “Incontrovertible” means indisputable or not open to question. If one were to insert this term into the text, the sentence would read “After David left him waiting for the third consecutive time, Kirk realized that the same behavior he had initially valued as spontaneous and carefree was, in fact, simply incontrovertible.” The phrase “initially valued” indicates that after being left waiting three times in a row, Kirk changed his opinion of David’s behavior. There is no reason to believe that Kirk would not have initially believed that David’s behavior was “incontrovertible,” or not open to question.

Explanation for Incorrect Answer C :  Choice (C) is incorrect. “Extraneous” means irrelevant and not essential. If one were to insert this term into the text, the sentence would read “After David left him waiting for the third consecutive time, Kirk realized that the same behavior he had initially valued as spontaneous and carefree was, in fact, simply extraneous.” The phrase “initially valued” suggests that after being left waiting three times in a row, Kirk changed his opinion of David’s behavior. If David’s behavior affected Kirk’s opinion, it is illogical to say that David’s behavior was “extraneous,” or irrelevant.

Explanation for Incorrect Answer D :  Choice (D) is incorrect. “Captivating” means irresistibly appealing. If one were to insert this term into the text, the sentence would read “After David had left him waiting for the third consecutive time, Kirk realized that the same behavior he had initially valued as spontaneous and carefree was, in fact, simply captivating.” The phrase “initially valued” indicates that after being left waiting three times in a

(A) capricious

(B) incontrovertible

(C) extraneous

(D) captivating

(E) inscrutable

页码,7/23The Official SAT Online Course

2006-11-12file://E:\新建文件夹\e3.htm

UnRe

gistered

Page 9: SAT practice test 5.pdf

严禁用于商业用途!

row, Kirk no longer valued David’s behavior. It is illogical to suggest that being left waiting caused Kirk to consider David’s behavior to be “captivating,” or irresistibly appealing.

Explanation for Incorrect Answer E :  Choice (E) is incorrect. “Inscrutable” means mysterious and not readily understood. If one were to insert this term into the text, the sentence would read “After David had left him waiting for the third consecutive time, Kirk realized that the same behavior he had initially valued as spontaneous and carefree was, in fact, simply inscrutable.” The phrase “the same behavior” indicates that while Kirk’s opinion of David’s behavior changed, David’s behavior was always the same. Therefore, the missing term should be another way to describe “spontaneous and carefree” behavior. Behavior that is spontaneous and carefree is not necessarily “inscrutable,” or mysterious.

       What was most likely the original  purpose of the human appendix? Experts  can only theorize on its use. It may haveLine had the same purpose it has in present-5 day herbivores, where it harbors colonies  of bacteria that help in the digestion of  cellulose. Another theory suggests that  tonsils and the appendix might  manufacture the antibody-producing white10 blood cells called B lymphocytes;  however, B lymphocytes could also be  produced by bone marrow. The third  theory is that the appendix may “attract”  body infections in order to localize the15 infection in one spot that is not critical to  body functioning.

9 The author of the passage uses quotation marks in line 13 in order to indicate that

ANSWERS AND EXPLANATIONS Explanation for Correct Answer C :  Choice (C) is correct. In the passage, three theories are given concerning the original purpose of the human appendix. In lines 12-16, the third and final theory is given: “the appendix may ‘attract’ body infections in order to localize the infection in one spot that is not critical to body functioning.” The use of the quotation marks around “attract” implies that there are probably technical terms to describe this “attraction,” but that the author is instead using a term that the general reader will be likely to understand. The reader knows that a bodily organ cannot really “attract,” or draw the attention of, an infection, but the use of this familiar verb helps clarify the concept.

Explanation for Incorrect Answer A :  Choice (A) is incorrect. There is no indication that the author of the passage supports the third theory any more than the other two theories, nor would the use of quotation marks necessarily serve this purpose.

Explanation for Incorrect Answer B :  Choice (B) is incorrect. There is no indication that this third theory is any less

(A) this theory is the one with which the author most nearly agrees

(B) this theory is less scientifically valid than the other theories in the passage

(C) a common word is being used to describe a unique biological process

(D) a word is being used in a humorous way

(E) a direct quotation from another source is being used

页码,8/23The Official SAT Online Course

2006-11-12file://E:\新建文件夹\e3.htm

UnRe

gistered

Page 10: SAT practice test 5.pdf

严禁用于商业用途!

scientifically valid than the other two theories. Using a non-technical term like “attracts” to clarify a concept does not suggest that that concept is less than valid.

Explanation for Incorrect Answer D :  Choice (D) is incorrect. While quotation marks might in certain cases connote humor, that is not the case here. The tone of the sentence—and of the whole passage—is objective and dispassionate.

Explanation for Incorrect Answer E :  Choice (E) is incorrect. While marking a direct quotation from another source is one of the main uses of quotation marks, there is no indication here that the author of the passage is deriving the word “attract” from another source.

10 How does the theory described in lines 3-7 primarily differ from the other two theories described in the passage?

ANSWERS AND EXPLANATIONS Explanation for Correct Answer E :  Choice (E) is correct. The theory described in lines 3-7 states that the human appendix may originally have harbored colonies of bacteria to aid in digestion, which is its purpose “in present-day herbivores.” Neither of the other theories in the passage—in lines 7-12 and 12-16—refers to present-day animals, herbivorous or carnivorous.

Explanation for Incorrect Answer A :  Choice (A) is incorrect. None of the three theories described in the passage pertains only to plants. The first theory mentions plants, but only in discussing the digestion of cellulose by herbivores.

Explanation for Incorrect Answer B :  Choice (B) is incorrect. It is the second theory (lines 7-12), not the theory in lines 3-7, that deals with a process possibly occurring in two areas of the human body: the production of lymphocytes in the appendix and the tonsils.

Explanation for Incorrect Answer C :  Choice (C) is incorrect. There is no indication that any of the three theories described in the passage enjoys more scientific support than do the others. The only mention of experts is in line 3 and pertains to all the theories.

Explanation for Incorrect Answer D :  Choice (D) is incorrect. Both the second and third theories could be said to pertain to disease prevention—the second by aiding in the production of antibodies, the third by isolating infections—so disease prevention cannot be said to be represented in the first theory only.

(A) It pertains only to plants.

(B) It concerns a physical process that occurs in more than one area of the human body.

(C) It is a theory supported by more experts in the field than are the other two theories.

(D) It is concerned with the prevention of disease.

(E) It makes reference to a process presently occurring in other animals.

       One hazard in historical study is  the necessity of dividing the whole into  segments, since not everything can beLine examined simultaneously. Common ways5 of dividing history are by period, country,  topic, artistic or political movement, or  theme. Each of these can be justified, but

页码,9/23The Official SAT Online Course

2006-11-12file://E:\新建文件夹\e3.htm

UnRe

gistered

Page 11: SAT practice test 5.pdf

严禁用于商业用途!

  all have their shortcomings. When  divisions are made according to country,10 the interconnections among events  occurring in two or more countries may go  unnoticed or remain unexplored. Division  into time periods may interrupt or obscure  ongoing developments, or may give undue15 emphasis to some event or type of activity  (especially war or politics) as crucial in  marking the end or beginning of a period  or movement.

11 Which of the following is most analogous to the “hazard” the author sees in the “division” of historical study?

ANSWERS AND EXPLANATIONS Explanation for Correct Answer E :  Choice (E) is correct. The author argues in this passage that dividing history into segments for instructional purposes is always risky. No matter how history is divided, distortions of emphasis will occur. Focusing on one part of history—an international period or a single nation’s history, for instance—leaves larger connections unexplored. This type of “hazard” is analogous to the challenge doctors face when assessing a patient’s health. A doctor might make a specific diagnosis associated with an ailing portion of a patient’s body in order to recommend treatment, but may inadvertently overlook larger, more global elements of the patient’s health.

Explanation for Incorrect Answer A :  Choice (A) is incorrect. The author argues in this passage that dividing history into segments for instructional purposes is always risky. No matter how history is divided, distortions of emphasis will occur. Focusing on one part of history—an international period or a single nation’s history, for instance—leaves larger connections unexplored. A lawyer accepting cases in too many different areas of legal practice would be a reverse of the “hazard” in historical studies: the lawyer would be spread too thin by too wide a focus, instead of missing the bigger picture because of a specializing too narrowly.

Explanation for Incorrect Answer B :  Choice (B) is incorrect. The author argues in this passage that dividing history into segments for instructional purposes is always risky. No matter how history is divided, distortions of emphasis will occur. Focusing on one part of history—an international period or a single nation’s history, for instance—leaves larger connections unexplored. Although coping with a large class size could be similar to dealing with large portions of history, the inability to give individual students attention is not the same type of “hazard” the author mentions in historical studies. The author’s point is that too much attention is given to individual sections of history at the expense of the bigger picture.

Explanation for Incorrect Answer C :  Choice (C) is incorrect. The author argues in this passage that dividing history into segments for instructional purposes is always risky. No matter how history is divided, distortions of emphasis will occur. Focusing on one part of history—an international period or a single nation’s history, for instance—leaves larger connections unexplored. A biologist studying large areas of forest at the expense of focusing on a specific bird species would be the reverse of the “hazard” in historical studies: the biologist would overlook details because of too wide a focus,

(A) A lawyer accepts cases in too many different areas of legal practice.

(B) A teacher must cope with large class sizes and is unable to give students sufficient individual instruction.

(C) A biologist studies large areas of forest but fails to examine in depth the nesting site of a specific bird species.

(D) An artist produces works in many different media, but does not excel in any one medium.

(E) A doctor diagnoses one ailment but overlooks elements of the patient’s overall health.

页码,10/23The Official SAT Online Course

2006-11-12file://E:\新建文件夹\e3.htm

UnRe

gistered

Page 12: SAT practice test 5.pdf

严禁用于商业用途!

instead of overlooking the big picture because of a narrow focus.

Explanation for Incorrect Answer D :  Choice (D) is incorrect. The author argues in this passage that dividing history into segments for instructional purposes is always risky. No matter how history is divided, distortions of emphasis will occur. Focusing on one part of history—an international period or a single nation’s history, for instance—leaves larger connections unexplored. An artist working in many different media would be the reverse of the “hazard” in historical studies: the artist would be spread too thin by too wide a focus, instead of missing the bigger picture because of specializing too narrowly.

12

The author implies which of the following about “war” and “politics” in historical studies (line 16)?

ANSWERS AND EXPLANATIONS Explanation for Correct Answer E :  Choice (E) is correct. In this passage, the author states that dividing history into “time periods” may overemphasize “some event or type of activity” commonly used to mark “the end or beginning of a period or movement.” The author includes “war or politics” as examples of types of activities that are used to mark the border of a historical period, that is, “the end or beginning of a period or movement.”

Explanation for Incorrect Answer A :  Choice (A) is incorrect. In this passage, the author mentions several ways in which history is divided for instructional purposes. “War” and “politics” are used as examples of events or activities used to define historical periods. While the author says that all such divisions have their shortcomings, he or she does not imply that these particular divisions—war and politics—“make the study of international movements difficult.” Such a division might, it could be argued, make the study of international movements easier than would, say, division according to nation.

Explanation for Incorrect Answer B :  Choice (B) is incorrect. In this passage, the author mentions several ways in which history is divided for instructional purposes. “War” and “politics” are used as examples of events or activities used to define historical periods. While the author says that all such divisions have their shortcomings, he or she does not imply that these particular divisions—war and politics—“serve primarily as a convenience to the reader.” In fact, the author seems to imply that such division may not be a convenience, but may actually inconvenience the reader by distorting the importance of particular events.

Explanation for Incorrect Answer C :  Choice (C) is incorrect. In this passage, the author mentions several ways in which history is divided for instructional purposes. “War” and “politics” are used as examples of events or activities commonly used to define historical periods. The author does not imply that these particular divisions—war and politics—are more useful in defining periods than in defining movements. Rather, the author states that all such divisions introduce certain hazards and oversights into historical study.

Explanation for Incorrect Answer D :  Choice (D) is incorrect. In this passage, the author mentions several ways in which history is divided for instructional purposes. “War” and “politics” are used as examples of events or activities commonly used to define historical periods. The author does not discuss the importance of these particular divisions—war and politics—to either the historian who writes the history or the reader who reads history; the author says only that such divisions are common and potentially hazardous.

(A) They make the study of international movements difficult.

(B) They serve primarily as a convenience to the reader.

(C) They are more helpful to use in defining periods than in defining movements.

(D) They are equally important to historians and to readers.

(E) They are commonly used to define historical periods.

页码,11/23The Official SAT Online Course

2006-11-12file://E:\新建文件夹\e3.htm

UnRe

gistered

Page 13: SAT practice test 5.pdf

严禁用于商业用途!

The following passages discuss a type of film called film noir, which, according to most film historians, had its high point around the time of the Second World War (1939-1945).

Passage 1

       Even though films now called film  noir by critics have been made in  Hollywood since 1939, film noir as a genreLine did not exist until 1946. In that year an5 exhibition of American movies was held in  Paris, and French film critics got their first  look at what had been going on in  Hollywood since the advent of World War  II. Among the films shown were Laura;10 The Maltese Falcon; Murder, My Sweet;  Double Indemnity; and The Woman in the  Window. Those five films shared enough  traits that critic Nino Frank gave them a  new classification: film noir, or literally,15 “black film.” The traits they shared were  both stylistic and thematic. They were  dark in both look and mood. Their  primary action took place at night on rain-  swept city streets, in narrow ash-can20 alleys, in claustrophobic diners, and in  dingy, shadowy hotel rooms with neon  signs flashing outside the windows, rooms  in which, as hard-boiled author Nelson  Algren once put it, “every bed you rent25 makes you an accessory to somebody  else’s shady past.” The characters in  these films were bookies, con men, killers,  cigarette girls, crooked cops, down-and-  out boxers, and calculating, scheming,30 and very deadly women. The well-lit,  singing and tap-dancing, happy-ending  world of the 1930’s had in ten short years  become a hostile, orderless place in which  alienation, obsession, and paranoia ruled.35 The universe seemed to conspire to defeat  and entrap the inhabitants who wandered  blindly through it. They were victims of  fate, their own worst enemies who,  looking for a score, ended by defeating40 themselves.       The five films mentioned earlier  that were shown at the 1946 exhibition  were the ones the French critiqued. These  high-budget studio productions most45 commonly come to the public’s mind when  the word noir is mentioned because they

页码,12/23The Official SAT Online Course

2006-11-12file://E:\新建文件夹\e3.htm

UnRe

gistered

Page 14: SAT practice test 5.pdf

严禁用于商业用途!

Passage 2

  are cited most often in the spate of  contemporary books that have recently  been published on the subject. But the50 noir cycle, although kick-started by the  success of those high-budget productions,  actually had its roots in the B movie, in  particular, in the B crime movie. Film noir  was made to order for the B, or low-55 budget, part of the movie double bill1. It  was cheaper to produce because it  required less lighting and smaller casts  and usually entailed story lines that  required limited-scale sets—an attractive60 quality to film studios operating on  reduced wartime budgets. Film noir was  character-driven, and its story lines, which  were unusual and compact, could often be  told in the 60 to 80 minutes required of B65 pictures.

       It may be that noir began in a way  of photographing that was as economical  as it was moody (less light meant less  money on decor—an important wartime70 consideration when studios faced limits on  construction material).       Where did noir come from? It’s an  intriguing question and one still not  adequately answered, despite the quantity75 of writing that wallows in that noir mood.  Don’t rule out the influence of German  film from the twenties, if only because  there were, by the early forties, so many  European refugees (writers, directors,80 camera operators, designers, actors)  working in Hollywood. Don’t forget the  impact of French films of the late thirties,  especially those of Marcel Carne. His Le  Jour Se Leve (1939, called Daybreak in85 the United States) was such a success  that it was remade in Hollywood in 1947  as The Long Night. Finally, don’t  underestimate the influence Citizen Kane  had on anyone whose art and craft was90 cinematography. The film was a box  office flop, but filmmakers were absorbed  by it. A landmark in so much, Kane is a  turning point in the opening up of a noir  sensibility.95      Equally, don’t forget that from the

页码,13/23The Official SAT Online Course

2006-11-12file://E:\新建文件夹\e3.htm

UnRe

gistered

Page 15: SAT practice test 5.pdf

严禁用于商业用途!

  forties onward, Los Angeles was much  beset by psychoanalysis, and the growing  intellectual interest in guilt, depression,  and nightmare. Don’t eliminate the100 impact, the memory, or the mere thought  of a war’s damage.       I want to stress how deeply noir  impulses lay in the common imagination—  that of the audience as well as the105 filmmakers. Mildred Pierce, for instance—  which appeared in 1945 when many  American women were running businesses  of their own just to survive while the men  were away at war—sighs and seems to110 say, “It doesn’t make any difference, why  bother, for there is something malign in  human nature or luck that will undermine  enterprise and hope.” Of course, not  every film was so bleak. Look instead at115 David Selznick’s very beautiful and  touching Since You Went Away, which is  all about Claudette Colbert, Jennifer  Jones, and Shirley Temple

2 coping in the

  absence of men. That, too, looks like a120 film noir. But the mood is entirely that of  innocent, ardent, flawless hope, and  assurance that when the war ends  everything will revert to calm and order.

 

1 In the 1940’s, a film showing would typically include a longer, more expensively-produced film (an “A film”) and a shorter, less-expensively produced film (a “B film”)—in other words, a “double bill.” 2 Film actresses who were popular during the 1940’s.

13

In lines 1 through 4 of Passage 1, the author suggests that “films now called film noir by critics”

ANSWERS AND EXPLANATIONS Explanation for Correct Answer A :  Choice (A) is correct. The passage states that while films now classified as film noir had been made since 1939, “film noir as a genre did not exist until 1946.” Thus the “films now called film noir by critics” were not classified as film noir when they were first made, since the term was not coined until after the Paris exhibition.

(A) were not classified as film noir when first made

(B) were reminiscent of earlier European films

(C) were uplifting in mood and theme

(D) were intended to contrast with films of the 1930’s

(E) were disliked by many French film critics

页码,14/23The Official SAT Online Course

2006-11-12file://E:\新建文件夹\e3.htm

UnRe

gistered

Page 16: SAT practice test 5.pdf

严禁用于商业用途!

Explanation for Incorrect Answer B :  Choice (B) is incorrect. The first four lines of the passage state that “films now called film noir” were made before film noir had been recognized as its own genre. So while the passage does speak of films “earlier than 1946,” nothing is suggested here about the relationship between these American films and “earlier European films.”

Explanation for Incorrect Answer C :  Choice (C) is incorrect. The first four lines of the passage state that “films now called film noir” were made before film noir had been recognized as its own genre. It stands to reason, then, that since these films were film noir in everything but name—since the name had not yet been invented—they were anything but “uplifting in mood and theme.” Rather, they were almost certainly bleak in mood and theme.

Explanation for Incorrect Answer D :  Choice (D) is incorrect. The first four lines of the passage state that “films now called film noir” were made before film noir had been recognized as its own genre. While it is suggested in lines 30-32 that certain films of the 1930’s (“well-lit, singing and tap-dancing, happy-ending”) did contrast markedly with the film noir, there is nothing in the passage to indicate that either type of film was intended to contrast with the other.

Explanation for Incorrect Answer E :  Choice (E) is incorrect. The first four lines of the passage state that “films now called film noir” were made before film noir had been recognized as its own genre. If these films were later termed film noir, a term coined by French critics, it stands to reason that French critics liked rather than disliked them.

14It can be inferred that the films listed in lines 9 through 12 were similar in each of the following ways EXCEPT:

ANSWERS AND EXPLANATIONS Explanation for Correct Answer E :  Choice (E) is correct. The passage discusses many similarities among the five films mentioned in lines 9 through 12; they “shared enough traits that critic Nino Frank gave them a new classification.” The passage states that the films were similar in “look and mood,” that they shared “thematic” traits, and that many of them featured the same types of characters. While the music in these films may or may not be similar, music is not discussed in the passage, and so no inferences can be drawn from the passage about it.

Explanation for Incorrect Answer A :  Choice (A) is incorrect. The passage states that the five films mentioned in lines 9 through 12 were “dark in both look and mood,” which implies that they were indeed similar in visual appearance.

Explanation for Incorrect Answer B :  Choice (B) is incorrect. The passage states that the five films mentioned in lines 9 through 12 were “dark in both look and mood”; further, the world of these films was “a hostile, orderless place in which alienation, obsession, and paranoia ruled.” Together, these descriptions imply that the films were indeed similar in emotional effect.

Explanation for Incorrect Answer C :  Choice (C) is incorrect. The passage explicitly states that the films shared similar types of characters. In lines 26- 29, several character types are listed, including

(A) visual appearance

(B) emotional effect

(C) characters

(D) theme

(E) music

页码,15/23The Official SAT Online Course

2006-11-12file://E:\新建文件夹\e3.htm

UnRe

gistered

Page 17: SAT practice test 5.pdf

严禁用于商业用途!

“crooked cops, down-and-out boxers, and calculating, scheming, and very deadly women.”

Explanation for Incorrect Answer D :  Choice (D) is incorrect. The passage states in line 16 that the traits the films shared “were both stylistic and thematic”—in other words, that they had similarities in theme.

15 In paragraph 2, the author says that the films discussed in paragraph 1 are NOT typical of their genre in regard to

ANSWERS AND EXPLANATIONS Explanation for Correct Answer B :  Choice (B) is correct. Paragraph 2 discusses the budgets of film noir films. The author argues that although the “high-budget productions” mentioned in the first paragraph helped kick-start film noir, the roots of film noir were in B movies. B movies, the passage states, were “low-budget,” “cheaper to produce” films used as the less expensive half of double features. The higher budget is the only difference presented here between the films mentioned in paragraph 1 and typical films of the film noir genre.

Explanation for Incorrect Answer A :  Choice (A) is incorrect. Although paragraph 2 states that typical film noir “entailed story lines that required limited-scale sets,” this is not mentioned as a difference between the typical film noir and the films mentioned in paragraph 1. The author’s point is not that these settings were different from those of the films mentioned, but that film noir did not, in general, require a large budget to produce.

Explanation for Incorrect Answer C :  Choice (C) is incorrect. Paragraph 2 says nothing about film noir’s country of origin, though the passage as a whole calls film noir an American phenomenon first named by French film critics. Rather, paragraph 2 discusses the low budgets of the typical film noir.

Explanation for Incorrect Answer D :  Choice (D) is incorrect. Although paragraph 2 states that typical film noir plots were “character-driven” and “unusual and compact,” this is not mentioned as a difference between the typical film noir and the films mentioned in paragraph 1. The author’s point is not that these plots were different from those of the films mentioned, but that film noir did not, in general, require a high budget to produce.

Explanation for Incorrect Answer E :  Choice (E) is incorrect. Paragraph 2 says nothing about film noir’s lighting, though the passage as a whole indicates that the typical film noir was dimly lighted. Rather, paragraph 2 discusses the low budgets of typical film noir.

(A) setting

(B) budget

(C) country of origin

(D) plot

(E) lighting

16

The author of Passage 1 uses the quotation in lines 24-26 primarily in order to

(A) critique a writer

(B) recount an incident

(C) evoke a place

(D) describe a character

(E) summarize a plot

页码,16/23The Official SAT Online Course

2006-11-12file://E:\新建文件夹\e3.htm

UnRe

gistered

Page 18: SAT practice test 5.pdf

严禁用于商业用途!

ANSWERS AND EXPLANATIONS Explanation for Correct Answer C :  Choice (C) is correct. In the second half of the first paragraph, the author describes the characteristics of film noir, including setting: the “claustrophobic diners” and “dingy, shadowy hotel rooms” where film noir is commonly set. The author quotes Nelson Algren’s description of the hotels in which “every bed you rent makes you an accessory to somebody else’s shady past” in order to evoke strongly the typical places where action occurs in film noir.

Explanation for Incorrect Answer A :  Choice (A) is incorrect. In the second half of the first paragraph, the author describes the characteristics of film noir, including setting. The author uses Nelson Algren’s quotation to help evoke the types of places typically found in film noir. There is no critique of Algren’s writing. In fact, the author seems to agree wholeheartedly with Algren’s description.

Explanation for Incorrect Answer B :  Choice (B) is incorrect. In the second half of the first paragraph, the author describes the characteristics of film noir, including setting. The author uses Nelson Algren’s quotation to help evoke the types of places found in film noir. The quote does not recount a specific incident, but, rather, refers to film noir settings in general.

Explanation for Incorrect Answer D :  Choice (D) is incorrect. In the second half of the first paragraph, the author describes the characteristics of film noir, including setting. The author uses Nelson Algren’s quotation to help evoke the type of places generally found in film noir, not to describe any specific character, though character types are discussed in the first paragraph of the passage.

Explanation for Incorrect Answer E :  Choice (E) is incorrect. In the second half of the first paragraph, the author describes the characteristics of film noir, including setting. The author uses Nelson Algren’s quotation to help evoke the types of places generally found in film noir, not to summarize the plot of any specific film.

17

The author of Passage 1 suggests that the “spate of contemporary books” (lines 47-48) on film noir

ANSWERS AND EXPLANATIONS Explanation for Correct Answer C :  Choice (C) is correct. The author states that the “spate of contemporary books” on film noir cites “most often” the five films “shown at the 1946 exhibition.” These five films, consequently, have dominated public perception of film noir. The author argues that this perception of film noir may be inaccurate, however, because the films most discussed in the books were much more costly to produce than were most film noir films. According to the author, film noir films were very often “low-budget” B movies.

Explanation for Incorrect Answer A :  Choice (A) is incorrect. The author states that the “spate of contemporary books” on film noir cites “most often” the five films “shown at the 1946 exhibition.” The author does not state that these five films are the only films discussed in contemporary writing about film noir.

(A) discusses only five films

(B) focuses on non-crime films

(C) focuses on relatively costly noir films

(D) is inaccurate in their historical data

(E) is from big-budget publishing companies

页码,17/23The Official SAT Online Course

2006-11-12file://E:\新建文件夹\e3.htm

UnRe

gistered

Page 19: SAT practice test 5.pdf

严禁用于商业用途!

Explanation for Incorrect Answer B :  Choice (B) is incorrect. The author states that the “spate of contemporary books” on film noir cites “most often” the five films “shown at the 1946 exhibition.” Although the author states that the roots of film noir were in “the B crime movie” and that the five high-budget films are uncharacteristic, this does not imply that the five films were not crime films themselves.

Explanation for Incorrect Answer D :  Choice (D) is incorrect. The author states that the “spate of contemporary books” on film noir cites “most often” the five films “shown at the 1946 exhibition.” Although the author argues that focusing on these five films produces a slightly inaccurate understanding of film noir, the historical data is not termed inaccurate. The author agrees that these films were shown at the exhibition and that the exhibition was vital to the establishment of film noir as a genre.

Explanation for Incorrect Answer E :  Choice (E) is incorrect. Although the author states that the “spate of contemporary books” focuses on films that were “high-budget studio productions,” nothing is said in the passage about the companies that published the books.

18 The authors of both passages imply that contemporary writing about film noir

ANSWERS AND EXPLANATIONS Explanation for Correct Answer B :  Choice (B) is correct. Both authors argue that contemporary writing on film noir fails to describe its origins accurately. The author of Passage 1 argues that “the spate of contemporary books” on film noir inaccurately portrays film noir’s origins by focusing on five “high-budget” films “shown at the 1946 exhibition.” According to the author, this focus de-emphasizes the fact that film noir “actually had its roots in the B movie,” which were much lower-budget productions. The author of Passage 2 states that “the quantity of writing that wallows in that noir mood” has “not adequately answered” the question, “Where did noir come from?” Thus, both authors think that many modern writers ignore or miss the point about the real origins of film noir.

Explanation for Incorrect Answer A :  Choice (A) is incorrect. Neither author mentions the national origin of contemporary writing about film noir. The author of Passage 1 discusses the importance French film critics played in establishing the film noir genre in 1946, but says nothing in specific about contemporary writing by French critics.

Explanation for Incorrect Answer C :  Choice (C) is incorrect. Both authors state that contemporary writing about film noir mischaracterizes the origins of film noir, not the mood of film noir. The authors seem to agree on the dark “mood” of film noir.

Explanation for Incorrect Answer D :  Choice (D) is incorrect. While both authors criticize contemporary writing as wrongheaded about the origins of film noir, neither author compares the quality of this writing with that of earlier writing.

Explanation for Incorrect Answer E :  Choice (E) is incorrect. Neither author states that contemporary writing has improved the understanding of film noir. On the contrary, they both argue that contemporary writing has inadequately described the origins of film noir.

(A) comes mainly from French film critics

(B) has failed to describe the origins of film noir accurately

(C) mischaracterizes the film noir mood

(D) is inferior to earlier writing on film noir

(E) has dramatically improved the understanding of film noir

19Both passages imply that the development of film noir can be attributed in part to

页码,18/23The Official SAT Online Course

2006-11-12file://E:\新建文件夹\e3.htm

UnRe

gistered

Page 20: SAT practice test 5.pdf

严禁用于商业用途!

ANSWERS AND EXPLANATIONS Explanation for Correct Answer C :  Choice (C) is correct. Both passages highlight the relatively low cost of making most film noir films. They both state also that this low cost was central to film noir production because low cost was consistent with economic restraints resulting from World War II. Passage 1 states that the “limited-scale sets” of film noir were “an attractive quality to film studios operating on reduced wartime budgets.” Passage 2 states that the low lighting that contributed to the mood of film noir was also cheaper, “an important wartime consideration when studios faced limits on construction material.”

Explanation for Incorrect Answer A :  Choice (A) is incorrect. Although Passage 2 claims the presence of European filmmakers in Hollywood was influential in the development of film noir, Passage 1 speaks of French film critics but makes no mention of European filmmakers in Hollywood.

Explanation for Incorrect Answer B :  Choice (B) is incorrect. Although Passage 1 discusses the importance of French film critics in the establishment of film noir as a genre, Passage 2 speaks of European film directors but does not mention French film critics at all.

Explanation for Incorrect Answer D :  Choice (D) is incorrect. Neither passage discusses high-paid actors or United States filmmakers’ attitudes towards them.

Explanation for Incorrect Answer E :  Choice (E) is incorrect. Although Passage 2 states that Citizen Kane was influential in the development of film noir, Passage 1 does not mention Citizen Kane at all.

(A) the presence of European filmmakers in Hollywood

(B) the influential writing of French film critics

(C) economic restraints resulting from World War II

(D) United States filmmakers’ dissatisfaction with high-paid actors

(E) the popularity of Citizen Kane

20The phrase “wallows in” in line 75 is closest in meaning to

ANSWERS AND EXPLANATIONS Explanation for Correct Answer A :  Choice (A) is correct. In lines 72-75, Passage 2 discusses the abundant material written about film noir. The author is saying that writers have failed to explain accurately the origins of film noir, despite the large quantity of writing that “wallows in,” or is excessively involved with, film noir. In other words, the writing has been insufficient despite its indulgence in the topic.

Explanation for Incorrect Answer B :  Choice (B) is incorrect. In lines 72-75, Passage 2 discusses the abundant material written about film noir. The passage is not arguing that the writing “conforms to,” or shares, the film noir mood, but that it has insufficiently explained film noir’s origins despite its excessive involvement with the topic.

(A) indulges in

(B) conforms to

(C) criticizes

(D) explores

(E) reveals

页码,19/23The Official SAT Online Course

2006-11-12file://E:\新建文件夹\e3.htm

UnRe

gistered

Page 21: SAT practice test 5.pdf

严禁用于商业用途!

Explanation for Incorrect Answer C :  Choice (C) is incorrect. In lines 72-75, Passage 2 discusses the abundant material written about film noir. The passage is not stating that the writing “criticizes,” or disapproves of, film noir, but that it has insufficiently explained film noir’s origins despite its excessive involvement with the topic.

Explanation for Incorrect Answer D :  Choice (D) is incorrect. In lines 72-75, Passage 2 discusses the abundant material written about film noir. The passage is not simply stating that this material “explores” film noir, but that is has insufficiently explained film noir’s origins, despite its excessive involvement with the topic.

Explanation for Incorrect Answer E :  Choice (E) is incorrect. In lines 72-75, Passage 2 discusses the abundant material written about film noir. The passage is not stating that this material has “revealed” or exposed, anything about film noir. On the contrary, it states that the writing has failed to sufficiently explain film noir’s origins despite its excessive involvement with the topic.

21 The author of Passage 2 mentions Since You Went Away in line 116 primarily to

ANSWERS AND EXPLANATIONS Explanation for Correct Answer A :  Choice (A) is correct. The first half of the fifth paragraph discusses the apathetic and hopeless mood of many film noir films. The author then mentions Since You Went Away to point out that not every film of the time “was so bleak.” The author argues that Since You Went Away “looks like a film noir,” even though its mood is “entirely that of innocent, ardent, flawless hope,” an attitude quite different from film noir films previously discussed in the passage.

Explanation for Incorrect Answer B :  Choice (B) is incorrect. Although the author mentions that Since You Went Away starred certain popular film actors, the author’s point is that the innocent and hopeful attitude of Since You Went Away is different from the “bleak” mood of other films mentioned in the passage. The film’s cast is not necessarily relevant to the argument about the film’s different attitude.

Explanation for Incorrect Answer C :  Choice (C) is incorrect. Although the author states that Since You Went Away deals with women during World War II, the author’s point is that that the innocent and hopeful attitude of Since You Went Away is different from the “bleak” mood of other films mentioned in the passage. The author does not say that any other film noir films dealt with World War II, though it is likely that many did.

Explanation for Incorrect Answer D :  Choice (D) is incorrect. Although the author states that Since You Went Away portrayed women dealing with the “absence of men” during World War II, the author’s point is that the innocent and hopeful attitude of Since You Went Away is different from the “bleak” mood of other films mentioned in the passage. The author does not say that any other film noir films dealt with “the absence of men.”

Explanation for Incorrect Answer E :  Choice (E) is incorrect. The author does not state that David Selznick was influential in defining film noir, but only that Selznick’s film Since You Went Away had a different, more hopeful mood, than the “bleak” mood of other films mentioned in the passage.

(A) indicate that not all films in the 1940’s expressed the same attitude

(B) illustrate the popularity of film noir actors

(C) demonstrate that most film noir dealt with WWII

(D) show that the “absence of men” was a major film noir theme

(E) point out David Selznick’s influence in defining film noir

页码,20/23The Official SAT Online Course

2006-11-12file://E:\新建文件夹\e3.htm

UnRe

gistered

Page 22: SAT practice test 5.pdf

严禁用于商业用途!

22 The second paragraph of Passage 2 is best described as

ANSWERS AND EXPLANATIONS Explanation for Correct Answer A :  Choice (A) is correct. Paragraph 2 begins with the question, “Where did noir come from?” The paragraph then suggests different possible sources for film noir by exhorting the reader not to “rule out” or “forget,” or “underestimate” various influences. The paragraph catalogs “German film from the twenties,” “French films of the late thirties,” and “Citizen Kane” as influential in the formation of the film noir genre.

Explanation for Incorrect Answer B :  Choice (B) is incorrect. Paragraph 2 does not address the film noir mood specifically. Instead, it lists possible answers to the question, “Where did noir come from?” The paragraph presents sources of film noir, not elements of its mood.

Explanation for Incorrect Answer C :  Choice (C) is incorrect. Paragraph 2 does not address misconceptions about film noir. The paragraph states that determining the origins of film noir is difficult, and then lists possible sources for consideration. The sources are presented as possibilities, not as misconceptions.

Explanation for Incorrect Answer D :  Choice (D) is incorrect. Paragraph 2 does not attempt to define the term “film noir.” The paragraph assumes a general definition of film noir, then lists, or catalogs, several possible sources of film noir.

Explanation for Incorrect Answer E :  Choice (E) is incorrect. Although paragraph 2 does mention several films, these films are not presented as major film noir films. Instead, the paragraph lists them as possible sources for the style that became film noir.

(A) a catalog of possible sources of the film noir mood

(B) a description of the mood that characterizes film noir

(C) a refutation of several misconceptions about film noir

(D) an argument for redefining the term “film noir”

(E) a list of the major film noir films

23 The quote “It doesn’t make…enterprise and hope” in lines 110-113 is primarily meant to

ANSWERS AND EXPLANATIONS Explanation for Correct Answer D :  Choice (D) is correct. The passage argues that the “noir impulses” reflected the common sentiments of people in general during the 1940’s. The quotation in lines 110-113 characterizes the attitude expressed by the film Mildred Pierce. This attitude, the passage argues, was present in the imagination of many people at that time.

Explanation for Incorrect Answer A :  Choice (A) is incorrect. The quotation in lines 110-113 is not actual dialogue from Mildred Pierce. The quotation is, according to the author, a characterization of the

(A) give an example of dialogue from Mildred Pierce

(B) summarize what critics thought about Mildred Pierce

(C) show how Mildred Pierce differs from other films made in 1945

(D) characterize the sentiments expressed in Mildred Pierce

(E) demonstrate the influence of Mildred Pierce on Since You Went Away

页码,21/23The Official SAT Online Course

2006-11-12file://E:\新建文件夹\e3.htm

UnRe

gistered

Page 23: SAT practice test 5.pdf

严禁用于商业用途!

general attitude the film presents. This attitude, the passage argues, was shared by many people during the 1940’s.

Explanation for Incorrect Answer B :  Choice (B) is incorrect. The quotation in lines 110-113 does not refer to comments critics made about Mildred Pierce. The quotation is, according to the author, a characterization of the general attitude the film presents. This attitude, the passage argues, was shared by many people during the 1940’s.

Explanation for Incorrect Answer C :  Choice (C) is incorrect. The quotation is a characterization of the general attitude the film presents—an attitude, the passage argues, that was shared by many people during the 1940’s. Although the passage does state that “not every film was so bleak,” the paragraph argues that this attitude was the characteristic of many film noir films. The quotation is used to express a general sentiment in many film noir films, not a difference between Mildred Pierce and other films.

Explanation for Incorrect Answer E :  Choice (E) is incorrect. Although the paragraph discusses both Mildred Pierce and Since You Went Away, the quotation is used to help distinguish between the two films. The quotation in lines 110-113 characterizes the attitude expressed by the film Mildred Pierce. This attitude, the passage argues, was present in the imagination of many people at that time. The paragraph explicitly states that this attitude was NOT shared by Since You Went Away.

24The word “impulses” in line 103 is closest in meaning to

ANSWERS AND EXPLANATIONS Explanation for Correct Answer E :  Choice (E) is correct. The passage argues that “noir impulses” reflected the common sentiments of people in the 1940’s. The author implies that these “impulses,” or feelings, were shared by both filmmakers and filmgoers and that such feelings were present “in the common imagination” of the time. The quotation that the author uses to support this point—lines 110-113—is an expression of a pessimistic feeling about life.

Explanation for Incorrect Answer A :  Choice (A) is incorrect. The passage argues that the “noir impulses” reflected the common sentiments of people in the 1940’s. These “impulses” were the feelings people had about their lives, not “incentives,” or motivating factors, to impel them to do something.

Explanation for Incorrect Answer B :  Choice (B) is incorrect. The passage argues that the “noir impulses” reflected the common sentiments of people in the 1940’s. These “impulses” were the feelings people had about their lives, not “stimulants” or factors that would stimulate them to do something.

Explanation for Incorrect Answer C :  Choice (C) is incorrect. The passage argues that the “noir impulses” reflected the common sentiments of people in the 1940’s. These “impulses” were the feelings, not “fantasies,” many people had about their lives. In fact, the grim feelings expressed in the paragraph are the opposite of what is usually thought of as a fantasy, or pleasant notion or daydream.

Explanation for Incorrect Answer D :  Choice (D) is incorrect. The passage argues that the “noir impulses” reflected the common sentiments of people in the 1940’s. These “impulses” were the feelings people had about their lives, not any type of “transformation,” or profound change, they might have experienced.

(A) incentives

(B) stimulants

(C) fantasies

(D) transformations

(E) feelings

页码,22/23The Official SAT Online Course

2006-11-12file://E:\新建文件夹\e3.htm

UnRe

gistered

Page 24: SAT practice test 5.pdf

严禁用于商业用途!

    

Back to Score Report  

Copyright © 2006 The College Board. All rights reserved. Privacy Policy Terms of Use Contact Us

 

页码,23/23The Official SAT Online Course

2006-11-12file://E:\新建文件夹\e3.htm

UnRe

gistered

Page 25: SAT practice test 5.pdf

严禁用于商业用途!

Help | Profile | My Organizer | My Bookmarks | Logout

Answers and Explanations

Test Sections

Section 1

Section 2

Section 3

Section 4

Section 5

Section 6

Section 8

Section 9

Section 10

Back to Score Report  

View Answers and Explanations     Online - Practice Test #5

1 Which of the following triples does NOT satisfy

ANSWERS AND EXPLANATIONS Explanation for Correct Answer C : 

Choice (C) is correct. If then

Therefore, the triple does not

satisfy

Explanation for Incorrect Answer A : 

Choice (A) is not correct. If then

The question asks which triple does NOT satisfy

Explanation for Incorrect Answer B : 

Choice (B) is not correct. If then

The question asks which triple does NOT satisfy

Explanation for Incorrect Answer D : 

Choice (D) is not correct. If then

The question asks which triple does NOT

satisfy

Explanation for Incorrect Answer E : 

Choice (E) is not correct. If then

The question asks which triple does NOT

satisfy

(A)

(B)

(C)

(D)

(E)

2Susan had trading cards. After giving away cards and receiving cards, she

had cards. What is the value of

(A)

(B)

(C)

页码,1/17The Official SAT Online Course

2006-11-12file://E:\新建文件夹\e4.htm

UnRe

gistered

Page 26: SAT practice test 5.pdf

严禁用于商业用途!

ANSWERS AND EXPLANATIONS Explanation for Correct Answer A :  Choice (A) is correct. Susan had trading cards. After giving away cards, she

had cards; then, after receiving cards, she had cards. Since

Susan ended up with cards, it follows that Subtracting

from each side of this equation and rearranging gives

Explanation for Incorrect Answer B :  Choice (B) is not correct. The value of is not

Explanation for Incorrect Answer C :  Choice (C) is not correct. The value of is not

Explanation for Incorrect Answer D :  Choice (D) is not correct. The value of is not

Explanation for Incorrect Answer E :  Choice (E) is not correct. The value of is not

(D)

(E)

3If is a positive integer divisible by and if what is the greatest possible

value of

ANSWERS AND EXPLANATIONS Explanation for Correct Answer B : 

Choice (B) is correct. If then is divisible by because

In addition, satisfies the condition There are no integers greater

than and less than that are divisible by Therefore, is the greatest possible integer that satisfies both conditions.

Explanation for Incorrect Answer A :  Choice (A) is not correct. The integer is not divisible by Therefore,

cannot be the greatest possible integer less than that is divisible by

Explanation for Incorrect Answer C :  Choice (C) is not correct. The integer is not divisible by Therefore,

cannot be the greatest possible integer less than that is divisible by

Explanation for Incorrect Answer D :  Choice (D) is not correct. The integer is not divisible by Therefore,

cannot be the greatest possible integer less than that is divisible by

Explanation for Incorrect Answer E :  Choice (E) is not correct. The integer is not divisible by Therefore,

cannot be the greatest possible integer less than that is divisible by

(A)

(B)

(C)

(D)

(E)

页码,2/17The Official SAT Online Course

2006-11-12file://E:\新建文件夹\e4.htm

UnRe

gistered

Page 27: SAT practice test 5.pdf

严禁用于商业用途!

4

The graph above shows the monthly sales for Company and Company for the second half of What was the largest one-month decrease in sales for Company

during this period of time?

ANSWERS AND EXPLANATIONS Explanation for Correct Answer E :  Choice (E) is correct. The sales for Company are represented by the dashed line,

so the sales for the six months for that company were

and Therefore, the largest one-

month decrease in sales for Company was

Explanation for Incorrect Answer A : 

Choice (A) is not correct. This choice, was the difference in sales

between Company and Company for the month of August. The question asks for the largest one-month decrease in sales for Company

Explanation for Incorrect Answer B : 

Choice (B) is not correct. This choice, was the largest one-month

increase in sales for Company The question asks for the largest one-month decrease in sales for Company

Explanation for Incorrect Answer C : 

Choice (C) is not correct. This choice, was the largest one-month

decrease in sales for Company The question asks for the largest one-month

decrease in sales for Company

Explanation for Incorrect Answer D : 

Choice (D) is not correct. This choice, was the largest one-month

increase in sales for Company The question asks for the largest one-month

decrease in sales for Company

(A)

(B)

(C)

(D)

(E)

页码,3/17The Official SAT Online Course

2006-11-12file://E:\新建文件夹\e4.htm

UnRe

gistered

Page 28: SAT practice test 5.pdf

严禁用于商业用途!

5 For parties, the number of cases of juice a caterer orders is directly proportional to the number of people attending. If the caterer orders cases for a party with people attending, how many cases would she order for a party with people attending?

ANSWERS AND EXPLANATIONS Explanation for Correct Answer E :  Choice (E) is correct. Let represent the number of cases of juice the caterer

would order for a party with people attending. Then and

Explanation for Incorrect Answer A : 

Choice (A) is not correct. This choice is but the correct number of cases for

people is

Explanation for Incorrect Answer B :  Choice (B) is not correct. Let represent the number of cases of juice the caterer

would order for a party with people attending. Then and

not

Explanation for Incorrect Answer C : 

Choice (C) is not correct. This choice is but the correct number of cases

for people is

Explanation for Incorrect Answer D :  Choice (D) is not correct. Let represent the number of cases of juice the caterer

would order for a party with people attending. Then and

not

(A)

(B)

(C)

(D)

(E)

6

Which of the following is an element of both the set of positive odd integers and the set of prime numbers?

(A)

(B)

(C)

(D)

(E)

页码,4/17The Official SAT Online Course

2006-11-12file://E:\新建文件夹\e4.htm

UnRe

gistered

Page 29: SAT practice test 5.pdf

严禁用于商业用途!

ANSWERS AND EXPLANATIONS Explanation for Correct Answer B :  Choice (B) is correct. The integer is odd. In addition, is greater than and

has no positive integer factors other than itself and so is a prime number.

Therefore, is an element of both the set of positive odd integers and the set of prime numbers.

Explanation for Incorrect Answer A :  Choice (A) is not correct. The number is even, not odd.

Explanation for Incorrect Answer C :  Choice (C) is not correct. Since the number is not prime.

Explanation for Incorrect Answer D :  Choice (D) is not correct. The number is even, not odd. In addition, since

the number is not prime.

Explanation for Incorrect Answer E :  Choice (E) is not correct. Since the number is not prime.

7

The length of a rectangular garden is feet more than its width. If the length of the garden is feet, what is the area of the garden in square feet?

ANSWERS AND EXPLANATIONS Explanation for Correct Answer C :  Choice (C) is correct. The length of the garden is feet. Since the length is feet

more than the width, it follows that the width of the garden is feet. Therefore,

the area of the garden, in square feet, is

Explanation for Incorrect Answer A :  Choice (A) is not correct. The length of the garden is feet. The width of the

garden is feet less than the length, or feet. Therefore, the area of the

garden, in square feet, is not

Explanation for Incorrect Answer B :  Choice (B) is not correct. The width of the garden is feet, but the length is

feet. Therefore, the area of the garden, in square feet, is not

Explanation for Incorrect Answer D :  Choice (D) is not correct. The length of the garden is feet, but the width is

feet. Therefore, the area of the garden, in square feet, is

not

Explanation for Incorrect Answer E :  Choice (E) is not correct. The length of the garden is feet. The width of the

garden is feet less than the length ( feet), not feet more ( feet).

Therefore, the area of the garden, in square feet, is not

(A)

(B)

(C)

(D)

(E)

页码,5/17The Official SAT Online Course

2006-11-12file://E:\新建文件夹\e4.htm

UnRe

gistered

Page 30: SAT practice test 5.pdf

严禁用于商业用途!

8

On the number line above, the tick marks are equally spaced. What is the value of

ANSWERS AND EXPLANATIONS Explanation for Correct Answer E :  Choice (E) is correct. The value of is the distance between and From the figure, we can see that this is the same as the distance between two adjacent tick marks. Since the equally spaced tick marks divide the interval from to into

equal subintervals, the distance between two adjacent tick marks is

Explanation for Incorrect Answer A :  Choice (A) is not correct. The value of is the distance between two adjacent

tick marks. Since the equally spaced tick marks divide the interval from to into

equal subintervals, the distance between two adjacent tick marks is

not

Explanation for Incorrect Answer B :  Choice (B) is not correct. The value of is the distance between two adjacent

tick marks. Since the equally spaced tick marks divide the interval from to into

equal subintervals, the distance between two adjacent tick marks is

not

Explanation for Incorrect Answer C :  Choice (C) is not correct. This choice is the value of but the question asks for the value of

Explanation for Incorrect Answer D :  Choice (D) is not correct. The value of is the distance between two adjacent

tick marks. Since the equally spaced tick marks divide the interval from to into

equal subintervals, the distance between two adjacent tick marks is

not This choice could be the result of miscounting the subintervals between

and

(A)

(B)

(C)

(D)

(E)

9Jamal has some coins in his pocket. Some of these coins are quarters, and none of

页码,6/17The Official SAT Online Course

2006-11-12file://E:\新建文件夹\e4.htm

UnRe

gistered

Page 31: SAT practice test 5.pdf

严禁用于商业用途!

the quarters in his pocket are dated earlier than Which of the following must be true?

ANSWERS AND EXPLANATIONS Explanation for Correct Answer C :  Choice (C) is correct. Jamal has some quarters in his pocket, and none of them are dated earlier than Therefore, all of the quarters in his pocket must be dated

or later. This means that the statement “Some of the coins in Jamal’s

pocket are dated or later” MUST be true.

Explanation for Incorrect Answer A :  Choice (A) is not correct. It may be true that some of the coins in Jamal’s pocket are not quarters. We know that none of the quarters in Jamal’s pocket are dated

earlier than but we have no information about the other coins. Therefore, the statement “None of the coins in Jamal’s pocket are dated earlier than

” is NOT necessarily true.

Explanation for Incorrect Answer B :  Choice (B) is not correct. We know that none of the quarters in Jamal’s pocket are

dated earlier than and we know that Jamal may have some coins in his pocket that are not quarters. We know that none of the quarters are dated earlier than and it is possible than none of the other coins are dated earlier than

either. Therefore, the statement “Some of the coins in Jamal’s pocket are

dated earlier than ” is NOT necessarily true.

Explanation for Incorrect Answer D :  Choice (D) is not correct. We know that Jamal may have some coins in his pocket that are not quarters, but we do not know how many coins are quarters and how many coins are not quarters. We also do not know anything about the dates of the coins that are not quarters. Therefore, the statement “Most of the coins in Jamal’s pocket are either quarters or dated earlier than ” may or may not be true.

Explanation for Incorrect Answer E :  Choice (E) is not correct. Although we know that Jamal has some quarters in his pocket and possibly some coins that are not quarters, we do not know how many of the coins are quarters and how many are not quarters. Therefore, the statement “Most of the coins in Jamal’s pocket are not quarters” is NOT necessarily true.

(A) None of the coins in Jamal’s pocket are dated earlier than

(B) Some of the coins in Jamal’s pocket are dated earlier than

(C) Some of the coins in Jamal’s pocket are dated or later.

(D) Most of the coins in Jamal’s pocket are either quarters or dated earlier than

(E) Most of the coins in Jamal’s pocket are not quarters.

10

The circumference of the circle with center shown above is What is the area of the shaded region?

(A)

页码,7/17The Official SAT Online Course

2006-11-12file://E:\新建文件夹\e4.htm

UnRe

gistered

Page 32: SAT practice test 5.pdf

严禁用于商业用途!

ANSWERS AND EXPLANATIONS Explanation for Correct Answer B :  Choice (B) is correct. The circumference of a circle is times its diameter. Thus, the circle, which has circumference has diameter Hence, the radius of the

circle is and the area of the entire circle is Since the two diameters

in the figure are perpendicular, the shaded region is of the circle.

Therefore, the area of the shaded region is

Explanation for Incorrect Answer A : 

Choice (A) is not correct. is one quarter of the circumference, but the question

asks for the area of one quarter of the circle.

Explanation for Incorrect Answer C :  Choice (C) is not correct. is the radius of the circle, but the question asks for the area of one quarter of the circle.

Explanation for Incorrect Answer D : 

Choice (D) is not correct. is one quarter of the diameter, but the question asks

for the area of one quarter of the circle.

Explanation for Incorrect Answer E :  Choice (E) is not correct. The shaded area is one quarter of the circle, but since the

circle has area the area of one quarter of the circle is

(B)

(C)

(D)

(E)

11

If and are positive integers, and if which of the following must be true?

ANSWERS AND EXPLANATIONS Explanation for Correct Answer D :  Choice (D) is correct. If the product of two numbers is then at least one of the

numbers must be equal to Thus, if then either or

(or both). Since is a positive integer, It follows that Therefore, it must be true that

(A)

(B)

(C)

(D)

(E)

页码,8/17The Official SAT Online Course

2006-11-12file://E:\新建文件夹\e4.htm

UnRe

gistered

Page 33: SAT practice test 5.pdf

严禁用于商业用途!

Explanation for Incorrect Answer A :  Choice (A) is not correct. If and then and are positive

integers such that yet it is not true that

Explanation for Incorrect Answer B :  Choice (B) is not correct. If and then and are positive

integers such that yet it is not true that

Explanation for Incorrect Answer C :  Choice (C) is not correct. If and then and are positive

integers such that yet it is not true that

Explanation for Incorrect Answer E :  Choice (E) is not correct. If and then and are positive

integers such that yet it is not true that

12

In the figure above, regular pentagon is divided into three nonoverlapping triangles. Which of the following is true about the three triangles?

ANSWERS AND EXPLANATIONS Explanation for Correct Answer D :  Choice (D) is correct. A regular pentagon has five sides of equal length, so

and are each isosceles, and the two sides of equal length in have

the same length as the two sides of equal length in The five angles of a

regular pentagon are of equal measure, so has the same measure as

Thus, and are congruent. Therefore, and

is also isosceles.

Explanation for Incorrect Answer A :  Choice (A) is not correct. and have the same area, but the area of

is greater. is the largest angle of so is the longest

side of Let on be the point such that has the same length as

a side of the pentagon. It can be shown that is congruent to Therefore,

Explanation for Incorrect Answer B :  Choice (B) is not correct. and are of equal perimeter, but the

perimeter of is greater. is the largest angle of so is

the longest side of The perimeter of is

(A) They have equal areas.

(B) They have equal perimeters.

(C) They are similar.

(D) They are isosceles.

(E) They each have at least one angle of measure

页码,9/17The Official SAT Online Course

2006-11-12file://E:\新建文件夹\e4.htm

UnRe

gistered

Page 34: SAT practice test 5.pdf

严禁用于商业用途!

whereas the perimeter of is

Explanation for Incorrect Answer C :  Choice (C) is not correct. and are similar; in fact, they are

congruent. But is not similar to the other two triangles. The angles of

are of measure and whereas the other two triangles have

angles of measure and

Explanation for Incorrect Answer E :  Choice (E) is not correct. None of the three triangles has any angles of measure

13

If or which of the following must be true?

I.

II.

III.

ANSWERS AND EXPLANATIONS Explanation for Correct Answer B : 

Choice (B) is correct. If or then In addition, by the

definition of absolute value, it is also true that Therefore, statements I

and II must both be true.

However, statement III could be false. The value of could be greater than

but it could be less. For example, if then but

Explanation for Incorrect Answer A : 

Choice (A) is not correct. It is not true that must be greater than The value

of could be greater than but it could be less. For example, if then

but

Explanation for Incorrect Answer C :  Choice (C) is not correct. Statement I must be true, but statement II must also be

true, and statement III could be false. The value of could be greater than

but it could be less. For example, if then but

Explanation for Incorrect Answer D :  Choice (D) is not correct. Statement II must be true, but statement I must also be

true, and statement III could be false. The value of could be greater than

but it could be less. For example, if then but

(A) III only

(B) I and II only

(C) I and III only

(D) II and III only

(E) I, II, and III

页码,10/17The Official SAT Online Course

2006-11-12file://E:\新建文件夹\e4.htm

UnRe

gistered

Page 35: SAT practice test 5.pdf

严禁用于商业用途!

Explanation for Incorrect Answer E :  Choice (E) is not correct. Statements I and II must be true, but statement III could

be false. The value of could be greater than but it could be less. For

example, if then but

14

In the figure above, and If is an integer, what is the least

possible value of

ANSWERS AND EXPLANATIONS Explanation for Correct Answer E :  Choice (E) is correct. The sum of the measures of the three angles of a triangle is

so Since it follows that Substituting

for gives this equation is

equivalent to Since it follows that

which implies that Since is an

integer and it must be true that is also an integer. Therefore, the

least possible value of is the least integer greater than which is

Explanation for Incorrect Answer A :  Choice (A) is not correct. If were equal to then would equal

The value of would then be However,

so cannot be Therefore, cannot equal

Explanation for Incorrect Answer B :  Choice (B) is not correct. If were equal to then would equal

The value of would then be However,

so cannot be Therefore, cannot equal

Explanation for Incorrect Answer C :  Choice (C) is not correct. If were equal to then would equal

The value of would then be However,

so cannot be Therefore, cannot equal

Explanation for Incorrect Answer D :  Choice (D) is not correct. If were equal to then would equal

The value of would then be However,

so cannot be Therefore, cannot equal

(A)

(B)

(C)

(D)

(E)

15

For all positive integers and let be defined by If and

are positive integers, which of the following CANNOT be the value of

页码,11/17The Official SAT Online Course

2006-11-12file://E:\新建文件夹\e4.htm

UnRe

gistered

Page 36: SAT practice test 5.pdf

严禁用于商业用途!

ANSWERS AND EXPLANATIONS Explanation for Correct Answer B : 

Choice (B) is correct. The value of cannot be equal to If

then There are no positive integers

and such that There are nonnegative integers that work (for

example, and ), but the question states that and are positive integers.

Explanation for Incorrect Answer A : 

Choice (A) is not correct. If then The question

asks which number CANNOT be the value of

Explanation for Incorrect Answer C : 

Choice (C) is not correct. If and then The

question asks which number CANNOT be the value of

Explanation for Incorrect Answer D : 

Choice (D) is not correct. If then The

question asks which number CANNOT be the value of

Explanation for Incorrect Answer E : 

Choice (E) is not correct. If and then The

question asks which number CANNOT be the value of

(A)

(B)

(C)

(D)

(E)

16

The functions and are defined above. For how many values of is it true that

ANSWERS AND EXPLANATIONS Explanation for Correct Answer A : 

Choice (A) is correct. If then This implies

that so that There are no real numbers that satisfy

this equation.

(A) None

(B) One

(C) Two

(D) Three

(E) More than three

页码,12/17The Official SAT Online Course

2006-11-12file://E:\新建文件夹\e4.htm

UnRe

gistered

Page 37: SAT practice test 5.pdf

严禁用于商业用途!

Another way to see that has no solutions is to graph and

in the -plane and observe that these two graphs do not intersect.

Therefore, there are no values for which

Explanation for Incorrect Answer B : 

Choice (B) is not correct. If there were one value of for which were

true, then there would be a real solution of the equation However, this

equation has no real solutions.

Explanation for Incorrect Answer C : 

Choice (C) is not correct. If there were two values of for which were

true, then there would be two real solutions of the equation However,

this equation has no real solutions.

Explanation for Incorrect Answer D : 

Choice (D) is not correct. If there were three values of for which

were true, then there would be at least one real solution of the equation

However, this equation has no real solutions. (In any case, the number

of solutions of a quadratic equation cannot be greater than two.)

Explanation for Incorrect Answer E :  Choice (E) is not correct. If there were more than three values of for which

were true, then there would be at least one real solution of the

equation However, this equation has no real solutions. (In any case,

the number of solutions of a quadratic equation cannot be greater than two.)

17

The function defined for is graphed above. Which of the

following gives all values of for which is positive?

ANSWERS AND EXPLANATIONS Explanation for Correct Answer D : 

Choice (D) is correct. The values of for which is positive are exactly those

(A)

(B)

(C)

(D) and

(E) and

页码,13/17The Official SAT Online Course

2006-11-12file://E:\新建文件夹\e4.htm

UnRe

gistered

Page 38: SAT practice test 5.pdf

严禁用于商业用途!

values on the -axis for which the graph of lies above the -axis. The graph

of lies above the -axis for and Therefore,

and are all the values of for which is positive.

Explanation for Incorrect Answer A : 

Choice (A) is not correct. For example, is positive for which is not in

the interval (Also, is not positive for many values of in the

interval )

Explanation for Incorrect Answer B : 

Choice (B) is not correct. It is true that is positive for all values of such

that but is also positive for all values of such that

Explanation for Incorrect Answer C :  Choice (C) is not correct. For example, is in the interval yet

is not positive. (Also, is positive for all values of in the interval

)

Explanation for Incorrect Answer E : 

Choice (E) is not correct. is negative exactly for those values of in the

intervals and However, the question asks for all the values

of for which is positive.

18

In the -plane, an equation of line is If line is the reflection of

line in the -axis, what is an equation of line

ANSWERS AND EXPLANATIONS Explanation for Correct Answer A : 

Choice (A) is correct. If is a point on line then it is true that

When line is reflected in the -axis, the point is reflected

to the point which is on line Since the coordinates of

point satisfy the relationship Therefore, an

equation of line is

Lines and are graphed in the figure below.

(A)

(B)

(C)

(D)

(E)

页码,14/17The Official SAT Online Course

2006-11-12file://E:\新建文件夹\e4.htm

UnRe

gistered

Page 39: SAT practice test 5.pdf

严禁用于商业用途!

Explanation for Incorrect Answer B :  Choice (B) is not correct. An equation of line is found by substituting for in the equation This incorrect answer may be the result of mistakenly

multiplying the entire right-hand side of by

Explanation for Incorrect Answer C :  Choice (C) is not correct. An equation of line is found by substituting for in the equation This incorrect answer may be the result of mistakenly

substituting for instead of for in the equation

Explanation for Incorrect Answer D : 

Choice (D) is not correct. is an equation of a line perpendicular to line

not of the line that is the reflection of line in the -axis.

Explanation for Incorrect Answer E : 

Choice (E) is not correct. is an equation of a line perpendicular to line

not of the line that is the reflection of line in the -axis.

19

For which of the following values for will the system of equations above have no solution?

ANSWERS AND EXPLANATIONS Explanation for Correct Answer A :  Choice (A) is correct. If then the system of equations becomes

This system has no solution: if then must

be equal to not

Explanation for Incorrect Answer B :  Choice (B) is not correct. If then the system of equations becomes

This system has the solution and

Explanation for Incorrect Answer C :  Choice (C) is not correct. If then the system of equations becomes

(A)

(B)

(C)

(D)

(E)

页码,15/17The Official SAT Online Course

2006-11-12file://E:\新建文件夹\e4.htm

UnRe

gistered

Page 40: SAT practice test 5.pdf

严禁用于商业用途!

This system has the solution and

Explanation for Incorrect Answer D :  Choice (D) is not correct. If then the system of equations becomes

This system has the solution and

Explanation for Incorrect Answer E :  Choice (E) is not correct. If then the system of equations becomes

This system has the solution and

20

The table above shows the number of items customers purchased from a hardware store over a -hour period. Which of the following can be determined from the information in the table?

I. The average (arithmetic mean) number of items purchased per customer

II. The median number of items purchased per customer

III. The mode of the number of items purchased per customer

ANSWERS AND EXPLANATIONS Explanation for Correct Answer D :  Choice (D) is correct. To determine the average number of items purchased per customer, it is necessary to know the total number of items purchased. This total can then be divided by to find the average. However, for of the

(A) None

(B) I and II only

(C) I and III only

(D) II and III only

(E) I, II, and III

页码,16/17The Official SAT Online Course

2006-11-12file://E:\新建文件夹\e4.htm

UnRe

gistered

Page 41: SAT practice test 5.pdf

严禁用于商业用途!

customers, the exact number of items purchased is not given in the table. “Fewer than ” could mean these customers bought as many as or as few as items. Therefore, I cannot be determined.

The median number of items purchased per customer is the average of the number of items purchased by the and customers when all customers are rank

ordered by the number of items purchased. The and customers in this rank ordering each purchased items, so the median is Therefore, II can be determined.

The mode is the number of items purchased by the greatest number of customers. The mode is because customers purchased items each, and no larger

group purchased a number of items different from Therefore, III can be determined.

Summarizing, only II and III can be determined from the information in the table. Therefore, choice (D) is the correct answer.

Explanation for Incorrect Answer A :  Choice (A) is not correct. The median number of items purchased per customer and the mode of the number of items purchased per customer can be determined from the information in the table.

Explanation for Incorrect Answer B :  Choice (B) is not correct. The median can be determined from the information in the table, but the mean cannot.

Explanation for Incorrect Answer C :  Choice (C) is not correct. The mode can be determined from the information in the table, but the mean cannot.

Explanation for Incorrect Answer E :  Choice (E) is not correct. The median and the mode can be determined from the information in the table, but the mean cannot.

    

Back to Score Report  

Copyright © 2006 The College Board. All rights reserved. Privacy Policy Terms of Use Contact Us

 

页码,17/17The Official SAT Online Course

2006-11-12file://E:\新建文件夹\e4.htm

UnRe

gistered

Page 42: SAT practice test 5.pdf

严禁用于商业用途!

Help | Profile | My Organizer | My Bookmarks | Logout

Answers and Explanations

Test Sections

Section 1

Section 2

Section 3

Section 4

Section 5

Section 6

Section 8

Section 9

Section 10

Back to Score Report  

View Answers and Explanations     Online - Practice Test #5

1 The border between the two properties, never ------- by legal means, had long been the subject of ------- between the antagonistic neighbors.

ANSWERS AND EXPLANATIONS Explanation for Correct Answer E :  Choice (E) is correct. “To establish” means to agree to institute something permanently. “Disputation” is the act of arguing. If one were to insert these terms into the text, the sentence would read “The border between the two properties, never established by legal means, had long been the subject of disputation between the antagonistic neighbors.” It is logical to suggest that antagonistic neighbors, or neighbors who express hostility and opposition, might argue over a border that had never been "established."

Explanation for Incorrect Answer A :  Choice (A) is incorrect. “To determine” means to settle or decide authoritatively. “Concord” is a state of agreement. If one were to insert these terms into the text, the sentence would read “The border between the two properties, never determined by legal means, had long been the subject of concord between the antagonistic neighbors.” Antagonistic neighbors, or neighbors who express hostility and opposition, would probably not have a state of agreement concerning a border that had not been "determined." On the contrary, an undetermined border would most likely cause arguments between antagonistic neighbors.

Explanation for Incorrect Answer B :  Choice (B) is incorrect. “To undermine” means to weaken or subvert. “Hostility” is an attitude of ill will and opposition. If one were to insert these terms into the text, the sentence would read “The border between the two properties, never undermined by legal means, had long been the subject of hostility between the antagonistic neighbors.” Although antagonistic neighbors might be hostile to one another, it is somewhat illogical to say that the property border had never been “undermined,” or weakened, by legal means.

Explanation for Incorrect Answer C :  Choice (C) is incorrect. “To verify” means to establish formally. “Consonance” is a state of harmony and agreement. If one were to insert these terms into the text, the sentence would read “The border between the two properties, never verified by legal means, had long been the subject of consonance between the antagonistic neighbors.” Antagonistic neighbors, or neighbors who express hostility and opposition, would most likely not have “consonance,” or a state of harmony, concerning a border that had never been formally established. On the contrary, a border that has not been “verified,” or established, would probably cause arguments between antagonistic neighbors.

Explanation for Incorrect Answer D :  Choice (D) is incorrect. “To quantify” means to measure or determine in terms of amount or magnitude. “Distraction” is something that draws attention away from something else. If one were to insert these terms into the text, the sentence would read “The border between the two properties, never quantified by legal means, had long been the subject of distraction between the antagonistic neighbors.”

(A) determined . . concord

(B) undermined . . hostility

(C) verified . . consonance

(D) quantified . . diversion

(E) established . . disputation

页码,1/24The Official SAT Online Course

2006-11-12file://E:\新建文件夹\e5.htm

UnRe

gistered

Page 43: SAT practice test 5.pdf

严禁用于商业用途!

Although a property border might be a “distraction” for antagonistic neighbors, it is somewhat illogical to suggest that the border could be “quantified,” or determined in terms of amount or magnitude.

2 Even though Charlie was in apparently good health, the doctor prescribed for him some ------- medication due to his familial history of high blood pressure.

ANSWERS AND EXPLANATIONS Explanation for Correct Answer E :  Choice (E) is correct. “Preventative” means preventing something. If one were to insert this term into the text, the sentence would read “Even though Charlie was in apparently good health, the doctor prescribed for him some preventative medication due to his familial history of high blood pressure.” The phrase “Even though” sets up a contrast between Charlie’s apparent good health and his family’s history of high blood pressure. The missing term should describe a medication that the doctor would prescribe for Charlie even in the absence of symptoms. It would make sense for the doctor to prescribe “preventative” medication to help Charlie avoid developing the high blood pressure that runs in his family.

Explanation for Incorrect Answer A :  Choice (A) is incorrect. “Presumptive” means based on an assumption. If one were to insert this term into the text, the sentence would read “Even though Charlie was in apparently good health, the doctor prescribed for him some presumptive medication due to his familial history of high blood pressure.” While the doctor might have presumed that Charlie should be taking medication due to his family’s history of high blood pressure, it does not make sense to say that the medication itself was “presumptive,” or based on an assumption that the doctor made about Charlie’s health.

Explanation for Incorrect Answer B :  Choice (B) is incorrect. “Predictive” means making a prediction. If one were to insert this term into the text, the sentence would read “Even though Charlie was in apparently good health, the doctor prescribed for him some predictive medication due to his familial history of high blood pressure.” It does not make sense to describe medication as “predictive.” Medication cannot predict a patient’s health.

Explanation for Incorrect Answer C :  Choice (C) is incorrect. “Preliminary” means coming before something else. If one were to insert this term into the text, the sentence would read “Even though Charlie was in apparently good health, the doctor prescribed for him some preliminary medication due to his familial history of high blood pressure.” The phrase “Even though” sets up a contrast between Charlie’s apparent good health and his family’s history of high blood pressure. The missing term should describe a medication that the doctor would prescribe for Charlie even in the absence of symptoms. For medication to be “preliminary,” or coming before something else, the doctor would have to know that Charlie would later need other medications. Charlie may or may not develop high blood pressure, so the medication would not necessarily be preliminary.

Explanation for Incorrect Answer D :  Choice (D) is incorrect. “Premeditated” means considered beforehand. If one were to insert this term into the text, the sentence would read “Even though Charlie was in apparently good health, the doctor prescribed for him some premeditated medication due to his familial history of high blood pressure.” While the doctor’s decision to prescribe medication for Charlie may have been “premeditated,” or considered beforehand, it does not make sense to say that the medication itself was premeditated.

(A) presumptive

(B) predictive

(C) preliminary

(D) premeditated

(E) preventative

页码,2/24The Official SAT Online Course

2006-11-12file://E:\新建文件夹\e5.htm

UnRe

gistered

Page 44: SAT practice test 5.pdf

严禁用于商业用途!

3 Though earlier anatomists had touched on the idea, Paul Broca was the first to --------- fully the modern notion that specific behaviors are controlled by particular areas of the human brain.

ANSWERS AND EXPLANATIONS Explanation for Correct Answer A :  Choice (A) is correct. “To articulate” means to express a concept. If one were to insert this term into the text, the sentence would read “Though earlier anatomists had touched on the idea, Paul Broca was the first to articulate fully the modern notion that specific behaviors are controlled by particular areas of the human brain.” While earlier anatomists had touched on the idea that certain behaviors are controlled by particular areas of the brain, Paul Broca was the first to fully “articulate,” or express, the concept.

Explanation for Incorrect Answer B :  Choice (B) is incorrect. “To derogate” means to belittle or to cause to seem inferior. If one were to insert this term into the text, the sentence would read “Though earlier anatomists had touched on the idea, Paul Broca was the first to derogate fully the modern notion that specific behaviors are controlled by particular areas of the human brain.” Nothing in the sentence suggests that either the earlier anatomists or Paul Broca in any way derogated, or belittled, the idea that certain behaviors are controlled by particular areas of the brain.

Explanation for Incorrect Answer C :  Choice (C) is incorrect. “To represent” means to serve as a sign or symbol of something. If one were to insert this term into the text, the sentence would read “Though earlier anatomists had touched on the idea, Paul Broca was the first to represent fully the modern notion that specific behaviors are controlled by particular areas of the human brain.” The fact that earlier anatomists touched on the notion that certain behaviors are controlled by particular areas of the brain is not logically connected to the idea that Paul Broca represented, or was a symbol of, that notion.

Explanation for Incorrect Answer D :  Choice (D) is incorrect. “To refute” means to prove wrong. If one were to insert this term into the text, the sentence would read “Though earlier anatomists had touched on the idea, Paul Broca was the first to refute fully the modern notion that specific behaviors are controlled by particular areas of the human brain.” Nothing in the sentence suggests that earlier anatomists had partially “refuted,” or proved wrong, the idea that certain behaviors are controlled by particular areas of the brain, so it would not make sense to say that Paul Broca fully refuted the idea.

Explanation for Incorrect Answer E :  Choice (E) is incorrect. “To iterate” means to say or do something repeatedly. If one were to insert this term into the text, the sentence would read “Though earlier anatomists had touched on the idea, Paul Broca was the first to iterate fully the modern notion that specific behaviors are controlled by particular areas of the human brain.” Although Paul Broca could have been the first to fully state the idea that specific areas of the human brain control specific behaviors, nothing in the sentence suggests that he stated this idea repeatedly.

(A) articulate

(B) derogate

(C) represent

(D) refute

(E) iterate

4 Designed as a gathering place, the new student lounge was appropriately ------- with tables, chairs, and even sofas where groups could assemble comfortably.

(A) indicated

(B) appointed

(C) denuded

页码,3/24The Official SAT Online Course

2006-11-12file://E:\新建文件夹\e5.htm

UnRe

gistered

Page 45: SAT practice test 5.pdf

严禁用于商业用途!

ANSWERS AND EXPLANATIONS Explanation for Correct Answer B :  Choice (B) is correct. “To appoint” means to provide with furnishings or equipment. If one were to insert this term into the text, the sentence would read “Designed as a gathering place, the new student lounge was appropriately appointed with tables, chairs, and even sofas where groups could assemble comfortably.” A student lounge that is intended to be a gathering place would most likely be “appointed,” or furnished, with tables, chairs, and sofas that would allow groups to gather together in comfortable surroundings.

Explanation for Incorrect Answer A :  Choice (A) is incorrect. “To indicate” means to show or make known. If one were to insert this term into the text, the sentence would read “Designed as a gathering place, the new student lounge was appropriately indicated with tables, chairs, and even sofas where groups could assemble comfortably.” It does not make sense to say that the lounge was “indicated,” or shown, by its tables, chairs, and sofas.

Explanation for Incorrect Answer C :  Choice (C) is incorrect. “To denude” means to make bare. If one were to insert this term into the text, the sentence would read “Designed as a gathering place, the new student lounge was appropriately denuded with tables, chairs, and even sofas where groups could assemble comfortably.” The term “denuded” does not make sense in this context. If the student lounge was “denuded,” or made bare, it would not have any furniture.

Explanation for Incorrect Answer D :  Choice (D) is incorrect. “To conflate” means to join or combine. If one were to insert this term into the text, the sentence would read “Designed as a gathering place, the new student lounge was appropriately conflated with tables, chairs, and even sofas where groups could assemble comfortably.” Although a gathering place might have tables, chairs, and sofas, it does not make sense to say that the student lounge was “conflated,” or joined, with these pieces of furniture.

Explanation for Incorrect Answer E :  Choice (E) is incorrect. “To venerate” means to honor or regard something with respect. If one were to insert this term into the text, the sentence would read “Designed as a gathering place, the new student lounge was appropriately venerated with tables, chairs, and even sofas where groups could assemble comfortably.” Although a gathering place might have tables, chairs, and sofas, it is not clear how these pieces of furniture would cause the student lounge to be “venerated,” or regarded with respect.

(D) conflated

(E) venerated

5

Named in honor of the school’s founder, the Richard Brownstone Community Service Fellowship is one of the highest awards ------- Brownstone School graduates.

ANSWERS AND EXPLANATIONS Explanation for Correct Answer C :  Choice (C) is correct. “To confer” means to give to or to bestow upon. If one were to insert this term into the text, the sentence would read “Named in honor of the school’s founder, the Richard Brownstone Community Service Fellowship is one of the highest awards conferred on Brownstone School graduates.” It makes sense to say that an award would be “conferred on,” or given to, graduates of Brownstone School.

(A) conscripted to

(B) redeemed for

(C) conferred on

(D) relegated to

(E) deprived of

页码,4/24The Official SAT Online Course

2006-11-12file://E:\新建文件夹\e5.htm

UnRe

gistered

Page 46: SAT practice test 5.pdf

严禁用于商业用途!

Explanation for Incorrect Answer A :  Choice (A) is incorrect. “To conscript” means to enroll into service. If one were to insert this term into the text, the sentence would read “Named in honor of the school’s founder, the Richard Brownstone Community Service Fellowship is one of the highest awards conscripted to Brownstone School graduates.” The term “conscripted” does not make sense in this context. It is illogical to suggest that an award could be enrolled into service.

Explanation for Incorrect Answer B :  Choice (B) is incorrect. “To redeem” means to exchange for something of value. If one were to insert this term into the text, the sentence would read “Named in honor of the school’s founder, the Richard Brownstone Community Service Fellowship is one of the highest awards redeemed for Brownstone School graduates.” The term “redeemed” does not make sense in this context. It does not make sense to say that an award can be “redeemed,” or exchanged, for graduates of the Brownstone School.

Explanation for Incorrect Answer D :  Choice (D) is incorrect. “To relegate” means to assign to a place of insignificance. If one were to insert this term into the text, the sentence would read “Named in honor of the school’s founder, the Richard Brownstone Community Service Fellowship is one of the highest awards relegated to Brownstone School graduates.” It does not make sense to suggest that presenting graduates of Brownstone School with an award is an action of assigning something to a place of insignificance. If some of the graduates are presented with “one of the highest awards,” it is illogical to describe either the award or the recipients as insignificant.

Explanation for Incorrect Answer E :  Choice (E) is incorrect. “To deprive” means to take away or withhold. If one were to insert this term into the text, the sentence would read “Named in honor of the school’s founder, the Richard Brownstone Community Service Fellowship is one of the highest awards deprived of Brownstone School graduates.” It does not make sense to suggest that the award is “deprived of,” or withheld from, graduates of Brownstone School.

The following passage on monarch butterflies was written in 2000 by a naturalist and insect specialist.

       The migration and the winter  gathering of monarch butterflies are  among the most spectacular of all naturalLine phenomena, unique in the insect world.5 Lincoln Brower wrote of his feeling on a  warm March morning as he watched tens  of thousands of these butterflies explode  from their resting places on the trees at  an overwintering site in Mexico: “Flying10 against the azure sky and past the green  boughs of the oyamels, this myriad of  dancing embers reinforced my earlier  conclusion that this spectacle is a treasure  comparable to the finest works of art that15 our world culture has produced over the  past 4000 years.” But even as I write this  paragraph, the winter gathering places of  the monarch are being destroyed by  illegal logging—indeed, all of the oyamel20 forests in Mexico are threatened by legal  and illegal logging. If the logging 

页码,5/24The Official SAT Online Course

2006-11-12file://E:\新建文件夹\e5.htm

UnRe

gistered

Page 47: SAT practice test 5.pdf

严禁用于商业用途!

continues at its present rate, all of the  overwintering sites in Mexico will be gone  by the first decades of the twenty-first25 century. So desperate is the situation  that the Union for the Conservation of  Nature and Natural Resources has  recognized the monarch migration as an  endangered biological phenomenon and30 has designated it the first priority in their  effort to conserve the butterflies of the  world.       All efforts to preserve the  overwintering sites in Mexico have failed.35 In August of 1986, the Mexican  government issued a proclamation  designating these sites as ecological  preserves. Five of the 12 known sites  were to receive complete protection.40 Logging and agricultural development  were to be prohibited in their core areas,  a total area of only 17 square miles, and  only limited logging was to be permitted in  buffer zones surrounding the cores, a total45 of another 43 miles. The proclamation  was largely ignored. One of the 5  “protected” sites has been clear-cut, some  buffer zones have been more or less  completely destroyed, and trees are being50 cut in all of the core areas. As Brower  told me, guards that were appointed to  protect the monarch colonies have not  prevented illegal logging but have barred  tourists, film crews, and scientists from55 witnessing logging activities. It is  incomprehensible to me that a way cannot  be found to protect a mere 60 square  miles of land that are home to one of the  world’s most spectacular biological60 phenomena.       If the monarchs are to survive, the  oyamel forests in which they spend the  winter must remain intact. Even minor  thinning of the core areas causes high65 mortality among the butterflies, because  the canopy of the intact forest serves as a  protective blanket and umbrella for them.  Within a dense stand of trees, the  temperature does not drop as low as it70 does elsewhere, enabling the monarchs to  survive freezing weather under the  blanket of trees. Thinning the trees puts  holes in the “umbrella” that protects the

页码,6/24The Official SAT Online Course

2006-11-12file://E:\新建文件夹\e5.htm

UnRe

gistered

Page 48: SAT practice test 5.pdf

严禁用于商业用途!

  monarchs, letting them get wet during75 winter storms. A wet butterfly loses its  resistance to freezing and dies. Even a  dry butterfly loses precious calories as its  body heat radiates out to the cold night  sky through holes in the canopy.

6 The quotation in lines 9-16 serves primarily as a

ANSWERS AND EXPLANATIONS Explanation for Correct Answer C :  Choice (C) is correct. The author begins the passage by stating that “the migrations and winter gathering of monarch butterflies are among the most spectacular of all natural phenomena.” To support this statement powerfully, the author uses Lincoln Brower’s vivid and dramatic description of his personal viewing of the monarchs. Brower’s portrayal of the monarchs as “this myriad of dancing embers” “flying against the azure sky” gives the reader the sense that the winter gathering of monarchs is an impressive event.

Explanation for Incorrect Answer A :  Choice (A) is incorrect. The quotation vividly describes the winter gathering of monarchs in Mexico. Although Lincoln Brower does claim that the event is “a treasure comparable to the finest works of art that our world culture has produced,” he does not claim that the winter gathering place itself has any cultural significance.

Explanation for Incorrect Answer B :  Choice (B) is incorrect. The quotation does recount a personal observation of Lincoln Brower’s, but the observation is about how spectacular the winter gathering of monarchs is, not about artistic awareness in general.

Explanation for Incorrect Answer D :  Choice (D) is incorrect. The author of the passage does claim that the winter gathering of monarchs is a rare phenomenon, “unique in the insect world.” But the quotation is a dramatic portrayal using vivid imagery, not a scientific account of the event.

Explanation for Incorrect Answer E :  Choice (E) is incorrect. Although the quotation by Lincoln Brower does state that the winter gathering of monarchs is a spectacle “comparable to the finest works of art our world culture has produced,” Brower does not develop a conclusive argument for its artistic importance. He simply thinks it is an extremely beautiful natural event.

(A) detailed explanation of the cultural significance of a place

(B) personal observation about artistic awareness

(C) dramatic portrayal of an impressive event

(D) scientific account of a rare phenomenon

(E) conclusive argument for the artistic importance of spectacle

7 The author views the “efforts” cited in line 33 as

ANSWERS AND EXPLANATIONS

(A) understandably futile

(B) necessarily limited

(C) scientifically misguided

(D) largely undesirable

(E) unjustifiably ineffective

页码,7/24The Official SAT Online Course

2006-11-12file://E:\新建文件夹\e5.htm

UnRe

gistered

Page 49: SAT practice test 5.pdf

严禁用于商业用途!

Explanation for Correct Answer E :  Choice (E) is correct. The “efforts” in line 33 refer to failed attempts “to preserve the overwintering sites [of monarchs] in Mexico” from the destruction brought on by unchecked “legal and illegal logging.” The author clearly believes that these efforts have been ineffective, reporting that “one of the five ‘protected’ sites has been clear-cut, some buffer zones have been more or less completely destroyed, and trees are being cut down in all of the core areas.” The author openly states that such ineffectiveness is unjustifiable: “It is incomprehensible to me that a way cannot be found to protect a mere 60 square miles of land that are home to one of the world’s most spectacular biological phenomena.”

Explanation for Incorrect Answer A :  Choice (A) is incorrect. The “efforts” in line 33 refer to failed attempts “to preserve the overwintering sites [of monarchs] in Mexico” from the destruction brought on by unchecked “legal and illegal logging.” While the author clearly believes that these efforts have been futile, he or she would not agree that this futility is understandable. The author cannot understand this futility: “It is incomprehensible to me that a way cannot be found to protect a mere 60 square miles of land that are home to one of the world’s most spectacular biological phenomena.”

Explanation for Incorrect Answer B :  Choice (B) is incorrect. The “efforts” in line 33 refer to failed attempts “to preserve the overwintering sites [of monarchs] in Mexico” from the destruction brought on by unchecked “legal and illegal logging.” The author might agree that the efforts have been limited, but not that this limitation is necessary. The author actually believes that it should be rather easy to protect the land: “It is incomprehensible to me that a way cannot be found to protect a mere 60 square miles of land that are home to one of the world’s most spectacular biological phenomena.”

Explanation for Incorrect Answer C :  Choice (C) is incorrect. The “efforts” in line 33 refer to failed attempts “to preserve the overwintering sites [of monarchs] in Mexico” from the destruction brought on by unchecked “legal and illegal logging.” The author does not comment on the scientific foundation underlying the efforts to protect the sites.

Explanation for Incorrect Answer D :  Choice (D) is incorrect. The “efforts” in line 33 refer to failed attempts “to preserve the overwintering sites [of monarchs] in Mexico” from the destruction brought on by unchecked “legal and illegal logging.” The author argues that the efforts have been ineffective, not undesirable. On the contrary, the author clearly desires stronger efforts to protect the sites: “It is incomprehensible to me that a way cannot be found to protect a mere 60 square miles of land that are home to one of the world’s most spectacular biological phenomena.”

8 The third paragraph is best described as

ANSWERS AND EXPLANATIONS Explanation for Correct Answer C :  Choice (C) is correct. The third paragraph describes the effects that destroying trees in the oyamel forest has on monarch butterflies. When trees are cut down, holes are created in the protective canopy that usually keeps the monarchs warm and dry. When the monarchs become cold and wet they often freeze to death. Such events are a disruption in the natural oyamel forest ecological system caused by the unchecked logging.

(A) an account of a natural struggle for survival

(B) a comparison between two types of environments

(C) a description of a disruption in an ecological system

(D) a demonstration of successful efforts to preserve an environment

(E) a guideline for opposing the destruction of a crucial habitat

页码,8/24The Official SAT Online Course

2006-11-12file://E:\新建文件夹\e5.htm

UnRe

gistered

Page 50: SAT practice test 5.pdf

严禁用于商业用途!

Explanation for Incorrect Answer A :  Choice (A) is incorrect. The description of the monarchs dying in the cold, wet night is certainly a struggle for survival. The monarch’s struggle, however, is not natural; it is caused by unchecked human loggers.

Explanation for Incorrect Answer B :  Choice (B) is incorrect. The third paragraph describes only one environment: the oyamel forest that is being destroyed by loggers.

Explanation for Incorrect Answer D :  Choice (D) is correct. The third paragraph does not describe successful efforts to preserve an environment. It describes the effects that the destruction of an environment—the oyamel forest—has on the wintering monarchs.

Explanation for Incorrect Answer E :  Choice (E) is incorrect. The paragraph offers no guideline for opposing the destruction of the oyamel forest. It simply describes the effects that the destruction has on the wintering monarchs.

9 The tone of the passage could best be characterized as

ANSWERS AND EXPLANATIONS Explanation for Correct Answer C :  Choice (C) is correct. The passage exhibits a concerned tone. The author begins by describing the spectacular beauty of wintering monarchs. Then the author states that “even as I write this paragraph, the winter gathering places of the monarch are being destroyed by illegal logging.” The passage goes on to describe the failing attempts being made to preserve the overwintering sites and the dying monarchs’ struggle to survive in the damaged forests. Overall, the author demonstrates a deep concern for the survival of the monarchs.

Explanation for Incorrect Answer A :  Choice (A) is incorrect. The author is not at all indifferent to the plight of the monarch butterfly. The author is very concerned with preserving the wintering sites of the monarch, explicitly stating: “It is incomprehensible to me that a way cannot be found to protect a mere 60 square miles of land that are home to one of the world’s most spectacular biological phenomena.”

Explanation for Incorrect Answer B :  Choice (B) is incorrect. The passage does not exhibit a hostile tone. Mainly the author is concerned about the plight of the monarch butterfly. The author is critical of the failed efforts to preserve the overwintering sites, but the tone is troubled, not hostile.

Explanation for Incorrect Answer D :  Choice (D) is incorrect. The tone of passage is concerned, not bewildered, or confused. The author fully understands that the monarchs are in danger of losing their overwintering habitat to logging.

Explanation for Incorrect Answer E :  Choice (E) is incorrect. The tone of the passage is concerned, not complimentary. The author does not compliment the “efforts” to preserve the monarchs’ overwintering sites, but rather criticizes them for their failure.

(A) indifferent

(B) hostile

(C) concerned

(D) bewildered

(E) complimentary

In this excerpt from a British novel published in 1938, a woman describes staying with her employer at a fashionable hotel in the resort city of Monte Carlo.

页码,9/24The Official SAT Online Course

2006-11-12file://E:\新建文件夹\e5.htm

UnRe

gistered

Page 51: SAT practice test 5.pdf

严禁用于商业用途!

       I wonder what my life would be to-  day, if Mrs. Van Hopper had not been a  snob.Line      Funny to think that the course of5 my existence hung like a thread upon that  quality of hers. Her curiosity was a  disease, almost a mania. At first I had  been shocked, wretchedly embarrassed  when I watched people laugh behind her10 back, leave a room hurriedly upon her  entrance, or even vanish behind a Service  door on the corridor upstairs. For many  years now she had come to the hotel Cote  d’Azur, and, apart from bridge, her one15 pastime, which was notorious by now in  Monte Carlo, was to claim visitors of  distinction as her friends had she but seen  them once at the other end of the post-  office. Somehow she would manage to20 introduce herself, and before her victim  had scented danger she had proffered an  invitation to her suite. Her method of  attack was so downright and sudden that  there was seldom opportunity to escape.25 At the Cote d’Azur she staked a claim  upon a certain sofa in the lounge, midway  between the reception hall and the  passage to the restaurant, and she would  have her coffee there after luncheon and30 dinner, and all who came and went must  pass her by. Sometimes she would  employ me as a bait to draw her prey,  and, hating my errand, I would be sent  across the lounge with a verbal message,35 the loan of a book or paper, the address  of some shop or other, the sudden  discovery of a mutual friend. It seemed  as though notables must be fed to her,  and though titles1 were preferred by her,40 any face once seen in a social paper  served as well. Names scattered in a  gossip column, authors, artists, actors and  their kind, even the mediocre ones, as  long as she had learnt of them in print.45      I can see her as though it were but  yesterday, on that unforgettable  afternoon—never mind how many years  ago—when she sat on her favourite sofa in  the lounge, debating her method of50 attack. I could tell by her abrupt manner,  and the way she tapped her lorgnette2

页码,10/24The Official SAT Online Course

2006-11-12file://E:\新建文件夹\e5.htm

UnRe

gistered

Page 52: SAT practice test 5.pdf

严禁用于商业用途!

  against her teeth, that she was questing  possibilities. I knew, too, when she had  missed the sweet and rushed through55 dessert, and she had wished to finish  luncheon before the new arrival and so  install herself where he must pass.  Suddenly she turned to me, her small  eyes alight.60      “Go upstairs quickly and find that  letter from my nephew. You remember,  the one written on his honeymoon, with  the snapshot. Bring it down right away.”       I saw then that her plans were65 formed, and the nephew was to be the  means of introduction. Not for the first  time I resented the part that I must play  in her schemes. Like a juggler’s assistant  I produced the props, then silent and70 attentive I waited on my cue. This new-  comer would not welcome intrusion, I felt  certain of that. In the little I had learnt of  him at luncheon, a smattering of hearsay  garnered by her ten months ago from the75 daily papers and stored in her memory for  future use, I could imagine, in spite of my  youth and inexperience of the world, that  he would resent this sudden bursting in  upon his solitude. Why he should have80 chosen to come to the Cote d’Azur at  Monte Carlo was not our concern, his  problems were his own, and anyone but  Mrs. Van Hopper would have understood.  Tact was a quality unknown to her,85 discretion too, and because gossip was  the breath of life to her this stranger must  be served for her dissection.

 

1 “Titles” here refers to members of the European nobility. 2 Eyeglasses on the end of a short handle.

10The passage is narrated from the point of view of

ANSWERS AND EXPLANATIONS

(A) an employee of the Cote d’Azur hotel

(B) an observer who is uninvolved in the action

(C) Mrs. Van Hopper

(D) a participant who is remembering the scene at a later time

(E) a tourist who has just met Mrs. Van Hopper

页码,11/24The Official SAT Online Course

2006-11-12file://E:\新建文件夹\e5.htm

UnRe

gistered

Page 53: SAT practice test 5.pdf

严禁用于商业用途!

Explanation for Correct Answer D :  Choice (D) is correct. We know that the narrator is a “participant” because she narrates the action in the first person—“Sometimes she [Mrs. Van Hopper] would employ me as a bait to draw her prey,” for example. Further, there are clear signs in the passage that the narrator is recalling the action from the distance of some years: “I can see her as though it were but yesterday”; “never mind how many years ago.” Together, these clues tell the reader that the person narrating is someone who stayed with Mrs. Van Hopper years before at the Cote d’Azur hotel and is remembering the action.

Explanation for Incorrect Answer A :  Choice (A) is incorrect. It is clear from context that the setting is the Cote d’Azur hotel, but it is also clear that the narrator works for Mrs. Van Hopper, not for the hotel: Mrs. Van Hopper might very well send a hotel employee on a personal errand as she does in paragraph 4, but no hotel employee could know as much about or spend as much time with Mrs. Van Hopper as does this narrator.

Explanation for Incorrect Answer B :  Choice (B) is incorrect. Certainly the narrator is an “observer” of Mrs. Van Hopper’s various strategies for “trapping” people, but we know at least as early as line 31 (“Sometimes she would employ me…”) that the narrator is also involved in the action.

Explanation for Incorrect Answer C :  Choice (C) is incorrect. It is unlikely—though not impossible—that Mrs. Van Hopper would be speaking of herself in the third person. Further, when Mrs. Van Hopper engages directly with the narrator in paragraph 4, we can be fairly certain that a separate participant is involved.

Explanation for Incorrect Answer E :  Choice (E) is incorrect. The last paragraph of the passage does focus on someone who, willingly or not, is about to meet Mrs. Van Hopper, but the passage clearly is not narrated from this person’s point of view.

11The “disease” mentioned in line 7 is best described as

ANSWERS AND EXPLANATIONS Explanation for Correct Answer E :  Choice (E) is correct. In the second paragraph of the passage, the narrator says that Mrs. Van Hopper’s “curiosity was a disease.” The narrator then sets about describing this “curiosity,” which turns out to be an abnormal interest in making the acquaintance of well-known people. The paragraph describes various of Mrs. Van Hopper’s techniques for meeting celebrities she does not know and claiming them as her “friends”—behavior that clearly embarrasses the narrator. The narrator says that this “disease,” or sickness, verges on mania, or an unreasonable enthusiasm—a “relentless drive,” in other words, to meet people who have been written about in gossip columns.

Explanation for Incorrect Answer A :  Choice (A) is incorrect. While the narrator acknowledges being “wretchedly embarrassed” at Mrs. Van Hopper’s importunate behavior with strangers, the “disease” referred to in line 7 is clearly attributed by the narrator to Mrs. Van Hopper, who shows no “embarrassment” at all in preying on celebrities.

Explanation for Incorrect Answer B :  Choice (B) is incorrect. In the first paragraph, the narrator describes Mrs. Van

(A) total embarrassment at another person’s behavior

(B) a refusal to speak to anyone who is not wealthy

(C) an intense need to avoid public notice

(D) a violent tendency to assault strangers

(E) a relentless drive to meet well-known people

页码,12/24The Official SAT Online Course

2006-11-12file://E:\新建文件夹\e5.htm

UnRe

gistered

Page 54: SAT practice test 5.pdf

严禁用于商业用途!

Hopper as a "snob," and “a refusal to speak to anyone who is not wealthy” could be considered a specific kind of snobbery. But it is clear from context that the “disease” is not Mrs. Van Hopper’s refusal to speak, but rather her insistence on speaking to the famous. While it can be inferred that most of those who frequent this expensive hotel are wealthy, it is celebrity rather than money that Mrs. Van Hopper is drawn to; this is her “disease.”

Explanation for Incorrect Answer C :  Choice (C) is incorrect. Far from “avoiding public notice,” Mrs. Van Hopper seats herself daily in an area of the hotel where she is almost certain to be noticed, much to the embarrassment of the narrator.

Explanation for Incorrect Answer D :  Choice (D) is incorrect. While Mrs. Van Hopper certainly does “assault” people in the figurative sense of approaching them without their permission, there is no indication that she is “violent” towards them, or even hostile. Rather, Mrs. Van Hopper’s technique would more likely be directed towards ingratiating herself with the celebrities she stalks than towards harming them.

12 In context, “employ” (line 32) most nearly means

ANSWERS AND EXPLANATIONS Explanation for Correct Answer B :  Choice (B) is correct. In this sentence, the narrator is describing being sent by Mrs. Van Hopper on useless errands whose sole purpose is to attract the attention of someone Mrs. Van Hopper wants to befriend. The narrator feels, under these circumstances, that she is being employed as “bait”—in other words, “used.”

Explanation for Incorrect Answer A :  Choice (A) is incorrect. In this sentence, the narrator is describing being sent by Mrs. Van Hopper on useless errands whose sole purpose is to attract the attention of someone Mrs. Van Hopper wants to befriend. In this context, it makes little sense to say that the narrator is being “serviced,” or provided with a service, when in fact she is providing a humiliating “service” for Mrs. Van Hopper.

Explanation for Incorrect Answer C :  Choice (C) is incorrect. In this sentence, the narrator is describing being sent by Mrs. Van Hopper on useless errands whose sole purpose is to attract the attention of someone Mrs. Van Hopper wants to befriend. In this context, it makes little sense to say that the narrator is being “attracted,” or having her attention solicited, when in fact she is being sent by Mrs. Van Hopper as "bait" to attract the latter's "prey."

Explanation for Incorrect Answer D :  Choice (D) is incorrect. In this sentence, the narrator is describing being sent by Mrs. Van Hopper on useless errands whose sole purpose is to attract the attention of someone Mrs. Van Hopper wants to befriend. In this context, it does not make sense to say that the narrator is being “devoted,” or dedicated, as “bait” to attract Mrs. Van Hopper’s “prey.”

Explanation for Incorrect Answer E :  Choice (E) is incorrect. In this sentence, the narrator is describing being sent by Mrs. Van Hopper on useless errands whose sole purpose is to attract the attention of someone Mrs. Van Hopper wants to befriend. Although the narrator is clearly “hired,” or employed, by Mrs. Van Hopper, in this sentence she is speaking of being sent on useless errands in order to attract the attention of someone Mrs. Van Hopper wants to befriend; she is being “used.”

(A) service

(B) use

(C) attract

(D) devote

(E) hire

页码,13/24The Official SAT Online Course

2006-11-12file://E:\新建文件夹\e5.htm

UnRe

gistered

Page 55: SAT practice test 5.pdf

严禁用于商业用途!

13 It is clear from context that by “debating” (line 49), the narrator means

ANSWERS AND EXPLANATIONS Explanation for Correct Answer B :  Choice (B) is correct. In this part of the passage, the narrator is speaking of one “unforgettable afternoon” when Mrs. Van Hopper is indulging in her usual predatory behavior towards celebrities; on this occasion, she has one special “victim” in mind. She has installed herself in her customary seat, and is “debating,” or deliberating, about which “method of attack” (or way of entrapping the stranger) to use. It is clear from context that she is reviewing “methods” in her head and not aloud—the narrator can “tell” by Mrs. Van Hopper’s actions what the latter is thinking about.

Explanation for Incorrect Answer A :  Choice (A) is incorrect. In this part of the passage, it is clear from context that Mrs. Van Hopper is reviewing “methods of attack”—that is, ways of entrapping a certain stranger—in her head and not aloud. She is clearly thinking silently and not “asking advice about” ways to proceed. The narrator can “tell” by Mrs. Van Hopper’s actions what the latter is thinking about.

Explanation for Incorrect Answer C :  Choice (C) is incorrect. In this part of the passage, it is clear from context that Mrs. Van Hopper is reviewing “methods of attack”—that is, ways of entrapping a certain stranger—in her head and not aloud. She is clearly thinking silently and not “talking aloud to herself about” ways to proceed. The narrator can “tell” by Mrs. Van Hopper’s actions what the latter is thinking about.

Explanation for Incorrect Answer D :  Choice (D) is incorrect. In this part of the passage, it is clear from context that Mrs. Van Hopper is reviewing “methods of attack”—that is, ways of entrapping a certain stranger—in her head and not aloud. She is clearly thinking silently and not “taking notes on” ways to proceed. The narrator can “tell” by Mrs. Van Hopper’s actions what the latter is thinking about.

Explanation for Incorrect Answer E :  Choice (E) is incorrect. In this part of the passage, it is clear from context that Mrs. Van Hopper is reviewing “methods of attack”—that is, ways of entrapping a certain stranger—in her head and not aloud. She is clearly thinking silently and not “arguing” with someone else about ways to proceed. The narrator can “tell” by Mrs. Van Hopper’s actions what the latter is thinking about.

(A) asking advice about

(B) considering strategies for

(C) talking aloud to herself about

(D) taking notes on

(E) arguing about

14It can be inferred that Mrs. Van Hopper sends her companion to retrieve something from upstairs primarily with the goal of

ANSWERS AND EXPLANATIONS Explanation for Correct Answer E :  Choice (E) is correct. We know from context that Mrs. Van Hopper is obsessed with meeting celebrities, and that she has used the narrator as “bait” to draw the intended “prey.” In the paragraph directly preceding the quotation, Mrs. Van Hopper is “debating her method of attack.” When, after this, she speaks, it is

(A) communicating with a relative

(B) having some time alone

(C) keeping information from the companion

(D) arranging a meeting for the companion

(E) establishing a connection with a stranger

页码,14/24The Official SAT Online Course

2006-11-12file://E:\新建文件夹\e5.htm

UnRe

gistered

Page 56: SAT practice test 5.pdf

严禁用于商业用途!

clearly because she has hit upon an idea for meeting the stranger: she sends the narrator to get something. In the next paragraph, the narrator says, “I saw then that…the nephew was to be the means of introduction.” Clearly, the errand is intended to “establish a connection with a stranger.”

Explanation for Incorrect Answer A :  Choice (A) is incorrect. While it is true that Mrs. Van Hopper sends the narrator to get a “communication from a relative”—that is, the postcard from the nephew—the goal of the errand is clearly not “communicating with a relative,” but getting acquainted with another hotel guest, a “new arrival.”

Explanation for Incorrect Answer B :  Choice (B) is incorrect. While it is true that sending the narrator to get something from upstairs will leave Mrs. Van Hopper momentarily alone, being alone is clearly not the latter’s goal; rather, she is sending the narrator to retrieve an object that will help Mrs. Van Hopper get acquainted with another hotel guest, a “new arrival.”

Explanation for Incorrect Answer C :  Choice (C) is incorrect. While one might send a companion on an errand in order to keep him or her from finding something out, that is clearly not the case here. Mrs. Van Hopper is not interested in what the narrator knows or does not know; rather, she is sending the narrator to retrieve an object that will help Mrs. Van Hopper get acquainted with another hotel guest, a “new arrival.”

Explanation for Incorrect Answer D :  Choice (D) is incorrect. While the long-term goal of the errand may be to “arrange a meeting,” the meeting is clearly meant to be between Mrs. Van Hopper and “the new arrival,” not between the companion and someone else.

15 In lines 64-70 (“I saw...cue”), the narrator imagines herself as a

ANSWERS AND EXPLANATIONS Explanation for Correct Answer C :  Choice (C) is correct. In the sentences cited, the narrator refers to the “part” she must play in Mrs. Van Hopper’s “schemes”: “Like a juggler’s assistant I produced the props, then silent and attentive I waited on my cue.” A stage “juggler’s assistant,” who must assist the performer by providing props (or articles to be used on stage) on cue (or when signaled) is a kind of “stagehand.”

Explanation for Incorrect Answer A :  Choice (A) is incorrect. In the sentences cited, the narrator refers to the “part” she must play in Mrs. Van Hopper’s “schemes”: “Like a juggler’s assistant I produced the props, then silent and attentive I waited on my cue.” While the imagery does have to do with the theatre, the narrator is imagining herself not as a writer, but, rather, as an assistant, or “stagehand.”

Explanation for Incorrect Answer B :  Choice (B) is incorrect. In the sentences cited, the narrator refers to the “part” she must play in Mrs. Van Hopper’s “schemes”: “Like a juggler’s assistant I produced the props, then silent and attentive I waited on my cue.” While the imagery does have to do with the theatre, and while actors do “wait on cues,” the narrator is imagining herself as someone who assists a performer, “producing props and waiting on my cue”—in other words, a “stagehand.”

Explanation for Incorrect Answer D :  Choice (D) is incorrect. In the sentences cited, the narrator refers to the “part”

(A) playwright

(B) actor

(C) stagehand

(D) criminal

(E) magician

页码,15/24The Official SAT Online Course

2006-11-12file://E:\新建文件夹\e5.htm

UnRe

gistered

Page 57: SAT practice test 5.pdf

严禁用于商业用途!

she must play in Mrs. Van Hopper’s “schemes”: “Like a juggler’s assistant I produced the props, then silent and attentive I waited on my cue.” There is no reference here to criminality; rather, the narrator is imagining herself as someone who assists a performer—in other words, a “stagehand.”

Explanation for Incorrect Answer E :  Choice (E) is incorrect. In the sentences cited, the narrator refers to the “part” she must play in Mrs. Van Hopper’s “schemes”: “Like a juggler’s assistant I produced the props, then silent and attentive I waited on my cue.” While the imagery does have to do with stage juggling, there is no real reference to magic: a juggler is not really a magician. Further, the narrator is imagining herself as someone who assists a performer—in other words, a “stagehand.”

16 On the whole, the passage is developed in which of the following ways?

ANSWERS AND EXPLANATIONS Explanation for Correct Answer D :  Choice (D) is correct. The passage begins with a general description of Mrs. Van Hopper’s customary behavior—deliberately and methodically stalking celebrity visitors to the Cote d’Azur hotel. The reader is both led to believe and told directly that this pattern has recurred for years. About halfway through the passage, however, the narrator begins to focus on a specific instance of this behavior: “on that unforgettable afternoon…Suddenly she turned to me, her small eyes alight.” This narrative takes up the rest of the passage. So the method of development of the passage is that of “a general description followed by a specific example.”

Explanation for Incorrect Answer A :  Choice (A) is incorrect. While a single episode is narrated within the passage—the last half or so of the passage takes place on one “unforgettable afternoon”—the passage as a whole is not composed of a single extended narration. Rather, the first half of the passage is taken up with a general description of Mrs. Van Hopper’s behavior.

Explanation for Incorrect Answer B :  Choice (B) is incorrect. While there is what might be termed “dialogue” in the passage—the direct quotation from Mrs. Van Hopper in the fourth paragraph—this speech is only a very small part of the passage as a whole and does not explain the passage’s development.

Explanation for Incorrect Answer C :  Choice (C) is incorrect. The “description” that makes up the first part of the passage is about a certain pattern of behavior and cannot properly be called a “physical” description, as of an object or a place. Further, the latter part of the passage, since it narrates a single event, cannot properly be called a “summary.”

Explanation for Incorrect Answer E :  Choice (E) is incorrect. While the first part of the passage, in which Mrs. Van Hopper’s customary behavior is described, might be called a kind of “character description,” there is only one speech in the passage—Mrs. Van Hopper’s instructions in the fourth paragraph—so the passage cannot be said to end with a “monologue,” or long speech.

(A) a single extended episode is narrated

(B) a dialogue is recounted

(C) a physical description is followed by a summary

(D) a general description is followed by a specific example

(E) a character description is followed by a monologue

17 In context, “garnered” (line 74) is closest in meaning to

(A) gathered

(B) earned

assumed

页码,16/24The Official SAT Online Course

2006-11-12file://E:\新建文件夹\e5.htm

UnRe

gistered

Page 58: SAT practice test 5.pdf

严禁用于商业用途!

ANSWERS AND EXPLANATIONS Explanation for Correct Answer A :  Choice (A) is correct. In this sentence, the narrator of the passage is speaking of Mrs. Van Hopper’s having “garnered” information about the "newcomer” from “daily papers.” In context, it makes sense to say that Mrs. Van Hopper “gathered,” or acquired, this information from her newspaper reading.

Explanation for Incorrect Answer B :  Choice (B) is incorrect. In this sentence, the narrator of the passage is speaking of Mrs. Van Hopper’s having “garnered” information about the "newcomer” from “daily papers.” In context, it makes no sense to say that Mrs. Van Hopper “earned,” or merited, this information from her newspaper reading.

Explanation for Incorrect Answer C :  Choice (C) is incorrect. In this sentence, the narrator of the passage is speaking of Mrs. Van Hopper’s having “garnered” information about the "newcomer” from “daily papers.” In context, it makes little sense to say that Mrs. Van Hopper “assumed,” or took upon herself, this information from her newspaper reading.

Explanation for Incorrect Answer D :  Choice (D) is incorrect. In this sentence, the narrator of the passage is speaking of Mrs. Van Hopper’s having “garnered” information about the "newcomer” from “daily papers.” In context, it makes little sense to say that Mrs. Van Hopper “inferred,” or concluded, this information from her newspaper reading.

Explanation for Incorrect Answer E :  Choice (E) is incorrect. In this sentence, the narrator of the passage is speaking of Mrs. Van Hopper’s having “garnered” information about the "newcomer” from “daily papers.” In context, it makes little sense to say that Mrs. Van Hopper “harvested,” or gathered a crop from, this information in her newspaper reading.

(C)

(D) inferred

(E) harvested

       New York in the wake of World War  II was a city on the verge of momentous  changes—economic, social, and political.Line For almost a century it had been a5 preeminent manufacturing and port city,  absorbing the unskilled millions who  flocked there from Europe, and had  yielded great fortunes for the astute and  daring. The Depression of the 1930’s had10 exacted a terrible toll, and leaders  conferred anxiously on how to avoid a  repetition of those doleful days as the war  economy wound down.       Even before the war, experts had15 been cautioning New York that it was  losing industry and business to other  locales. A 1939 study showed the City’s  percentage of wage earners down in fifty-  eight industries.20      At the close of World War II, the  great port of New York remained the

页码,17/24The Official SAT Online Course

2006-11-12file://E:\新建文件夹\e5.htm

UnRe

gistered

Page 59: SAT practice test 5.pdf

严禁用于商业用途!

  largest general cargo port in the country  and the second largest in the world,  deferring only to Rotterdam. Week in and25 week out forty thousand longshoremen  worked the ninety-six piers encircling  Manhattan and the eighty lining the  Brooklyn waterfront, manually loading and  unloading sixteen million tons a year *.30 Hundreds of tugs and barges plied the  harbor and nearby rivers, guiding the big  ships to their berths and ferrying cargo.  Looking ahead, the new Port Authority of  New York cleared forty old piers in35 Brooklyn and replaced them with fourteen  spacious, modern piers, the best in the  world.       In the mid-1950’s Malcolm McLean  of Winston-Salem invented a method of40 shipping that was to revolutionize cargo  ports and make the new piers obsolete.  His brainchild was the truck-sized sealed  containers that slashed loading and  unloading time to almost nothing. A small45 crew of men could use cranes to load the  gargantuan containers, filling a ship  twenty times faster than the old gangs  grappling with crates, boxes, and bales.  The impregnable containers guaranteed50 against waterfront theft or breakage,  eliminating altogether the dockside  pilferage that previously had consumed up  to 15 percent of some cargoes. Ships that  had traditionally spent a week in port55 could now finish their loading in a day.       The new containers required huge  storing areas, far larger than were  available in either Manhattan or Brooklyn.  When the City of New York proposed60 modernizing its East River piers to handle  the containers, the Port Authority said it  would have to clear all the land from the  river west to Third Avenue to do so. Each  berth for a ship carrying containers of65 cargo needed fifty acres of surrounding  land, compared to an old-fashioned berth  of 195,000 square feet. The Port  Authority erected container ports at Port  Newark and Port Elizabeth, New Jersey,70 with their vast stretches of empty land.  The old piers of Manhattan and Brooklyn  languished—rotting, deserted white 

页码,18/24The Official SAT Online Course

2006-11-12file://E:\新建文件夹\e5.htm

UnRe

gistered

Page 60: SAT practice test 5.pdf

严禁用于商业用途!

elephants. The ranks of the  longshoremen, once forty thousand75 strong, dwindled to nine thousand. In the  new technical, mechanized world of  container shipping, man-hours fell from  40 million man-hours right after the war  to 13.5 million in 1983.

 

* The island of Manhattan is bounded by the Hudson River and the state of New Jersey to the west and by the East River and the New York City borough of Brooklyn on the east.

18 In line 5, “preeminent” is closest in meaning to

ANSWERS AND EXPLANATIONS Explanation for Correct Answer B :  Choice (B) is correct. The first sentence of the paragraph says that post-World War II New York was poised for important changes; the second sentence goes on to discuss the city’s longtime importance as a shipping and manufacturing center. The comparison between the past and present is implicit: New York “had been” a “preeminent,” or outstanding, hub for industries that were now declining in importance.

Explanation for Incorrect Answer A :  Choice (A) is incorrect. The sentence is clearly saying that New York “had been” a “preeminent,” or outstanding, hub for industries that were declining in importance. It makes little sense to say that the city had been “knowledgeable,” or knowing.

Explanation for Incorrect Answer C :  Choice (C) is incorrect. The first sentence of the paragraph says that post-World War II New York was poised for important changes; the second sentence goes on to discuss the city’s longtime importance as a shipping and manufacturing center. While it makes a certain amount of sense to call New York a “growing” industrial center, or one increasing in size or importance, the comparison between past and present is implied by the context: New York “had been” a “preeminent,” or outstanding, hub for industries that were now declining in importance.

Explanation for Incorrect Answer D :  Choice (D) is incorrect. The sentence is clearly saying that New York “had been” a “preeminent,” or outstanding, hub for industries that were declining in importance. It makes little sense in context to say that the city had been “abnormal,” or deviating from normal, as an industrial center.

Explanation for Incorrect Answer E :  Choice (E) is incorrect. The first sentence of the paragraph says that post-World War II New York was poised for important changes; the second sentence goes on to discuss the city’s longtime importance as a shipping and manufacturing center. “Notorious” means “widely but unfavorably known”; so while it might make sense to call attention to New York’s fame as an industrial center, there is no reason to assume that its fame was accompanied by something negative.

(A) knowledgeable

(B) outstanding

(C) growing

(D) abnormal

(E) notorious

页码,19/24The Official SAT Online Course

2006-11-12file://E:\新建文件夹\e5.htm

UnRe

gistered

Page 61: SAT practice test 5.pdf

严禁用于商业用途!

19 In context, the “unskilled millions” (line 6) apparently refers to people who

ANSWERS AND EXPLANATIONS Explanation for Correct Answer C :  Choice (C) is correct. In these lines, the author is speaking of the century-long importance of New York as a “manufacturing and port city,” a city that had both absorbed “the unskilled millions who flocked there from Europe” and “yielded great fortunes for the astute and daring.” It is clear from context that the overriding topic is the New York City economy, which had been thriving for a century before World War II but which was expected to falter somewhat in the post-War years. In this reading, the “astute and daring” are those who made great sums of money by speculation, and the “unskilled millions” the European emigrants who had been employed—or absorbed—in manufacturing and seaport trades during the city’s period of greatest importance.

Explanation for Incorrect Answer A :  Choice (A) is incorrect. In these lines, the author is speaking of the century-long importance of New York as a “manufacturing and port city,” a city that had both absorbed “the unskilled millions who flocked there from Europe” and “yielded great fortunes for the astute and daring.” It is clear from context that the latter—those who made great fortunes—are separate from the European emigrants, who clearly were employed—or absorbed by—the manufacturing and seaport trades.

Explanation for Incorrect Answer B :  Choice (B) is incorrect. While this paragraph does mention the Depression, it does so in the context of the Depression’s having “exacted a terrible toll”—or had a disastrous effect—on New York City’s economy. It is clear from context that the “unskilled millions” had found rather than lost jobs in the several decades before the Depression of the 1930’s.

Explanation for Incorrect Answer D :  Choice (D) is incorrect. While this paragraph does imply that various people were uneasy about the New York economy in the wake of World War II, it is clearly business “leaders,” not the “unskilled millions” of workers who had found employment in the preceding century, who are experiencing this anxiety.

Explanation for Incorrect Answer E :  Choice (E) is incorrect. While there is a suggestion in this paragraph that New York City was poised to lose jobs in the post-World War II period, there is no discussion of people who left the area in order to find work.

(A) made great fortunes

(B) were unprepared for the Depression of the 1930’s

(C) took jobs in shipping and manufacturing

(D) were uneasy about the U.S. economy

(E) left New York to find work

20The third paragraph is best described as a description of

ANSWERS AND EXPLANATIONS Explanation for Correct Answer A :  Choice (A) is correct. The third paragraph of the passage clearly discusses the state of New York City’s shipping industry “at the close of World War II.” But as the rest of the passage makes clear, this was an industry whose circumstances were about to change drastically—and for the worse—in the decade of the 1950’s; shipping as it was carried on the 1940’s was about to become “obsolete.”

(A) a process that would soon be obsolete

(B) a blueprint for changing an area

(C) a plan that drew much criticism

(D) a decline in the importance of an industry

(E) an event that foreshadowed future happenings

页码,20/24The Official SAT Online Course

2006-11-12file://E:\新建文件夹\e5.htm

UnRe

gistered

Page 62: SAT practice test 5.pdf

严禁用于商业用途!

Explanation for Incorrect Answer B :  Choice (B) is incorrect. The only real discussion of “changing an area” occurs in the fifth paragraph of the passage, not in the third; and this change—the decline of the old New York piers—comes about gradually and unintentionally, not according to a “blueprint.”

Explanation for Incorrect Answer C :  Choice (C) is incorrect. While it might be inferable that the shipping revolution described in the fourth paragraph would “draw criticism,” there is no indication that the description given in the third paragraph did so; further, it is more a description of a process than it is a “plan.”

Explanation for Incorrect Answer D :  Choice (D) is incorrect. While the third paragraph of the passage does indeed discuss the shipping industry—an industry, as the rest of the passage tells us, that would soon decline—this paragraph discusses New York shipping at its height, not at the time of its “decline in importance.”

Explanation for Incorrect Answer E :  Choice (E) is incorrect. While the last sentence in the third paragraph might be construed as a “foreshadowing” or indication of future events—the clearing of the old piers to make room for the new piers, which would themselves become obsolete—the great majority of this paragraph is devoted to discussing the shipping industry of New York at its height, not in its decline.

21 The author considers Malcolm McLean to be

ANSWERS AND EXPLANATIONS Explanation for Correct Answer E :  Choice (E) is correct. An “innovator” is someone who introduces important changes. Clearly the author of the passage sees McLean, who invented a revolutionary method of shipping that made the old methods (and piers) “obsolete,” as an innovator.

Explanation for Incorrect Answer A :  Choice (A) is incorrect. A “dreamer” is someone who lives in a world of unreality. A dreamer may have great ideas, but they are usually of an impractical nature. Clearly the author of the passage sees McLean, whose invention was both revolutionary and practical, as an innovator rather than as a dreamer.

Explanation for Incorrect Answer B :  Choice (B) is incorrect. An “opportunist” is someone who takes advantage of opportunities, but who does so with little thought for principle or ethics. While the author of the passage clearly considers McLean someone who seized opportunities, there is no indication in the passage that McLean was unprincipled or unethical.

Explanation for Incorrect Answer C :  Choice (C) is incorrect. An “eccentric” is someone who behaves in an unconventional manner. While there is a conception in popular culture of inventors being eccentric, there is no indication in this passage that the author considers McLean or his behavior unconventional.

Explanation for Incorrect Answer D :  Choice (D) is incorrect. An “obstructionist” is someone who deliberately interferes with progress. Far from considering McLean as an obstacle to progress, the author clearly sees McLean as someone who aided or sped progress—an innovator.

(A) a dreamer

(B) an opportunist

(C) an eccentric

(D) an obstructionist

(E) an innovator

页码,21/24The Official SAT Online Course

2006-11-12file://E:\新建文件夹\e5.htm

UnRe

gistered

Page 63: SAT practice test 5.pdf

严禁用于商业用途!

22 The passage mentions each of the following as an advantage of container shipping EXCEPT:

ANSWERS AND EXPLANATIONS Explanation for Correct Answer A :  Choice (A) is correct. While the passage does indeed discuss “large storing areas” in the last paragraph, this characteristic of container shipping is presented more as a disadvantage than an advantage. Container shipping needed “huge storing areas, far larger than were available in either Manhattan or Brooklyn.” Further, the plan to modernize New York’s piers proved impractical, leaving the old piers “languishing” and “rotting.”

Explanation for Incorrect Answer B :  Choice (B) is incorrect. The author of the passage presents reduced loading time as a decided advantage of container shipping, in that it “slashed loading and unloading time to almost nothing”—“Ships that had traditionally spent a week in port could now finish their loading in a day.”

Explanation for Incorrect Answer C :  Choice (C) is incorrect. The author of the passage presents theft deterrence as an advantage of container shipping: “The impregnable containers guaranteed against waterfront theft . . . eliminating the dockside pilferage that previously had consumed up to 15 percent of some cargoes.”

Explanation for Incorrect Answer D :  Choice (D) is incorrect. The author of the passage mentions personnel reduction in a favorable light in paragraph 4: “A small crew of men could use cranes to load the gargantuan containers, filling a ship twenty times faster than the old gangs grappling with crates, boxes, and bales.”

Explanation for Incorrect Answer E :  Choice (E) is incorrect. The author of the passage presents breakage prevention as an advantage of container shipping: “The impregnable containers guaranteed against waterfront theft or breakage,” implying that both theft and breakage were problems associated with older shipping practices.

(A) large storing areas

(B) reduced loading time

(C) theft deterrence

(D) personnel reduction

(E) breakage prevention

23 According to the passage, the City of New York was unable to carry out its pier modernization plan because which of the following was lacking?

ANSWERS AND EXPLANATIONS Explanation for Correct Answer B :  Choice (B) is correct. The first two sentences of the last paragraph make it clear that there was not sufficient space available to fully adapt New York's piers to the new container shipping system: “The new containers required huge storing areas, far larger than were available in either Manhattan or Brooklyn. When the City of New York proposed modernizing its East River piers to handle the containers, the Port Authority said it would have to clear all the land from the river west to Third

(A) Money

(B) Space

(C) Time

(D) Materials

(E) Motivation

页码,22/24The Official SAT Online Course

2006-11-12file://E:\新建文件夹\e5.htm

UnRe

gistered

Page 64: SAT practice test 5.pdf

严禁用于商业用途!

Avenue to do so.”

Explanation for Incorrect Answer A :  Choice (A) is incorrect. The passage does not mention a lack of financial resources as being behind the failure of the pier modernization plan.

Explanation for Incorrect Answer C :  Choice (C) is incorrect. The only mention of time in connection with container shipping in the passage is of how much time was saved in comparison to the older shipping methods; there is no discussion of there not being sufficient time to modernize the New York piers.

Explanation for Incorrect Answer D :  Choice (D) is incorrect. The passage does not mention a lack of materials as being behind the failure of the pier modernization plan.

Explanation for Incorrect Answer E :  Choice (E) is incorrect. The passage illustrates that the City of New York was, on the contrary, highly motivated to modernize its piers, but that it lacked the space required to do so.

24 In the final paragraph of the passage, the author presents an example of

ANSWERS AND EXPLANATIONS Explanation for Correct Answer D :  Choice (D) is correct. The last few sentences of the passage create an unmistakable image of the desolation brought about by modernization: the useless piers, languishing like “white elephants”; the loss of shipping jobs created by automation. These are bleak images of the down side of technological progress.

Explanation for Incorrect Answer A :  Choice (A) is incorrect. While the proposal to modernize New York’s piers might conceivably be called a “vision for the future,” the proposal is a failed one, and there is no attempt to connect it to “one man.”

Explanation for Incorrect Answer B :  Choice (B) is incorrect. While the paragraph might be said to contain certain contrasts between container shipping and older shipping methods, these are not “two large [and separate] industries.”

Explanation for Incorrect Answer C :  Choice (C) is incorrect. The paragraph mentions New York and two cities in New Jersey, but it is not concerned with their “rapid growth.”

Explanation for Incorrect Answer E :  Choice (E) is incorrect. The dilemma presented in the paragraph is not described as being the result of political corruption, but of modernization and automation.

(A) one man’s vision for the future

(B) the contrasts between two large industries

(C) the rapid growth of a city

(D) a negative aspect of modernization

(E) the results of political corruption

    

Back to Score Report  

Copyright © 2006 The College Board. All rights reserved. Privacy Policy Terms of Use Contact Us

页码,23/24The Official SAT Online Course

2006-11-12file://E:\新建文件夹\e5.htm

UnRe

gistered

Page 65: SAT practice test 5.pdf

严禁用于商业用途!

 

页码,24/24The Official SAT Online Course

2006-11-12file://E:\新建文件夹\e5.htm

UnRe

gistered

Page 66: SAT practice test 5.pdf

严禁用于商业用途!

Help | Profile | My Organizer | My Bookmarks | Logout

Answers and Explanations

Test Sections

Section 1

Section 2

Section 3

Section 4

Section 5

Section 6

Section 8

Section 9

Section 10

Back to Score Report  

View Answers and Explanations     Online - Practice Test #5

1 There are bookcases in a house. Each bookcase contains at least books but not more than books. Which of the following could be the total number of books in all bookcases?

ANSWERS AND EXPLANATIONS Explanation for Correct Answer D :  Choice (D) is correct. If each of the bookcases contained the maximum number

of books, then the total number of books would be If each

bookcase contained the minimum number of books, then the total number of

books would be Therefore, the total number of books in all

bookcases must be at least but no more than Of the choices, only is in this range.

Explanation for Incorrect Answer A :  Choice (A) is not correct. Since each bookcase contains at least books, the

total number of books in all bookcases must be at least Therefore, the

total number of books cannot be

Explanation for Incorrect Answer B :  Choice (B) is not correct. Since each bookcase contains at least books, the

total number of books in all bookcases must be at least Therefore, the

total number of books cannot be

Explanation for Incorrect Answer C :  Choice (C) is not correct. Since each bookcase contains at least books, the

total number of books in all bookcases must be at least Therefore, the

total number of books cannot be

Explanation for Incorrect Answer E :  Choice (E) is not correct. Since each bookcase contains no more than books,

the total number of books in all bookcases can be no more than

Therefore, the total number of books cannot be

(A)

(B)

(C)

(D)

(E)

2

页码,1/12The Official SAT Online Course

2006-11-12file://E:\新建文件夹\e6.htm

UnRe

gistered

Page 67: SAT practice test 5.pdf

严禁用于商业用途!

In the figure above, lines and are parallel. If what is the value of

ANSWERS AND EXPLANATIONS Explanation for Correct Answer E :  Choice (E) is correct. Use the properties of parallel lines cut by a transversal.

Alternate interior angles are supplementary. Thus, in the figure above,

Since it follows that Since vertical

angles are of equal measure, it follows that Corresponding angles are also

of equal measure. Thus, The angles labeled and are

supplementary, so Therefore,

Explanation for Incorrect Answer A :  Choice (A) is not correct. Although from the figure, it may appear that

and the figure is not drawn to scale.

Explanation for Incorrect Answer B :  Choice (B) is not correct. This incorrect answer may be the result of estimating angle measures from the figure instead of reasoning from geometric principles.

Explanation for Incorrect Answer C :  Choice (C) is not correct. Although from the figure, it may appear that

and the figure is not drawn to scale.

Explanation for Incorrect Answer D :  Choice (D) is not correct. This incorrect answer may be the result of estimating angle measures from the figure instead of reasoning from geometric principles.

(A)

(B)

(C)

(D)

(E)

3

页码,2/12The Official SAT Online Course

2006-11-12file://E:\新建文件夹\e6.htm

UnRe

gistered

Page 68: SAT practice test 5.pdf

严禁用于商业用途!

Graphed above is the amount that a computer shop charges for a repair job as a function of the number of hours required to do the job. Which of the following is most consistent with the information in the graph?

ANSWERS AND EXPLANATIONS Explanation for Correct Answer D :  Choice (D) is correct. The graph is a horizontal line segment up to hours, which corresponds to a fixed amount charged for the first hours or less of work. Then the graph is a line segment with positive slope, corresponding to an hourly rate for work beyond hours.

Explanation for Incorrect Answer A :  Choice (A) is not correct. If the shop charged the same amount for any job of any length, then the graph would be a horizontal line, which it is not.

Explanation for Incorrect Answer B :  Choice (B) is not correct. If the shop charged at an hourly rate for work with no fixed initial amount, then the graph would be a line through the origin, which it is not.

Explanation for Incorrect Answer C :  Choice (C) is not correct. If the shop charged a fixed initial amount plus an hourly rate for work, starting at the beginning of the first hour, then the graph would show a greater charge for 2 hours than for hour, but these two charges are the same.

Explanation for Incorrect Answer E :  Choice (E) is not correct. If the shop charged at an hourly rate for work beginning at the start of the first hour, and the rate decreased after hours of work, then the slope of the graph would decrease after hours, but it does not.

(A) The shop charges the same amount for any job of any length.

(B) The shop charges at an hourly rate for work with no fixed initial amount.

(C) The shop charges a fixed initial amount plus an hourly rate for work, starting at the beginning of the first hour.

(D) The shop charges a fixed amount for the first hours or less of work. The shop charges at an hourly rate for work beyond hours.

(E) The shop charges at an hourly rate for work beginning at the start of the first hour. This rate decreases after hours of work.

4

For how many values of is the equation above true?

(A) None

页码,3/12The Official SAT Online Course

2006-11-12file://E:\新建文件夹\e6.htm

UnRe

gistered

Page 69: SAT practice test 5.pdf

严禁用于商业用途!

ANSWERS AND EXPLANATIONS Explanation for Correct Answer C : 

Choice (C) is correct. The equation means that either or

The solutions of these last two equations are and Both

these values of satisfy and they are the only two values that satisfy

Explanation for Incorrect Answer A : 

Choice (A) is not correct. The values and both satisfy

Explanation for Incorrect Answer B : 

Choice (B) is not correct. The values and both satisfy

Explanation for Incorrect Answer D : 

Choice (D) is not correct. The values and both satisfy

but they are the only two values of that satisfy

Explanation for Incorrect Answer E : 

Choice (E) is not correct. The values and both satisfy

but they are the only two values of that satisfy

(B) One

(C) Two

(D) Four

(E) More than four

5

An amusement park awards tickets that can be exchanged for prizes to reward high scores at an arcade game. Some scores and the corresponding number of prize tickets awarded are shown in the table above. For scores from up to the number of tickets awarded increases by a constant amount for each -point score

increase. How many tickets would be awarded for a score of

(A)

(B)

(C)

页码,4/12The Official SAT Online Course

2006-11-12file://E:\新建文件夹\e6.htm

UnRe

gistered

Page 70: SAT practice test 5.pdf

严禁用于商业用途!

ANSWERS AND EXPLANATIONS Explanation for Correct Answer D :  Choice (D) is correct. For a score of points, there are tickets awarded, and

for a score of there are tickets awarded. For this -point score

increase, the increase in the number of tickets awarded is For scores from

up to the number of tickets awarded increases by a constant amount for each

-point score increase. Thus, any -point score increase within this interval

increases the number of tickets awarded by Therefore, since tickets are

awarded for a score of points, it follows that tickets are

awarded for a score of points, and tickets are awarded for a

score of

Explanation for Incorrect Answer A :  Choice (A) is not correct. The number of tickets awarded for a score of points

is but the number of tickets awarded for a score of is

Explanation for Incorrect Answer B :  Choice (B) is not correct. The number of tickets awarded for a score of points

is but the number of tickets awarded for a score of is

Explanation for Incorrect Answer C :  Choice (C) is not correct. Since a -point score increase results in an additional

ticket awarded, the number of tickets awarded for a score of is

not

Explanation for Incorrect Answer E :  Choice (E) is not correct. The number of tickets awarded for a score of points

is but the number of tickets awarded for a score of

is

(D)

(E)

6

There are students in a class, only one of whom is a junior. If seniors are added to the class, how many students in the class will NOT be juniors?

(A)

(B)

(C)

(D)

(E)

页码,5/12The Official SAT Online Course

2006-11-12file://E:\新建文件夹\e6.htm

UnRe

gistered

Page 71: SAT practice test 5.pdf

严禁用于商业用途!

ANSWERS AND EXPLANATIONS Explanation for Correct Answer D :  Choice (D) is correct. There are originally students in the class, only one of whom is a junior. Thus, students in the original class are not juniors. Then seniors are added to the class; since these students are seniors, they are not juniors. Therefore, after the seniors are added to the class, there are

students in the class who are not juniors.

Explanation for Incorrect Answer A :  Choice (A) is not correct. If exactly students in the class were not juniors after the seniors had been added to the class, then originally there would have

been students in the class who were not juniors. But originally

there were students who were not juniors.

Explanation for Incorrect Answer B :  Choice (B) is not correct. If exactly students in the class were not juniors after the seniors had been added to the class, then originally there would have

been students in the class who were not juniors. But originally

there were students who were not juniors.

Explanation for Incorrect Answer C :  Choice (C) is not correct. If exactly students in the class were not juniors

after the seniors had been added to the class, then originally there would have

been students in the class who were not juniors. But originally

there were students who were not juniors.

Explanation for Incorrect Answer E :  Choice (E) is not correct. After the seniors were added to the class, there were a

total of students in the class. However, the question asks how many of these

students were not juniors, which is

7

The quadratic function graphed above has equation Which of the

following must be true?

ANSWERS AND EXPLANATIONS Explanation for Correct Answer A : 

Choice (A) is correct. Since the point is on the graph

of Thus, the intersection of the graph of and the -axis is units above the

origin. Since the graph of intersects the positive -axis, the value of must be

(A)

(B)

(C) is even.

(D) is odd.

(E) is the square of an integer.

页码,6/12The Official SAT Online Course

2006-11-12file://E:\新建文件夹\e6.htm

UnRe

gistered

Page 72: SAT practice test 5.pdf

严禁用于商业用途!

positive. It cannot be negative. The value of may be even, odd, or the square of an integer, but it need not be. Therefore, the only one of the five statements that must be true is

Explanation for Incorrect Answer B : 

Choice (B) is not correct. The point is the intersection of the graph of and

the -axis. Since the graph of intersects the positive -axis, the value of cannot be negative.

Explanation for Incorrect Answer C :  Choice (C) is not correct. It may be true that is an even integer, but it need not be true.

Explanation for Incorrect Answer D :  Choice (D) is not correct. It may be true that is an odd integer, but it need not be true.

Explanation for Incorrect Answer E :  Choice (E) is not correct. It may be true that is the square of an integer, but it need not be true.

8

Which of the following inequalities is true about the lengths and of the sides of the triangle above?

ANSWERS AND EXPLANATIONS Explanation for Correct Answer E :  Choice (E) is correct. The length of each side of a triangle is less than the sum of the lengths of the other two sides; thus, Squaring both sides of this

inequality yields

Explanation for Incorrect Answer A :  Choice (A) is not correct. The length of each side of a triangle is less than the sum of the lengths of the other two sides; thus, Squaring both sides of this

inequality yields not

Explanation for Incorrect Answer B :  Choice (B) is not correct. The length of each side of a triangle is less than the sum of the lengths of the other two sides; thus, Squaring both sides of this

inequality yields not

(A)

(B)

(C)

(D)

(E)

页码,7/12The Official SAT Online Course

2006-11-12file://E:\新建文件夹\e6.htm

UnRe

gistered

Page 73: SAT practice test 5.pdf

严禁用于商业用途!

Explanation for Incorrect Answer C :  Choice (C) is not correct. The length of each side of a triangle is less than the sum of the lengths of the other two sides; thus, Squaring both sides of this

inequality yields not

Explanation for Incorrect Answer D :  Choice (D) is not correct. The length of each side of a triangle is less than the sum of the lengths of the other two sides; thus, Squaring both sides of this

inequality yields not

9 If a circle has circumference what is the radius of the circle?

Your Response:  

Correct Response(s):  3.5, 7/2

Explanation:  

The correct answer is or . The circumference of a circle is equal to

where is the radius of the circle. Thus, Solving for gives

The answer may also be gridded as the equivalent decimal .

10 If and and are positive integers, what is one possible value of

Your Response:  

Correct Response(s):  5 or 10 or 15 or 20

Explanation:  

The possible correct answers are and . If then

Since and have no prime factors in common,

must be a factor of In addition, and, since is positive,

Therefore, the possible values of are and (The corresponding

values of are and respectively.) Any one of or may be gridded as the answer.

11

   

The figure above shows, on the left, the call buttons for the floors of an apartment building, and on the right, the buttons labeled A through I for the

页码,8/12The Official SAT Online Course

2006-11-12file://E:\新建文件夹\e6.htm

UnRe

gistered

Page 74: SAT practice test 5.pdf

严禁用于商业用途!

apartments on each floor of the building. How many different apartments can be called by pressing one button on the left and one button on the right?

Your Response:  

Correct Response(s):  108

Explanation:  

The correct answer is . There are floors in the building, and there are apartments on each of the floors. Therefore, by pressing one button on the left, for one of the floors, and then one button on the right, for one of the

apartments on that floor, different apartments can be called.

12If the function is defined by for what value of does

Your Response:  

Correct Response(s):  66

Explanation:  

The correct answer is . If then therefore,

and

13

Students at Northern High School are represented in Figure above. Figure above is another way to show which pets these students have. If the same students are represented in both figures, what is the total number of students represented by the shaded circle in Figure

页码,9/12The Official SAT Online Course

2006-11-12file://E:\新建文件夹\e6.htm

UnRe

gistered

Page 75: SAT practice test 5.pdf

严禁用于商业用途!

Your Response:  

Correct Response(s):  275

Explanation:  

The correct answer is . The shaded circle in Figure represents all students at Northern High School who have cats. This includes students who have cats only and students who have both cats and dogs. Figure shows that of the

students at Northern High School have cats only and have both cats and

dogs. Thus, the shaded circle in Figure represents of the

students at Northern High School. Since both figures represent the same

students, the shaded circle in Figure represents of students, that is,

students.

14 If is a positive integer, and what is the value of

Your Response:  

Correct Response(s):  25

Explanation:  

The correct answer is . Adding to both sides of gives the

equation dividing both sides of this equation by gives the equation

Therefore, the value of is

15 If what is the value of

Your Response:  

Correct Response(s):  2

Explanation:  

The correct answer is . Use the distributive law.

Since it follows that

Therefore, and the value of is .

16

页码,10/12The Official SAT Online Course

2006-11-12file://E:\新建文件夹\e6.htm

UnRe

gistered

Page 76: SAT practice test 5.pdf

严禁用于商业用途!

The sphere above has radius inches. What is the volume, in cubic inches, of the smallest cube that can contain the entire sphere?

Your Response:  

Correct Response(s):  1728

Explanation:  

The correct answer is .

As shown above, when the sphere is placed inside the smallest cube that can contain it, the greatest distance between two points on the sphere (the diameter of the sphere) must be equal to the shortest distance between two points on opposite faces of the cube (which is equal to the edge length of the cube). Thus, the edge length of this cube will be equal to the diameter of the sphere, which is inches. The volume of the cube, in cubic inches, will be .

17If the value of is twice the value of what is the value of

Your Response:  

Correct Response(s):  4

Explanation:  

The correct answer is . Since the value of is twice the value of the

equation is true. Cross-multiplying gives

or It follows that or

18

页码,11/12The Official SAT Online Course

2006-11-12file://E:\新建文件夹\e6.htm

UnRe

gistered

Page 77: SAT practice test 5.pdf

严禁用于商业用途!

In the -coordinate plane above, and is perpendicular to

If the coordinates of point are what is the -intercept of line

Your Response:  

Correct Response(s):  16/3, 5.33

Explanation:  

The correct answer is or . Since is perpendicular to it

follows that is a right triangle. By the Pythagorean Theorem,

The coordinates of point are

so the coordinates of are and the coordinates of

are Thus, the slope of line is Hence the equation

of line is where is the -intercept of Point is on

so Therefore, The answer can be

gridded as or as its rounded decimal equivalent, .

    

Back to Score Report  

Copyright © 2006 The College Board. All rights reserved. Privacy Policy Terms of Use Contact Us

 

页码,12/12The Official SAT Online Course

2006-11-12file://E:\新建文件夹\e6.htm

UnRe

gistered

Page 78: SAT practice test 5.pdf

严禁用于商业用途!

Help | Profile | My Organizer | My Bookmarks | Logout

Answers and Explanations

Test Sections

Section 1

Section 2

Section 3

Section 4

Section 5

Section 6

Section 8

Section 9

Section 10

Back to Score Report  

View Answers and Explanations     Online - Practice Test #5

1 Numismatics, or coin collecting, and becoming popular in the United States around 1857, when the replacement of the large cent by the new flying-eagle cent led enthusiasts to start collecting the earlier coin.

ANSWERS AND EXPLANATIONS Explanation for Correct Answer D :  Choice (D) is correct. It avoids the error of the original by providing a main verb (“became”) to perform the action of the sentence (“Numismatics … became …”).

Explanation for Incorrect Answer A :  Choice (A) involves improper coordination that results in a sentence fragment. The coordinating conjunction “and” is not needed, and the participle “becoming” cannot serve as the main verb in this sentence. The other verb, “led,” is the verb for the subordinating clause beginning with “when” and cannot serve as the main verb. A main verb is needed to complete the action of the sentence.

Explanation for Incorrect Answer B :  Choice (B) creates a sentence fragment. The participle “becoming” cannot serve as a main verb. The other verb, “led,” is the verb for the subordinating clause beginning with “when” and cannot serve as the main verb. Therefore, a main verb is needed to complete the action of the sentence.

Explanation for Incorrect Answer C :  Choice (C) involves an error in verb tense that results in faulty logic. The action of the sentence takes place in the past (“around 1857”), so a past-tense verb, “became,” not a future-tense verb, is needed.

Explanation for Incorrect Answer E :  Choice (E) involves an error in pronoun use that results in a sentence fragment. The pronoun “it” is used unnecessarily, so the subject of the sentence, “Numismatics,” has no main verb to complete the action of the sentence.

(A) and becoming

(B) becoming

(C) will become

(D) became

(E) it became

2

Just as his predecessor Mozart, for whom composing music seemed to come easily, Beethoven always struggled to perfect his work.

(A) Just as

(B) Even though

(C) Whereas

(D) Unlike with

(E) Unlike

页码,1/24The Official SAT Online Course

2006-11-12file://E:\新建文件夹\e7.htm

UnRe

gistered

Page 79: SAT practice test 5.pdf

严禁用于商业用途!

ANSWERS AND EXPLANATIONS Explanation for Correct Answer E :  Choice (E) is correct. It avoids the improper word choice of the original by replacing the adverb “Just” and the preposition “as” with the appropriate preposition “Unlike” to indicate that the idea expressed in the prepositional phrase (“Unlike…Mozart”) contrasts with the idea expressed in the independent clause (“Beethoven…work”).

Explanation for Incorrect Answer A :  Choice (A) involves inappropriate word choice that creates faulty logic. The phrase “Just as” is inappropriate in the context of this sentence because it expresses similarity, not contrast. The sentence explains how Beethoven was not like Mozart, so the preposition “Unlike” is needed to indicate that the idea expressed in the prepositional phrase (“Unlike…Mozart”) contrasts with the idea expressed in the independent clause (“Beethoven…work”).

Explanation for Incorrect Answer B :  Choice (B) involves the misuse of a subordinating conjunction. The subordinating conjunction “even though” does express contrast, but it is used inappropriately to introduce a prepositional phrase, not a subordinate clause.

Explanation for Incorrect Answer C :  Choice (C) involves the misuse of a subordinating conjunction. The subordinating conjunction “whereas” does express contrast, but it is used inappropriately to introduce a prepositional phrase, not a subordinate clause.

Explanation for Incorrect Answer D :  Choice (D) involves an unnecessary preposition. The preposition “Unlike” is appropriate to introduce a contrasting idea, but the added preposition “with” is unnecessary in the context of this sentence.

3

The Incas kept records with knotted cords called quipus, historians believe they were used to document crop production and to aid in tax collection.

ANSWERS AND EXPLANATIONS Explanation for Correct Answer D :  Choice (D) is correct. It avoids the comma-splice error and ambiguous pronoun of the original by turning the second independent clause (“historians … collection”) into a subordinate clause introduced by the relative pronoun “which,” and by making clear that the relative pronoun “which” refers to the noun “quipus.”

Explanation for Incorrect Answer A :  Choice (A) involves a comma-splice error and an ambiguous pronoun. Two independent clauses (“The … quipus” and “historians … collection”) are joined only by a comma rather than by a comma and a coordinating conjunction or by a semicolon. Also, the plural pronoun “they” could refer to “Incas,” “records,” or “quipus.”

Explanation for Incorrect Answer B :  Choice (B) contains awkward phrasing and an error in pronoun–antecedent agreement. The singular possessive pronoun “its” does not agree in number with the plural noun “quipus” to which it refers. Also, in the context of this sentence, the use of the preposition “in” after the participle “believing” is not idiomatic.

(A) historians believe they were used

(B) historians believing in its use

(C) which historians believe they use

(D) which historians believe were used

(E) and historians believe it was used

页码,2/24The Official SAT Online Course

2006-11-12file://E:\新建文件夹\e7.htm

UnRe

gistered

Page 80: SAT practice test 5.pdf

严禁用于商业用途!

Explanation for Incorrect Answer C :  Choice (C) involves an error in verb tense and an unclear pronoun reference. The present tense of the verb “use” is inconsistent with the past tense of the main verb “kept.” Also, the plural pronoun “they” could refer either to “Incas” or (illogically) to “historians.”

Explanation for Incorrect Answer E :  Choice (E) creates an error in pronoun–antecedent agreement. Although the addition of the coordinating conjunction “and” corrects the comma-splice error of the original, the singular pronoun “its” does not agree with the plural noun “quipus” to which it refers.

4 While females were not allowed to compete in the Olympic games of ancient Greece; they were not even allowed to watch them.

ANSWERS AND EXPLANATIONS Explanation for Correct Answer C :  Choice (C) is correct. It avoids the improper subordination of the original by deleting the subordinating conjunction “While” and thereby making the clause an independent clause, which can stand alone.

Explanation for Incorrect Answer A :  Choice (A) involves improper subordination. The first clause is a subordinate clause because it is introduced by a subordinate conjunction (“While”), but it cannot stand alone (as indicated by the semicolon). To make the clause an independent clause that is equal in importance to the other independent clause, the subordinating conjunction “While” should be omitted.

Explanation for Incorrect Answer B :  Choice (B) involves improper subordination. The first clause is a subordinate clause because it is introduced by a subordinate conjunction (“Even though”), but it cannot stand alone (as indicated by the semicolon). To make the clause an independent clause that is equal in importance to the sentence’s other independent clause, the subordinating conjunction “Even though” should be omitted.

Explanation for Incorrect Answer D :  Choice (D) involves a pronoun error. Although the first clause has been turned into an independent clause, equal in importance to the other independent clause of the sentence, it now has a pronoun (“They”) without an antecedent to which it refers.

Explanation for Incorrect Answer E :  Choice (E) involves awkward phrasing. The addition of the preposition “With” and the pronoun “they” is unnecessary. The noun “females” can stand on its own as the subject of the first independent clause.

(A) While females were not allowed

(B) Even though females were not allowed

(C) Females were not allowed

(D) They did not allow females

(E) With females, they were not allowed

5 In psychological theory, the term social learning is used to describe learning that is influenced by one’s environment and not the influence of innate or internal forces.

(A) and not the influence of

(B) and not, instead,

(C) rather than by

(D) rather than when it is

(E) rather than being influenced due to

页码,3/24The Official SAT Online Course

2006-11-12file://E:\新建文件夹\e7.htm

UnRe

gistered

Page 81: SAT practice test 5.pdf

严禁用于商业用途!

ANSWERS AND EXPLANATIONS Explanation for Correct Answer C :  Choice (C) is correct. It avoids the error in parallelism and faulty logic of the original by replacing the phrase “and not the influence of” with the conjunction “rather than” and the preposition “by.” This revision makes the grammatical structure that precedes the conjunction “rather than” parallel with the grammatical structure that follows it. This revision also fixes the faulty logic in the phrase “learning that is … not the influence.”

Explanation for Incorrect Answer A :  Choice (A) involves a lack of parallelism and faulty logic. The grammatical structure of the phrase after “and” is made up of an adverb (“not”) modifying “is” and a noun (“the influence”) that functions as a subject complement renaming “learning.” It does not make sense to say “learning that is not the influence.” Also, this grammatical structure is not parallel with the first structure (“influenced by one’s environment”). Replacing this phrase with the “rather than by” fixes both problems.

Explanation for Incorrect Answer B :  Choice (B) involves redundancy and a lack of parallelism. The adverbs “not” and “instead” are redundant, so “instead” is not needed. Also, for the structure following “and” to be parallel with the structure preceding “and,” the preposition “by” is needed: “is influenced by … and not by ….”

Explanation for Incorrect Answer D :  Choice (D) involves a lack of parallelism. The structure following the conjunction “rather than” is a subordinate clause introduced by the relative adverb “when.” It is not parallel with the prepositional phrase (“by one’s environment”) that precedes the conjunction. Also, the phrase “when it is” is awkward and can be replaced by the simple preposition “by.”

Explanation for Incorrect Answer E :  Choice (E) involves an error in verb tense and awkward phrasing. The progressive tense of the verb “being” is inconsistent with the simple present tense of the verb “is.” Also, the phrase “rather than being influenced due to” can be replaced by the much simpler, and more idiomatic, phrase “rather than by.”

6 Giant stars, or those that weigh 100 times what the Sun weighs, are important to galaxies and the universe because with furious combustion, it produces many important elements that form planets and other bodies, including carbon, oxygen, sodium, and neon.

ANSWERS AND EXPLANATIONS Explanation for Correct Answer B :  Choice (B) is correct. It avoids the pronoun error and imprecise language of the original by removing the pronoun “it,” which could refer to “the Sun,” “the universe,” or “combustion.”

Explanation for Incorrect Answer A :  Choice (A) involves a pronoun error and imprecise language. The antecedent of the pronoun “it” is unclear; it could be “the Sun,” “the universe,” or “combustion.”

Explanation for Incorrect Answer C :  Choice (C) involves faulty logic, imprecise language, and an unclear pronoun

(A) because with furious combustion, it produces

(B) because their furious combustion produces

(C) due to their furious combustion, producing

(D) as their furious combustion produced

(E) in that with furious combustion, they produced

页码,4/24The Official SAT Online Course

2006-11-12file://E:\新建文件夹\e7.htm

UnRe

gistered

Page 82: SAT practice test 5.pdf

严禁用于商业用途!

reference. The prepositional phrase introduced by “due to” creates faulty logic. It illogically suggests that the giant stars are important because of their combustion, not because of what their combustion produces. Also, the participial phrase introduced by “producing” describes the subject, “Giant stars,” and imprecisely suggests that the giant stars, not their combustion, produce many important elements.

Explanation for Incorrect Answer D :  Choice (D) contains an incorrect verb tense and ambiguous language. The past tense of “produced” is inconsistent with the present tense of the main verb “are” and also illogically suggests that the combustion of giant stars no longer occurs. The subordinating conjunction “as” could mean “when” or “because.” In the context of this sentence, its meaning is not clear.

Explanation for Incorrect Answer E :  Choice (E) involves imprecise language and an incorrect verb tense. The subordinate clause introduced by “in that” is imprecise, suggesting that the giant stars, not their combustion, produce many important elements. Also, the past tense of “produced” is inconsistent with the present tense of the main verb “are” and illogically suggests that the combustion of giant stars no longer occurs.

7 American photographer Annie Leibovitz is renowned for her revealing, eye-catching portraits of celebrities, and characterized by carefully staged settings and the use of vivid primary colors.

ANSWERS AND EXPLANATIONS Explanation for Correct Answer E :  Choice (E) is correct. It avoids the errors of the other options by providing a relative pronoun, “which,” to introduce the subordinate clause that describes “portraits” and by providing the plural verb “are,” which is the appropriate helping verb for the passive verb “characterized.”

Explanation for Incorrect Answer A :  Choice (A) involves an incorrectly used coordinating conjunction and faulty logic. The coordinating conjunction “and” that precedes the verb “characterized” forms a compound verb for the subject of the independent clause, “Annie Leibovitz.” It illogically suggests that Annie Leibovitz, not her portraits, is “characterized by carefully staged settings and the use of vivid primary colors.”

Explanation for Incorrect Answer B :  Choice (B) involves faulty logic. The addition of the pronoun “she,” which refers to the subject of the sentence, “Annie Leibovitz,” and the addition of the singular helping verb “is” create a second independent clause. This new clause illogically suggests that Annie Leibovitz, not her portraits, is “characterized by carefully staged settings and the use of vivid primary colors.”

Explanation for Incorrect Answer C :  Choice (C) creates a comma-splice error and a subject–verb agreement error. The addition of a subject (“each”) and a helping verb (“are”) creates an independent clause connected to the other independent clause (“American … celebrities”) by only a comma. Also, the plural verb “are” does not agree in number with its subject, the singular pronoun “each.”

Explanation for Incorrect Answer D :  Choice (D) involves an error in subject–verb agreement. Although the addition of the phrase “each of which are” creates a subordinate clause that modifies “portraits,” the plural verb “are” does not agree in number with its subject, the singular pronoun “each.”

(A) and

(B) and she is

(C) each of them are

(D) each of which are

(E) which are

页码,5/24The Official SAT Online Course

2006-11-12file://E:\新建文件夹\e7.htm

UnRe

gistered

Page 83: SAT practice test 5.pdf

严禁用于商业用途!

8 Decorative arts, art forms that have a mainly practical or ornamental purpose, and include ceramics, basketry, furniture making, and glassblowing.

ANSWERS AND EXPLANATIONS Explanation for Correct Answer D :  Choice (D) is correct. It avoids the errors of the original by removing the unnecessary coordinating conjunction “and,” so that the sentence has a main plural verb, “include,” that agrees with the plural subject, “Decorative arts.”

Explanation for Incorrect Answer A :  Choice (A) involves an unnecessary coordinating conjunction, “and.” The conjunction “and” connects the verbs “have” and “include” as compound verbs of the subordinate clause, so the subject of the main clause has no verb to complete the action of the sentence. To make “include” the verb of the independent clause, the “and” should be omitted.

Explanation for Incorrect Answer B :  Choice (B) results in a sentence fragment. The participle “including” introduces a participial phrase that functions as an adjective describing “arts,” so the sentence has no main verb. To fix this problem, the verb “include” is needed.

Explanation for Incorrect Answer C :  Choice (C) involves an error in subject–verb agreement. The singular verb “includes” does not agree with the plural noun and subject of the sentence, “arts.”

Explanation for Incorrect Answer E :  Choice (E) creates a sentence fragment. The relative pronoun “which” makes all that follows it a subordinate clause that describes “arts,” so the sentence has no main verb. Without the relative pronoun, the verb “include” can serve as the main verb of the sentence.

(A) and include

(B) including

(C) includes

(D) include

(E) which include

9 About 35 percent of the world's orange juice is produced by Florida, compared with Brazil, the world’s largest orange producer, which has nearly 50 percent.

ANSWERS AND EXPLANATIONS Explanation for Correct Answer E :  Choice (E) is correct. It avoids the illogical comparison of the original by replacing “Brazil” with “50 percent” as the object of the multi-word preposition “compared with.”

Explanation for Incorrect Answer A :  Choice (A) involves an illogical comparison. The prepositional phrase introduced by the multi-word preposition “compared with” functions as an adjective modifying

(A) Brazil, the world’s largest orange producer, which has nearly 50 percent

(B) Brazil, with nearly 50 percent, it is the world’s largest orange producer

(C) nearly 50 percent that are produced by Brazil as the world’s largest orange producer

(D) the production of Brazil, the world’s largest orange producer, with nearly 50 percent

(E) nearly 50 percent produced by Brazil, the world’s largest orange producer

页码,6/24The Official SAT Online Course

2006-11-12file://E:\新建文件夹\e7.htm

UnRe

gistered

Page 84: SAT practice test 5.pdf

严禁用于商业用途!

the subject of the sentence, “35 percent,” so the amount, 35 percent, is illogically compared with the country, “Brazil,” rather than with another amount, “50 percent.”

Explanation for Incorrect Answer B :  Choice (B) involves an illogical comparison and a comma-splice error. The prepositional phrase introduced by the multi-word preposition “compared with” functions as an adjective modifying the subject of the sentence, “35 percent,” so the amount, “35 percent,” is illogically compared with the country, “Brazil,” rather than with another amount, “50 percent.” Also, two independent clauses (“About … percent” and “it … producer”) are joined only by a comma rather than by a semicolon or by a comma and a subordinating conjunction.

Explanation for Incorrect Answer C :  Choice (C) contains an error in subject–verb agreement and awkward phrasing. The relative pronoun “that” refers to the singular amount noun “50 percent,” so it requires the singular verb “is.” Also, the long phrase beginning with “nearly,” which includes an unnecessary subordinate clause (“that are produced by Brazil”) and preposition (“as”), can be reduced to the simpler phrase “nearly 50 percent produced by Brazil, the world’s largest orange producer.”

Explanation for Incorrect Answer D :  Choice (D) contains a non-idiomatic preposition and awkward phrasing. In the context of this sentence, the prepositional phrase “production of Brazil” is not idiomatic and creates nonsense. Brazil is not being produced. The idiomatic preposition in this case is “by” (“production by Brazil”). Also, the long phrase beginning with “nearly” can be reduced to the simpler phrase “nearly 50 percent produced by Brazil, the world’s largest orange producer.”

10Many of blues great Bessie Smith’s songs describe the experiences of southern Black migrants, especially the struggles of Black women to adjust to urban life in the northern United States.

ANSWERS AND EXPLANATIONS Explanation for Correct Answer A :  Choice (A) is correct. It avoids the errors of the other options by providing an appositive, “the struggles of Black women to adjust,” to rename the “experiences” that Bessie Smith sang about.

Explanation for Incorrect Answer B :  Choice (B) involves awkward, unidiomatic phrasing. The infinitive “to adjust” is the idiomatic verb form to follow “struggled” in this context. In addition, the entire awkward phrase “how Black women struggled at adjusting” can be reduced to the more precise “the struggles of Black women to adjust.”

Explanation for Incorrect Answer C :  Choice (C) involves unnecessary pronoun use and faulty logic. It does not make sense to use the relative adverb “when” to describe “experiences.” In addition, there is nothing in the sentence to which the pronoun “it” can logically refer.

Explanation for Incorrect Answer D :  Choice (D) involves faulty logic. What comes after “especially” should be an appositive that renames or describes “the experiences” that Bessie Smith sang about, but people (“Black women”) cannot logically be “experiences.”

Explanation for Incorrect Answer E :  Choice (E) involves awkward and wordy phrasing, resulting in nonsense. What comes after “especially” should be an appositive that renames or describes “the experiences” that Bessie Smith sang about, but the prepositional phrase “for

(A) the struggles of Black women to adjust

(B) how Black women struggled at adjusting

(C) when it was Black women struggling to adjust

(D) Black women, they struggled with adjusting

(E) for Black women and struggling to adjust

页码,7/24The Official SAT Online Course

2006-11-12file://E:\新建文件夹\e7.htm

UnRe

gistered

Page 85: SAT practice test 5.pdf

严禁用于商业用途!

Black women” cannot logically be “experiences.” In addition, the entire awkward phrase “for Black women and struggling to adjust” can be reduced to the more precise phrase “struggles of Black women to adjust.”

11 The Bretons who began emigrating to the United States from the Brittany region of France in the early twentieth century were distinguished from other French citizens by their Celtic origin, but about 40 percent of those who emigrated spoke a Celtic language closely related to Welsh.

ANSWERS AND EXPLANATIONS Explanation for Correct Answer D :  Choice (D) is correct. It avoids the coordination error of the original by deleting the coordinating conjunction “but” and turning the independent clause (“but … Welsh”) into a prepositional phrase that functions as an adverb modifying the verb “were distinguished.” The coordinating conjunction “but” indicates contrast, but the idea expressed in the clause following “but” does not contrast with the idea in the clause preceding “but.” Instead, the clause following “but” explains how 40 percent of the French immigrants were distinguished from the other French immigrants.

Explanation for Incorrect Answer A :  Choice (A) involves a coordination error. The coordinating conjunction “but” indicates contrast, but the idea expressed in the clause following “but” does not contrast with the idea in the clause preceding “but.” Instead, the clause following “but” explains how 40 percent of the French immigrants were distinguished from the other French immigrants.

Explanation for Incorrect Answer B :  Choice (B) involves an unclear pronoun. The relative pronoun “whom” has no clear antecedent; it could refer to “Bretons” or to “French citizens.”

Explanation for Incorrect Answer C :  Choice (C) involves errors in pronoun reference. The relative pronoun “which” has no clear antecedent; it could refer to “Bretons” or to “French citizens.” In addition, “which” should be used to refer to things, not to people.

Explanation for Incorrect Answer E :  Choice (E) involves an inappropriate relative adverb and a verb-tense error. The relative adverb “where” introduces a subordinate clause that functions as an adjective, but the noun to which it refers not clear. It could be “United States,” “Brittany region,” or “France.”

(A) but about 40 percent of those who emigrated spoke

(B) about 40 percent of whom spoke

(C) about 40 percent of which were speaking

(D) with about 40 percent of those who emigrated speaking

(E) where 40 percent of them were speaking

12

Although fast runners, wolves a loping run

miles, running the night if necessary.  

ANSWERS AND EXPLANATIONS Corrected Sentence: Although they are not fast runners, wolves can maintain a loping run for many miles, running throughout the night if necessary.

Explanation for Correct Answer B : 

they are not could have maintained

for many throughout No error

页码,8/24The Official SAT Online Course

2006-11-12file://E:\新建文件夹\e7.htm

UnRe

gistered

Page 86: SAT practice test 5.pdf

严禁用于商业用途!

The error in this sentence occurs at (B), where an incorrect verb tense is used. The conditional present perfect tense of “could have maintained” is not consistent with the simple present tense of the subordinate clause (“are”).

Explanation for Incorrect Answer A :  There is no error at (A). The plural pronoun “they” agrees in number with the plural noun to which it refers, “wolves”; the plural verb “are” agrees with the plural subject of the subordinate clause, “they”; and the adverb “not” appropriately modifies the verb “are.”

Explanation for Incorrect Answer C :  There is no error at (C). The preposition “for” is an appropriate preposition to introduce the prepositional phrase (“for many miles”) that functions as an adjective describing the noun “run.” The adjective “many” appropriately modifies the noun “miles.”

Explanation for Incorrect Answer D :  There is no error at (D). The preposition “throughout” properly introduces the prepositional phrase “throughout the night,” which functions as an adverb modifying the participle “running.”

Explanation for Incorrect Answer E :  There is an error in the sentence.

13

The credit for making Franz Kafka famous as a writer to his

friend, novelist Max Brod, Kafka’s unpublished manuscripts had

them published, despite Kafka’s dying wishes to the contrary.  

ANSWERS AND EXPLANATIONS Corrected Sentence: The credit for making Franz Kafka internationally famous belongs to his friend, novelist Max Brod, who edited Kafka’s unpublished manuscripts and then had them published, despite Kafka’s dying wishes to the contrary.

Explanation for Correct Answer B :  The error in this sentence occurs at (B), where a plural verb (“belong”) does not agree in number with its singular subject (“credit”).

Explanation for Incorrect Answer A :  There is no error at (A). The adverb “internationally” appropriately modifies the adjective “famous.”

Explanation for Incorrect Answer C :  There is no error at (C). The relative pronoun “who” properly refers to the proper noun “Max Brod,” a person, and properly introduces the subordinate clause (“who … published”) that describes that proper noun.

Explanation for Incorrect Answer D :  There is no error at (D). The coordinating conjunction “and” properly connects the two verb phrases (“edited … manuscripts” and “then … published”) that complete the action of the subordinate clause introduced by the relative pronoun “who.” The adverb “then” appropriately modifies the verb “had.”

Explanation for Incorrect Answer E :  There is an error in the sentence.

internationally belong

who edited and then

No error

页码,9/24The Official SAT Online Course

2006-11-12file://E:\新建文件夹\e7.htm

UnRe

gistered

Page 87: SAT practice test 5.pdf

严禁用于商业用途!

14

country-and-western bands, often feature the harmonica, banjo, or

fiddle, rock bands   equipment, including amplifiers, guitars,

and organs.  

ANSWERS AND EXPLANATIONS Corrected Sentence: Unlike country-and-western bands, which often feature the harmonica, banjo, or fiddle, rock bands tend to use electronic equipment, including amplifiers, guitars, and organs.

Explanation for Correct Answer C :  The error in this sentence occurs at (C), where an incorrect verb tense is used. The past tense of “tended” is not consistent with the simple present tense of the subordinate clause, “feature.” The simple present tense (“tend”) is needed instead.

Explanation for Incorrect Answer A :  There is no error at (A). The preposition “unlike” is an appropriate preposition to introduce the prepositional phrase (“Unlike … bands”) that modifies the subject of the independent clause (“rock bands”) and to signal that a contrasting idea will follow.

Explanation for Incorrect Answer B :  There is no error at (B). The relative pronoun “which” properly refers to the noun “bands” and properly introduces the subordinate clause (“which … fiddle”) that describes “bands.”

Explanation for Incorrect Answer D :  There is no error at (D). The adjective “electronic” appropriately modifies the noun “equipment.”

Explanation for Incorrect Answer E :  There is an error in the sentence.

15

her nonfiction, Jean Craighead George’s fiction not only from

published material but also firsthand observations of animals ecological

systems.  

ANSWERS AND EXPLANATIONS Corrected Sentence: Like her nonfiction, Jean Craighead George’s fiction draws extensively not only from published material but also from firsthand observations of animals and ecological systems.

Explanation for Correct Answer C :  The error in this sentence occurs at (C), where there is an error in parallel structure. The correlative conjunctions “not only … but also” require that the grammatical structure following the first half the pair is the same as the grammatical structure following the second half. The independent clause “she had firsthand observations” is not grammatically parallel to the prepositional phrase, “from published material,” that follows “not only.” The preposition “from” is needed instead (“but also from…”)

Unlike which

tended to use electronic

No error

Like draws extensively

she had and

No error

页码,10/24The Official SAT Online Course

2006-11-12file://E:\新建文件夹\e7.htm

UnRe

gistered

Page 88: SAT practice test 5.pdf

严禁用于商业用途!

Explanation for Incorrect Answer A :  There is no error at (A). The preposition “like” is an appropriate preposition to introduce the prepositional phrase (“like … nonfiction”) that modifies the subject of the independent clause, “fiction.”

Explanation for Incorrect Answer B :  There is no error at (B). The singular verb “draws” agrees in number with its singular subject “fiction,” and the adverb “extensively” appropriately modifies the verb “draws.”

Explanation for Incorrect Answer D :  There is no error at (D). The coordinating conjunction “and” appropriately connects the two objects of the preposition “of,” “animals” and “systems.”

Explanation for Incorrect Answer E :  There is an error in the sentence.

16

ships move the Suez Canal under their own power, extremely large

ships must be a tugboat.  

ANSWERS AND EXPLANATIONS Corrected Sentence: Most ships move through the Suez Canal under their own power, but extremely large ships must be assisted by a tugboat.

Explanation for Correct Answer C :  The error in this sentence occurs at (C), where an inappropriate coordinating conjunction, “so,” connects two independent clauses that express contrasting ideas. The coordinating conjunction “but” is needed to properly express the contrast between the ideas in the sentence.

Explanation for Incorrect Answer A :  There is no error at (A). The word “Most” appropriately functions as an adjective modifying the noun “ships.”

Explanation for Incorrect Answer B :  There is no error at (B). The preposition “through” is an idiomatic preposition to follow the verb “move” and properly introduces the prepositional phrase (“through the Suez Canal”) that functions as an adverb modifying the verb “move.”

Explanation for Incorrect Answer D :  There is no error at (D). The past participle “assisted” is the appropriate verb form to create the present tense of a passive verb, “be assisted.” The preposition “by” is the idiomatic preposition to follow a passive verb and to introduce the prepositional phrase that modifies that verb.

Explanation for Incorrect Answer E :  There is an error in the sentence.

17

Ralph Ellison’s Invisible Man  the story of a nameless young Black man who

to forge his own identity   assigned to

Most through so

assisted by No error

is

ultimately decides rather than accept the one

页码,11/24The Official SAT Online Course

2006-11-12file://E:\新建文件夹\e7.htm

UnRe

gistered

Page 89: SAT practice test 5.pdf

严禁用于商业用途!

him.  

ANSWERS AND EXPLANATIONS Corrected Sentence: 

Explanation for Correct Answer E :  There is no error in this sentence.

Explanation for Incorrect Answer A :  There is no error at (A). The singular verb “is” agrees in number with its singular subject, “Ralph Ellison’s Invisible Man.”

Explanation for Incorrect Answer B :  There is no error at (B). The adverb “ultimately” correctly modifies the verb “decides,” and the singular “decides” agrees in number with the singular subject of the subordinate clause, “man.”

Explanation for Incorrect Answer C :  There is no error at (C). The phrase “rather than” is an idiomatic phrase that serves as a conjunction meaning “and not.” It correctly connects the two phrases “forge his own identity” and “accept the one assigned to him.”

Explanation for Incorrect Answer D :  There is no error at (D). The definite article “the” properly modifies the pronoun “one,” which, in turn, properly serves as the direct object of “accept.”

18

The Sun for nearly five billion years and to have

thermonuclear fuel to shine for about another five billion.  

ANSWERS AND EXPLANATIONS Corrected Sentence: The Sun has been shining for nearly five billion years and is thought to have sufficient thermonuclear fuel in its core to shine for about another five billion.

Explanation for Correct Answer A :  The error in this sentence occurs at (A), where an incorrect verb tense is used. The past progressive tense of “was shining” is incorrect for expressing an action that started in the past and is still occurring. The present perfect progressive, “has been shining,” is needed instead.

Explanation for Incorrect Answer B :  There is no error at (B). The singular verb “is” agrees in number with the singular subject of the sentence, “Sun,” and the past participle “thought” is the appropriate verb form to create the present tense of a passive verb, “is thought.”

Explanation for Incorrect Answer C :  There is no error at (C). The adjective “sufficient” appropriately describes the noun “fuel.”

Explanation for Incorrect Answer D :  There is no error at (D). The preposition “in” properly introduces the prepositional phrase “in its core,” which functions as an adjective describing the noun “fuel”; the singular possessive pronoun “its” agrees with the singular

No error

was shining is thought sufficient

in its core No error

页码,12/24The Official SAT Online Course

2006-11-12file://E:\新建文件夹\e7.htm

UnRe

gistered

Page 90: SAT practice test 5.pdf

严禁用于商业用途!

noun, “Sun,” to which it refers; and the noun “core” properly serves as the object of the preposition “in.”

Explanation for Incorrect Answer E :  There is an error in the sentence.

19

The study that children small amounts of added sugars

on average one serving more of grains per day children who ate larger

amounts of these same sugars.  

ANSWERS AND EXPLANATIONS Corrected Sentence: The study showed that children who consumed small amounts of added sugars ate on average one serving more of grains per day than did children who ate larger amounts of these same sugars.

Explanation for Correct Answer C :  The error in this sentence occurs at (C), where an improper verb form is used. The participle “eating” cannot serve as a main verb for the subject of this subordinate clause, “children,” and its present tense is inconsistent with the simple past tense of the other verbs in the sentence (“showed,” “consumed,” “did,” and “ate”). The past-tense verb “ate” should be used instead.

Explanation for Incorrect Answer A :  There is no error at (A). The past tense of the verb “showed” appropriately expresses an action completed in the past.

Explanation for Incorrect Answer B :  There is no error at (B). The relative pronoun “who” appropriately refers to the noun “children” and properly introduces the subordinate clause (“who consumed small amounts of added sugars”) that describes “children.”

Explanation for Incorrect Answer D :  There is no error at (D). The conjunction “than” is necessary to complete the comparison begun with “more.” Also, the past tense of “did” is consistent with the past tense of the other verbs in the sentence (“showed,” “consumed,” and “ate”).

Explanation for Incorrect Answer E :  There is an error in the sentence.

20

In many parts of the world grasses cover vast expanses of land, periodic,

controlled burning in order to keep woody brush a foothold

and continuing grass growth.  

ANSWERS AND EXPLANATIONS Corrected Sentence: In many parts of the world where grasses cover vast expanses of land, periodic, controlled burning is practiced in order to keep woody brush from gaining a foothold and to stimulate continuing grass growth.

showed who consumed eating

than did

No error

where

is practiced from gaining

it stimulates No error

页码,13/24The Official SAT Online Course

2006-11-12file://E:\新建文件夹\e7.htm

UnRe

gistered

Page 91: SAT practice test 5.pdf

严禁用于商业用途!

Explanation for Correct Answer D :  The error in this sentence occurs at (D), where there is an error in parallelism. The independent clause “it stimulates … growth” is not parallel with the infinitive phrase “to keep” to which it is connected by the coordinating conjunction “and.” The infinitive phrase “to stimulate” is needed instead.

Explanation for Incorrect Answer A :  There is no error at (A). The relative adverb “where” correctly introduces the subordinate clause that modifies the noun “parts.”

Explanation for Incorrect Answer B :  There is no error at (B). The passive verb “is practiced” is appropriately used in this sentence, and the singular “is” agrees in number with its singular subject, “burning.”

Explanation for Incorrect Answer C :  There is no error at (C). The preposition “from” is an idiomatic preposition to follow “keep,” and the gerund “gaining,” which is a noun, properly serves as the object of that preposition. In addition, the prepositional phrase appropriately functions as an adverb modifying “keep.”

Explanation for Incorrect Answer E :  There is an error in the sentence.

21

The newspaper   the increase in the minimum wage,

are still having trouble ends meet.  

ANSWERS AND EXPLANATIONS Corrected Sentence: The newspaper reported that even with the increase in the minimum wage, many people are still having trouble making ends meet.

Explanation for Correct Answer B :  The error in this sentence occurs at (B), where improper phrasing is used. The participle “having” introduces a participial phrase that incorrectly modifies the subject of the sentence, “newspaper.” So that the phrase “the increase in the minimum wage” describes the intended noun, “people,” the phrase must be introduced by a preposition, in this case, “with.”

Explanation for Incorrect Answer A :  There is no error at (A). The past tense of the verb “reported” appropriately expresses an action that was completed in the past, and the pronoun “that” properly introduces the subordinate clause “that … meet,” which functions as the direct object of the independent clause.

Explanation for Incorrect Answer C :  There is no error at (C). The adjective “many” appropriately modifies the noun “people,” which, in turn, properly functions as the subject of the subordinate clause introduced by “that.”

Explanation for Incorrect Answer D :  There is no error at (D). The participle “making” properly introduces the participial phrase “making ends meet” that functions as an adjective describing the noun “trouble.”

Explanation for Incorrect Answer E :  There is an error in the sentence.

22

reported that having many people

making No error

页码,14/24The Official SAT Online Course

2006-11-12file://E:\新建文件夹\e7.htm

UnRe

gistered

Page 92: SAT practice test 5.pdf

严禁用于商业用途!

in the United States and Canada, American Sign Language,

comprises hand signs, facial expressions, and fingerspelling,   regional

differences and slang.  

ANSWERS AND EXPLANATIONS Corrected Sentence: Used primarily in the United States and Canada, American Sign Language, which comprises hand signs, facial expressions, and fingerspelling, includes many regional differences and slang.

Explanation for Correct Answer C :  The error in this sentence occurs at (C), where a participle, “including,” is used instead of a main verb, “includes,” which is needed to complete the action of the sentence.

Explanation for Incorrect Answer A :  There is no error at (A). The past participle “used” appropriately introduces the participial phrase that modifies the subject of the independent clause, “American Sign Language.” The adverb “primarily” properly modifies the participle “used,” which functions as an adjective.

Explanation for Incorrect Answer B :  There is no error at (B). The relative pronoun “which” appropriately refers to the proper noun “American Sign Language” and introduces the subordinate clause that describes “American Sign Language.”

Explanation for Incorrect Answer D :  There is no error at (D). The plural adjective “many” idiomatically describes the plural noun “differences.”

Explanation for Incorrect Answer E :  There is an error in the sentence.

23

The United States is the world’s largest cranberries and cranberry

products, of which there Canada.  

ANSWERS AND EXPLANATIONS Corrected Sentence: The United States is the world’s largest producer of cranberries and cranberry products, most of which are consumed there and in Canada.

Explanation for Correct Answer C :  The error in this sentence occurs at (C), where an incorrect verb is used. The singular helping verb “is” does not agree in number with the plural subject of the subordinate clause (“most”) that refers to the compound plural objects of the preposition, “cranberries and cranberry products.”

Explanation for Incorrect Answer A :  There is no error at (A). The singular noun “producer” appropriately renames the singular subject of the independent clause, “United States.” The preposition

Used primarily which

including many

No error

producer of

most is consumed and in No error

页码,15/24The Official SAT Online Course

2006-11-12file://E:\新建文件夹\e7.htm

UnRe

gistered

Page 93: SAT practice test 5.pdf

严禁用于商业用途!

“of” is an idiomatic preposition to follow “producer” and to introduce the prepositional phrase (“of … products”) that functions as an adjective describing “producer.”

Explanation for Incorrect Answer B :  There is no error at (B). The pronoun “most,” which can be singular or plural, agrees in number with the compound plural objects of the preposition, “cranberries and cranberry products,” to which it refers.

Explanation for Incorrect Answer D :  There is no error at (D). The coordinating conjunction “and” appropriately connects two adverbs, “there,” and the prepositional phrase, “in Canada,” that modify the verb “consumed.” The preposition “in” is an idiomatic preposition to follow the verb “consumed.”

Explanation for Incorrect Answer E :  There is an error in the sentence.

24

Almost all animals prepare for during summer large

amounts of food, to thick layers of fat.  

ANSWERS AND EXPLANATIONS Corrected Sentence: Almost all animals that hibernate prepare for hibernation during summer by eating large amounts of food, which they convert to thick layers of fat.

Explanation for Correct Answer B :  The error in this sentence occurs at (B), where an improper pronoun is used. The pronoun “it” has no antecedent to which it can logically refer.

Explanation for Incorrect Answer A :  There is no error at (A). The relative pronoun “that” correctly refers to “animals” and properly introduces the subordinate clause that describes “animals.” The plural verb of this subordinate clause, “hibernate,” agrees in number with the pronoun “that,” which refers to the plural noun “animals.”

Explanation for Incorrect Answer C :  There is no error at (C). The preposition “by” is the idiomatic preposition to follow the word “prepare,” and the gerund “eating” properly serves as the object of that preposition. The prepositional phrase appropriately serves as an adverb modifying the verb “prepare.”

Explanation for Incorrect Answer D :  There is no error at (D). The relative pronoun “which” introduces the clause (“which they convert”) that modifies the noun “food.” The plural pronoun “they” agrees in number with the plural noun to which it refers, “animals,” and the plural verb “convert” agrees in number with its plural subject, “they.”

Explanation for Incorrect Answer E :  There is an error in the sentence.

25

Among birds in tree trunks, nuthatches are the only ones that

with their heads facing downward, food in bark

crevices their upward search.  

that hibernate it by eating

which they convert No error

that forage

regularly feed when they often find

overlooked in No error

页码,16/24The Official SAT Online Course

2006-11-12file://E:\新建文件夹\e7.htm

UnRe

gistered

Page 94: SAT practice test 5.pdf

严禁用于商业用途!

ANSWERS AND EXPLANATIONS Corrected Sentence: Among birds that forage in tree trunks, nuthatches are the only ones that regularly feed with their heads facing downward, often finding food in bark crevices overlooked in their upward search.

Explanation for Correct Answer C :  The error in this sentence occurs at (C), where a subordinate clause creates faulty logic. The use of “when,” an adverb that modifies the verb “feed,” makes the sentence illogically suggest that nuthatches regularly feed “when they often find food.”

Explanation for Incorrect Answer A :  There is no error at (A). The relative pronoun “that” correctly refers to “birds” and properly introduces the subordinate clause that describes “birds.” The plural verb of this subordinate clause, “forage,” agrees in number with the pronoun “that,” which refers to the plural noun “birds.”

Explanation for Incorrect Answer B :  There is no error at (B). The adverb “regularly” properly modifies the plural verb “feed,” and the plural verb agrees in number with the relative pronoun “that,” which refers to the plural noun “ones.”

Explanation for Incorrect Answer D :  There is no error at (D). The past participle “overlooked” properly functions as an adjective describing the noun “food,” and the preposition “in” appropriately introduces the prepositional phrase (“in … search”) that serves as an adverb modifying the participle “overlooked.”

Explanation for Incorrect Answer E :  There is an error in the sentence.

26

If you the necessary calories gasoline instead of by eating

food, would be able 26 miles on about one-twelfth of a gallon of gas.

 

ANSWERS AND EXPLANATIONS Corrected Sentence: If you could acquire the necessary calories by drinking gasoline instead of by eating food, you would be able to run 26 miles on about one-twelfth of a gallon of gas.

Explanation for Correct Answer A :  The error in this sentence occurs at (A), where an incorrect verb tense is used. The present tense of the verb “can acquire” in the subordinate clause is inconsistent with the past tense of the verb in the independent clause, “would be able.”

Explanation for Incorrect Answer B :  There is no error at (B). The preposition “by” is the idiomatic preposition to follow “acquire,” and the gerund “drinking,” which is a noun, appropriately serves as the object of that preposition. The prepositional phrase appropriately functions as an adverb modifying the verb “acquire.”

Explanation for Incorrect Answer C :  There is no error at (C). The second-person pronoun “you” properly serves as

can acquire by drinking

you to run

No error

页码,17/24The Official SAT Online Course

2006-11-12file://E:\新建文件夹\e7.htm

UnRe

gistered

Page 95: SAT practice test 5.pdf

严禁用于商业用途!

the subject of the independent clause and is consistent with the second-person pronoun “you” in the introductory subordinate clause (“If … food”).

Explanation for Incorrect Answer D :  There is no error at (D). The infinitive “to run” is the idiomatic verb form to follow the verb “able.”

Explanation for Incorrect Answer E :  There is an error in the sentence.

27

In many large cities in the United States, the presence of a diverse

population to repeated calls curricula taught in languages

other than English.  

ANSWERS AND EXPLANATIONS Corrected Sentence: In many large cities in the United States, the presence of a culturally diverse population has led to repeated calls for curricula taught wholly or partly in languages other than English.

Explanation for Correct Answer C :  The error in this sentence occurs at (C), where the relative pronoun (“that”) is not idiomatic and is incorrectly used to introduce a prepositional phrase. The preposition “for” is needed instead (“calls for curricula…”).

Explanation for Incorrect Answer A :  There is no error at (A). The adverb “culturally” appropriately modifies the adjective “diverse,” which, in turn, describes the noun “population.”

Explanation for Incorrect Answer B :  There is no error at (B). The singular helping verb “has” agrees in number with the subject of the independent clause, “presence,” and the perfect tense (“has led”) is appropriate for expressing an action completed in the present time.

Explanation for Incorrect Answer D :  There is no error at (D). The compound adverb phrase “wholly or partly” appropriately modifies the past participle “taught,” which functions as an adjective describing the noun “curricula.”

Explanation for Incorrect Answer E :  There is an error in the sentence.

28

, the game of checkers in Egypt of the

pharaohs and in the writings of Homer and Plato.  

ANSWERS AND EXPLANATIONS Corrected Sentence: 

Explanation for Correct Answer E :  There is no error in this sentence.

culturally

has led that wholly or partly

No error

Of ancient origin was played during the time

is mentioned No error

页码,18/24The Official SAT Online Course

2006-11-12file://E:\新建文件夹\e7.htm

UnRe

gistered

Page 96: SAT practice test 5.pdf

严禁用于商业用途!

Explanation for Incorrect Answer A :  There is no error at (A). The preposition “of” is used properly to introduce the prepositional phrase, “Of ancient origin,” that modifies the subject of the sentence, “game.”

Explanation for Incorrect Answer B :  There is no error at (B). The singular helping verb “was” agrees in number with the singular subject of the sentence, “game,” and the past participle “played” is the appropriate verb form to create the present tense of a passive verb (“was played”).

Explanation for Incorrect Answer C :  There is no error at (C). The preposition “during” combines with “the time” to produce an idiomatic phrase that indicates when checkers was played in Egypt.

Explanation for Incorrect Answer D :  There is no error at (D). The singular helping verb “is” agrees in number with the singular subject “game,” and the past participle “mentioned” is the appropriate verb form to create the present tense of a passive verb (“is mentioned”).

29

the early 1920's, Louis Armstrong recorded solos as a member of King

Oliver’s Creole Jazz Band in “Chimes Blues” and “Tears,”

he composed with pianist Lil Hardin.  

ANSWERS AND EXPLANATIONS Corrected Sentence: In the early 1920’s, Louis Armstrong recorded his first solos as a member of King Oliver’s Creole Jazz Band in such pieces as “Chimes Blues” and “Tears,” which he composed with pianist Lil Hardin.

Explanation for Correct Answer A :  The error in this sentence occurs at (A), where an inappropriate preposition (“By”) is used. In the context of this sentence, the preposition “In” is the appropriate preposition to introduce the prepositional phrase that functions as an adverb modifying the verb “recorded.”

Explanation for Incorrect Answer B :  There is no error at (B). The singular possessive pronoun “his” agrees in number and gender with the singular noun “Louis Armstrong” to which it refers, and it appropriately combines with the adjective “first” to describe the plural noun “solos.”

Explanation for Incorrect Answer C :  There is no error at (C). The adjective “such” appropriately modifies the noun “pieces,” which properly functions as the object of the preposition “in,” and the preposition “as” appropriately introduces the prepositional phrase that functions as an adjective modifying the noun “pieces.”

Explanation for Incorrect Answer D :  There is no error at (D). The relative pronoun “which” is used appropriately to refer to the pieces that Armstrong composed with pianist Lil Hardin.

Explanation for Incorrect Answer E :  There is an error in the sentence.

(1) The early history of astronomy was full of misunderstandings. (2) Some of

By his first

such pieces as which

No error

页码,19/24The Official SAT Online Course

2006-11-12file://E:\新建文件夹\e7.htm

UnRe

gistered

Page 97: SAT practice test 5.pdf

严禁用于商业用途!

them were funny, it's like the controversy of the "canali" on Mars. (3) In the late 1800’s an Italian astronomer named Giovanni Schiaparelli studied Mars. (4) He had a high-powered telescope that he used to look at Mars. (5) Schiaparelli thought he saw channels criss-crossing the planet's surface. (6) He was intrigued: perhaps these channels were evidence that Mars had great flowing rivers like the Earth. (7) Schiaparelli made charts of the surface of Mars and labeled it with the Italian word "canali." 

(8) Unfortunately, "canali" can be translated into English as either "channels" or "canals." (9) Channels and canals are two different things because channels are formed naturally by water, while canals are constructed by people. (10) Some people translated "canali" as "canals," word began to spread that the lines Schiaparelli saw through his telescope were actually canals that had been built by intelligent beings. (11) One of them was an amateur astronomer named Percival Lowell. (12) He wrote a series of best-selling books. (13) In these books Lowell publicized the notion that these "canals" were built by Martian farmers who understood irrigation. 

(14) In 1965 a U.S. spacecraft flying close to the surface of Mars sent back conclusive pictures. (15) There are no prominent channels anywhere on the planet. (16) Lowell and Schiaparelli saw what they wanted to see. (17) Lowell was wrong, of course, but so was Schiaparelli. 

30 Which is the best version of the underlined portion of sentence 2 (reproduced below)? Some of them were funny, it's like the controversy of the "canali" on Mars.

ANSWERS AND EXPLANATIONS Explanation for Correct Answer C :  Choice (C) is correct. The sentence properly introduces the controversy that is the subject of the passage as one of a number of funny misunderstandings.

Explanation for Incorrect Answer A :  Choice (A) is unsatisfactory because it joins two independent thoughts with only a comma.

Explanation for Incorrect Answer B :  Choice (B) is unsatisfactory because it is illogical; "it's" does not make sense in this context.

Explanation for Incorrect Answer D :  Choice (D) is unsatisfactory because "even" suggests that others do not find the "canali" controversy funny. The passage does not indicate this.

Explanation for Incorrect Answer E :  Choice (E) is unsatisfactory because "as funny as" puts the emphasis of the sentence on the other misunderstandings, with the expectation that they will be compared to the "canali" controversy. Such a comparison does not happen in the passage.

(A) (as it is now)

(B) funny; it's like

(C) funny, like

(D) funny, even

(E) as funny as

31Which is the best way to combine sentences 3 and 4 (reproduced below)? In the late 1800’s an Italian astronomer named Giovanni Schiaparelli studied Mars. He had a high-powered telescope that he used to look at Mars.

In the late 1800’s an Italian astronomer named Giovanni Schiaparelli

页码,20/24The Official SAT Online Course

2006-11-12file://E:\新建文件夹\e7.htm

UnRe

gistered

Page 98: SAT practice test 5.pdf

严禁用于商业用途!

ANSWERS AND EXPLANATIONS Explanation for Correct Answer D :  Choice (D) is correct. The sentence nicely joins the ideas of sentences 3 and 4 without repeating information unnecessarily.

Explanation for Incorrect Answer A :  Choice (A) is unsatisfactory because it is unidiomatic to write that Schiaparelli "studied Mars by" a telescope; "with" or "through" would be more appropriate.

Explanation for Incorrect Answer B :  Choice (B) is unsatisfactory because it repeats information about studying/looking at Mars unnecessarily.

Explanation for Incorrect Answer C :  Choice (C) is unsatisfactory because it joins two complete thoughts with only a comma.

Explanation for Incorrect Answer E :  Choice (E) is unsatisfactory because it unnecessarily separates the act of looking at Mars from the act of studying Mars. As the passage indicates, the activities are the same.

(A) studied Mars by a high-powered telescope.

(B) In the late 1800’s an Italian astronomer named Giovanni Schiaparelli studied Mars with a high-powered telescope that he used to look at Mars.

(C) In the late 1800’s an Italian astronomer named Giovanni Schiaparelli studied Mars, he had a high-powered telescope that he used.

(D) In the late 1800’s an Italian astronomer named Giovanni Schiaparelli used a high-powered telescope to study Mars.

(E) In the late 1800’s an Italian astronomer named Giovanni Schiaparelli, using a high-powered telescope to look, studied Mars.

32

Which word would be best to insert at the beginning of sentence 10 (reproduced below)? Some people translated "canali" as "canals," word began to spread that the lines Schiaparelli saw through his telescope were actually canals that had been built by intelligent beings.

ANSWERS AND EXPLANATIONS Explanation for Correct Answer E :  Choice (E) is correct. The word "Because" appropriately signifies the relationship between the translation problem and the misunderstanding about the "canals."

Explanation for Incorrect Answer A :  Choice (A) is unsatisfactory because "Whereas" indicates that the mistranslation of "canali" and the misunderstanding about the building of the "canals" were contradictory ideas, when in fact one caused the other.

Explanation for Incorrect Answer B :  Choice (B) is unsatisfactory because "However" makes no sense in this context.

(A) Whereas

(B) However

(C) If

(D) Although

(E) Because

页码,21/24The Official SAT Online Course

2006-11-12file://E:\新建文件夹\e7.htm

UnRe

gistered

Page 99: SAT practice test 5.pdf

严禁用于商业用途!

Explanation for Incorrect Answer C :  Choice (C) is unsatisfactory because "If" suggests that there is some doubt as to whether or not people mistranslated "canali" in this way. The implication of the passage is that people did mistranslate the term, resulting in a misconception about the building of the "canals."

Explanation for Incorrect Answer D :  Choice (D) is unsatisfactory because "Although" implies that one would not expect word to spread given that the mistranslation occurred, when the passage indicates the opposite.

33 What is the best version of the underlined portion of sentence 11 (reproduced below)? One of them was an amateur astronomer named Percival Lowell.

ANSWERS AND EXPLANATIONS Explanation for Correct Answer C :  Choice (C) is correct. It properly signifies the relationship between the idea presented in the previous sentence and the astronomer Lowell.

Explanation for Incorrect Answer A :  Choice (A) is unsatisfactory because the pronoun "them" seems to refer to "intelligent beings" who built canals on Mars, and Lowell was clearly not one of these.

Explanation for Incorrect Answer B :  Choice (B) is unsatisfactory because Lowell is not described elsewhere in the passage as intelligent; in fact, his theory is shown to be based on a simple misunderstanding.

Explanation for Incorrect Answer D :  Choice (D) is unsatisfactory because it results in an awkward and illogical sentence.

Explanation for Incorrect Answer E :  Choice (E) is unsatisfactory because, according to the passage, Lowell created more problems than he solved.

(A) (As it is now)

(B) One of the most intelligent was

(C) This idea was popularized by

(D) It was spread even more by someone else,

(E) The person who solved the problem was

34

What is the best way to combine sentences 12 and 13 (reproduced below)? He wrote a series of bestselling books. In these books Lowell publicized the notion that these "canals" were built by Martian farmers who understood irrigation.

(A) In a series of bestselling books, Lowell publicized the notion that these "canals" were built by Martian farmers who understood irrigation.

(B)He wrote a series of books that was a bestseller and publicized the notion that these "canals" were built by Martian farmers who understood irrigation.

(C) His books that were bestsellers publicized the notion that these "canals" were built by Martian farmers who understood irrigation.

(D) He wrote a series of bestselling books; Lowell publicized the notion that these "canals" were built by Martian farmers who understood irrigation.

(E) In these books, which were bestsellers, Lowell publicized the notion that these "canals" were built by Martian farmers who understood irrigation.

页码,22/24The Official SAT Online Course

2006-11-12file://E:\新建文件夹\e7.htm

UnRe

gistered

Page 100: SAT practice test 5.pdf

严禁用于商业用途!

ANSWERS AND EXPLANATIONS Explanation for Correct Answer A :  Choice (A) is correct. The resulting sentence maintains the effective structure of the original sentence 13 while adding the important information from sentence 12.

Explanation for Incorrect Answer B :  Choice (B) is unsatisfactory because it is improper to describe a series of books as "a bestseller."

Explanation for Incorrect Answer C :  Choice (C) is unsatisfactory because, following sentence 11, it is more appropriate for the subject of the sentence to be "Lowell" than to be "his books."

Explanation for Incorrect Answer D :  Choice (D) is unsatisfactory because it does not indicate the relationship between the books Lowell wrote and his popularization of the "intelligent beings" idea.

Explanation for Incorrect Answer E :  Choice (E) is unsatisfactory because it suggests that Lowell's books have been previously mentioned in the passage ("In these books"), when this is the first mention of them.

35 What would best replace "it" in sentence 15 (reproduced below)? According to these images, there are no prominent channels anywhere on it.

ANSWERS AND EXPLANATIONS Explanation for Correct Answer A :  Choice (A) is correct. It corrects the error of the original by clearly specifying the object on which there are no prominent channels: the planet Mars.

Explanation for Incorrect Answer B :  Choice (B) is unsatisfactory because, in the context of the passage, it makes no sense to discuss "channels" on a spacecraft.

Explanation for Incorrect Answer C :  Choice (C) is unsatisfactory because the "images" mentioned earlier in the sentence are the "pictures" taken by the spacecraft. There is no need to refer to them twice.

Explanation for Incorrect Answer D :  Choice (D) is unsatisfactory because the word “these” does not logically refer to anything before or after sentence 15.

Explanation for Incorrect Answer E :  Choice (E) is unsatisfactory because it is unclear to what "them" refers.

(A) the planet

(B) the spacecraft

(C) the pictures

(D) these

(E) them

    

页码,23/24The Official SAT Online Course

2006-11-12file://E:\新建文件夹\e7.htm

UnRe

gistered

Page 101: SAT practice test 5.pdf

严禁用于商业用途!

Back to Score Report  

Copyright © 2006 The College Board. All rights reserved. Privacy Policy Terms of Use Contact Us

 

页码,24/24The Official SAT Online Course

2006-11-12file://E:\新建文件夹\e7.htm

UnRe

gistered

Page 102: SAT practice test 5.pdf

严禁用于商业用途!

Help | Profile | My Organizer | My Bookmarks | Logout

Answers and Explanations

Test Sections

Section 1

Section 2

Section 3

Section 4

Section 5

Section 6

Section 8

Section 9

Section 10

Back to Score Report  

View Answers and Explanations     Online - Practice Test #5

1The anthropology professor hoped that his latest book would appeal to popular as well as to ------- readers, thereby earning him ------- in both realms.

ANSWERS AND EXPLANATIONS Explanation for Correct Answer E :  Choice (E) is correct. “Scholarly” means learned or academic. “Acclaim” is praise or applause. If one were to insert this term into the text, the sentence would read “The anthropology professor hoped that his latest book would appeal to popular as well as to scholarly readers, thereby earning him acclaim in both realms.” The phrases “as well as” and “both realms” indicate that the professor hoped his book would appeal to two separate groups of readers. It makes sense to say that the professor hoped to earn “acclaim,” or praise, from both popular readers and “scholarly,” or academic, readers.

Explanation for Incorrect Answer A :  Choice (A) is incorrect. “General,” in this context, means not confined by specialization. “Disdain” is a feeling of contempt or scorn. If one were to insert these terms into the text, the sentence would read “The anthropology professor hoped that his latest book would appeal to popular as well as to general readers, thereby earning him disdain in both realms.” The phrases “as well as” and “both realms” indicate that the professor hoped his book would appeal to two separate groups of readers. The term “general” does not make logical sense in this context because there is not a clear distinction between the terms “general” and “popular” as they refer to similar groups of people. Additionally, the anthropology professor wanted his book to appeal to readers, so it is illogical to suggest that the anthropology professor hoped to earn “disdain,” or contempt.

Explanation for Incorrect Answer B :  Choice (B) is incorrect. “Lay” means not associated with a profession or any particular knowledge. “Attention” is observation or notice. If one were to insert these terms into the text, the sentence would read “The anthropology professor hoped that his latest book would appeal to popular as well as to lay readers, thereby earning him attention in both realms.” The phrases “as well as” and “both realms” indicate that the professor hoped his book would appeal to two separate groups of readers. The term “lay” does not make logical sense in this context because the terms “lay” and “popular” both refer to readers from one group, the general public.

Explanation for Incorrect Answer C :  Choice (C) is incorrect. “Academic” means scholarly or related to higher learning. “Anonymity” is a state of being unknown or unidentified. If one were to insert these terms into the text, the sentence would read “The anthropology professor hoped that his latest book would appeal to popular as well as to academic readers, thereby earning him anonymity in both realms.” The professor likely hoped his book would appeal to readers in both the “popular” realm and the “academic,” or scholarly, realm, but it is illogical to suggest that the professor hoped to earn “anonymity.” Anonymity, or a state of being unknown, is not usually described as something one earns, and there is no reason to believe that the professor would not want to be known for his work.

(A) general. . disdain

(B) lay. . attention

(C) academic. . anonymity

(D) avid. . remuneration

(E) scholarly. . acclaim

页码,1/18The Official SAT Online Course

2006-11-12file://E:\新建文件夹\e8.htm

UnRe

gistered

Page 103: SAT practice test 5.pdf

严禁用于商业用途!

Explanation for Incorrect Answer D :  Choice (D) is incorrect. “Avid” means enthusiastic and vigorous. “Remuneration” is payment for a service. If one were to insert these terms into the text, the sentence would read “The anthropology professor hoped that his latest book would appeal to popular as well as to avid readers, thereby earning him remuneration in both realms.” The phrases “as well as” and “both realms” indicate that the professor hoped his book would appeal to two separate groups of readers. The terms “avid” and “popular” do not necessarily describe separate groups. Popular readers, or readers from the general public, can also be “avid,” or enthusiastic, readers.

2 As if intended to squelch rumors of cutbacks, the company’s annual celebration was as ------- as ever.

ANSWERS AND EXPLANATIONS Explanation for Correct Answer D :  Choice (D) is correct. “Lavish” means extravagant and elaborate. If one were to insert this term into the text, the sentence would read “As if intended to squelch rumors of cutbacks, the company’s annual celebration was as lavish as ever.” The word “squelch” indicates that the missing term will describe a celebration that might suppress rumors of cutbacks, or reductions in spending. A “lavish,” or extravagant, celebration could certainly suppress such rumors.

Explanation for Incorrect Answer A :  Choice (A) is incorrect. “Sparing” means restrained, modest, or thrifty. If one were to insert this term into the text, the sentence would read “As if intended to squelch rumors of cutbacks, the company’s annual celebration was as sparing as ever.” The word “squelch” indicates that the missing term will describe a celebration that might suppress rumors of cutbacks, or reductions in spending. A "sparing," or thrifty, celebration would more likely encourage rumors of cutbacks than squelch them.

Explanation for Incorrect Answer B :  Choice (B) is incorrect. “Tawdry” means cheap. If one were to insert this term into the text, the sentence would read “As if intended to squelch rumors of cutbacks, the company’s annual celebration was as tawdry as ever.” A “tawdry,” or cheap, celebration would most likely strengthen rumors of cutbacks rather than squelch, or suppress, them.

Explanation for Incorrect Answer C :  Choice (C) is incorrect. “Belated” means delayed or late. If one were to insert this term into the text, the sentence would read “As if intended to squelch rumors of cutbacks, the company’s annual celebration was as belated as ever.” The word "squelch" indicates that the missing term will describe a celebration that might suppress rumors of cutbacks, or reductions in spending. Although a celebration could be “belated,” it is not logical to suggest that a late celebration might suppress such rumors.

Explanation for Incorrect Answer E :  Choice (E) is incorrect. “Sated” means satisfied almost to the point of excess. If one were to insert this term into the text, the sentence would read “As if intended to squelch rumors of cutbacks, the company’s annual celebration was as sated as ever.” While a person can be described as “sated,” it does not make logical sense to describe a celebration as “sated,” or satisfied.

(A) sparing

(B) tawdry

(C) belated

(D) lavish

(E) sated

3 The professor’s ------- lecture on American history served more to confuse the students than to clarify the difficult material.

页码,2/18The Official SAT Online Course

2006-11-12file://E:\新建文件夹\e8.htm

UnRe

gistered

Page 104: SAT practice test 5.pdf

严禁用于商业用途!

ANSWERS AND EXPLANATIONS Explanation for Correct Answer E :  Choice (E) is correct. “Disjointed” means out of order and incoherent. If one were to insert this term into the text, the sentence would read “The professor’s disjointed lecture on American history served more to confuse the students than to clarify the difficult material.” A “disjointed,” or incoherent, lecture would most likely confuse the students more than it would help them to understand the difficult material.

Explanation for Incorrect Answer A :  Choice (A) is incorrect. “Perspicacious” means keen or having acute mental abilities. If one were to insert this term into the text, the sentence would read “The professor’s perspicacious lecture on American history served more to confuse the students than to clarify the difficult material.” It is unlikely that a lecture could be both “perspicacious” and confusing to the students.

Explanation for Incorrect Answer B :  Choice (B) is incorrect. “Exhaustive” means thorough. If one were to insert this term into the text, the sentence would read “The professor’s exhaustive lecture on American history served more to confuse the students than to clarify the difficult material.” There is no reason to believe that an “exhaustive,” or thorough, lecture would confuse the students more than it would help them to understand the difficult material. On the contrary, an exhaustive lecture would be likely to clarify the material.

Explanation for Incorrect Answer C :  Choice (C) is incorrect. “Cogent” means relevant. If one were to insert this term into the text, the sentence would read “The professor’s cogent lecture on American history served more to confuse the students than to clarify the difficult material.” There is no reason to believe that a “cogent,” or relevant, lecture would confuse the students more than it would help them to understand the difficult material.

Explanation for Incorrect Answer D :  Choice (D) is incorrect. “Scintillating” means lively and witty. If one were to insert this term into the text, the sentence would read “The professor’s scintillating lecture on American history served more to confuse the students than to clarify the difficult material.” There is no reason to believe that a “scintillating,” or lively, lecture would confuse the students more than it would help them to understand the difficult material.

(A) perspicacious

(B) exhaustive

(C) cogent

(D) scintillating

(E) disjointed

4Since the foreign correspondent was accustomed to completing his assignments under ------- conditions, the commotion of the subway at rush hour presented no ------- his creativity.

ANSWERS AND EXPLANATIONS Explanation for Correct Answer C :  Choice (C) is correct. “Tumultuous” means chaotic and disorderly. An “impediment” means something that slows down or gets in the way of something

(A) squalid . . boon for

(B) tranquil . . obstruction to

(C) tumultuous . . impediment to

(D) destructive . . demonstration of

(E) flagrant . . benefit to

页码,3/18The Official SAT Online Course

2006-11-12file://E:\新建文件夹\e8.htm

UnRe

gistered

Page 105: SAT practice test 5.pdf

严禁用于商业用途!

else. If one were to insert these terms into the text, the sentence would read “Since the foreign correspondent was accustomed to completing his assignments under tumultuous conditions, the commotion of the subway at rush hour presented no impediment to his creativity.” The term “Since” indicates that there is a relationship between the conditions to which the correspondent was accustomed and his ability to work during the “commotion,” or noisy confusion, of the subway at rush hour. The noisy confusion of the subway was similar to the “tumultuous,” or chaotic and disorderly, conditions the correspondent was used to, so the commotion was not an “impediment” to his creativity.

Explanation for Incorrect Answer A :  Choice (A) is incorrect. “Squalid” means dirty and neglected. A “boon” is a benefit or favor. If one were to insert these terms into the text, the sentence would read “Since the foreign correspondent was accustomed to completing his assignments under squalid conditions, the commotion of the subway at rush hour presented no boon for his creativity.” The term “Since” indicates that there is a relationship between the conditions to which the correspondent was accustomed and his ability to work during the “commotion,” or noisy confusion, of the subway at rush hour. Because there is not necessarily any connection between “squalid,” or dirty and neglected, conditions and noisily confusing conditions, there is no reason to believe that the commotion of the subway would or would not have been a “boon,” or benefit, to the correspondent’s creativity.

Explanation for Incorrect Answer B :  Choice (B) is incorrect. “Tranquil” means calm and free from disturbance. An “obstruction” is something that blocks or gets in the way of something else. If one were to insert these terms into the text, the sentence would read “Since the foreign correspondent was accustomed to completing his assignments under tranquil conditions, the commotion of the subway at rush hour presented no obstruction to his creativity.” A correspondent who is used to working under “tranquil,” or calm, conditions would likely find the “commotion,” or noisy confusion, of the subway to be an “obstruction” to his creativity. Therefore, it is illogical to suggest that the commotion presented “no obstruction to,” or did not get in the way of, his creativity.

Explanation for Incorrect Answer D :  Choice (D) is incorrect. “Destructive” means causing harm and destroying. A “demonstration” is a public display. If one were to insert these terms into the text, the sentence would read “Since the foreign correspondent was accustomed to completing his assignments under destructive conditions, the commotion of the subway at rush hour presented no demonstration of his creativity.” Although the first term makes sense in this context, the second term does not. While the “commotion,” or noisy confusion, of the subway could affect the correspondent’s ability to be creative, it is illogical to suggest that a commotion could “demonstrate” his creativity.

Explanation for Incorrect Answer E :  Choice (E) is incorrect. “Flagrant” means obviously offensive. A “benefit” means an advantage or something that is useful. If one were to insert these terms into the text, the sentence would read “Since the foreign correspondent was accustomed to completing his assignments under flagrant conditions, the commotion of the subway at rush hour presented no benefit to his creativity.” Although the second term makes sense in this context, the first does not. It is illogical to describe a condition as “flagrant,” or obviously offensive.

5Unlike the ------- presentations of the other students in class, Mary Catherine’s presentation demonstrated a thorough and mature grasp of the material.

ANSWERS AND EXPLANATIONS Explanation for Correct Answer B :  Choice (B) is correct. “Sophomoric” means very immature and poorly informed.

(A) astute

(B) sophomoric

(C) incredulous

(D) cloying

(E) scintillating

页码,4/18The Official SAT Online Course

2006-11-12file://E:\新建文件夹\e8.htm

UnRe

gistered

Page 106: SAT practice test 5.pdf

严禁用于商业用途!

If one were to insert this term into the text, the sentence would read “Unlike the sophomoric presentations of the other students in class, Mary Catherine’s presentation demonstrated a thorough and mature grasp of the material.” The term “Unlike” indicates that the presentations of the other students differed from Mary Catherine’s in that they did not demonstrate a thorough, or complete, and mature grasp of the material. Presentations that did not demonstrate a mature and complete grasp of the material would most likely be described as “sophomoric,” or immature and poorly informed.

Explanation for Incorrect Answer A :  Choice (A) is incorrect. “Astute” means shrewd or clever. If one were to insert this term into the text, the sentence would read “Unlike the astute presentations of the other students in class, Mary Catherine’s presentation demonstrated a thorough and mature grasp of the material.” The term “Unlike” indicates that the presentations of the other students differed from Mary Catherine’s in that they did not demonstrate a thorough and mature grasp of the material. It is unlikely that presentations that did not demonstrate a thorough, or complete, grasp of the material would be described as “astute,” or shrewd and clever.

Explanation for Incorrect Answer C :  Choice (C) is incorrect. “Incredulous” means skeptical or expressing disbelief. If one were to insert this term into the text, the sentence would read “Unlike the incredulous presentations of the other students in class, Mary Catherine’s presentation demonstrated a thorough and mature grasp of the material.” Although the students could be “incredulous” themselves, it is illogical to say that the presentations were “incredulous” or skeptical.

Explanation for Incorrect Answer D :  Choice (D) is incorrect. “Cloying” means excessively sweet and sentimental. If one were to insert this term into the text, the sentence would read “Unlike the cloying presentations of the other students in class, Mary Catherine’s presentation demonstrated a thorough and mature grasp of the material.” The term “Unlike” indicates that the presentations of the other students differed from Mary Catherine’s in that they did not demonstrate a mature and thorough grasp of the material. The term “cloying” is somewhat illogical in this context, because an excessively sweet and sentimental presentation could still demonstrate a mature and complete understanding of the material.

Explanation for Incorrect Answer E :  Choice (E) is incorrect. “Scintillating” means brilliantly lively and witty. If one were to insert this term into the text, the sentence would read “Unlike the sophomoric presentations of the other students in class, Mary Catherine’s presentation demonstrated a thorough and mature grasp of the material.” The term “Unlike” indicates that the presentations of the other students differed from Mary Catherine’s in that they did not demonstrate a mature and thorough grasp of the material. The term “scintillating” is somewhat illogical in this context, because a brilliantly lively presentation could certainly demonstrate a mature and complete understanding of the material.

6Sadly, the author never ------- the rewards of literary success during her lifetime; public recognition and appreciation of her talent were completely -------.

ANSWERS AND EXPLANATIONS Explanation for Correct Answer B :  Choice (B) is correct. “To reap” means to obtain. “Posthumous” means following or occurring after death. If one were to insert these terms into the text, the sentence would read “Sadly, the author never reaped the rewards of literary success during her lifetime; public recognition and appreciation of her talent were completely posthumous.” The semicolon indicates that the idea presented in the second part of the sentence will provide evidence in support of the idea presented in the first part of the sentence. If public recognition and appreciation of the

(A) predicted . . conclusive

(B) reaped . . posthumous

(C) acknowledged . . fulsome

(D) appreciated . . gratuitous

(E) pursued . . discredited

页码,5/18The Official SAT Online Course

2006-11-12file://E:\新建文件夹\e8.htm

UnRe

gistered

Page 107: SAT practice test 5.pdf

严禁用于商业用途!

author’s talent were completely “posthumous,” or following her death, it makes sense to say that the author never “reaped,” or obtained, the rewards of literary success during her lifetime.

Explanation for Incorrect Answer A :  Choice (A) is incorrect. “To predict” means to foresee something. “Conclusive” means related to an outcome or conclusion. If one were to insert these terms into the text, the sentence would read “Sadly, the author never predicted the rewards of literary success during her lifetime; public recognition and appreciation of her talent were completely conclusive.” Although it is plausible to suggest that the author never “predicted,” or foresaw, that she would achieve literary success in her lifetime, it does not make sense to say that the recognition and appreciation she received were “conclusive,” or related to a conclusion.

Explanation for Incorrect Answer C :  Choice (C) is incorrect. “To acknowledge” means to take notice of something. “Affected” means a quality of behavior that is not natural to oneself. If one were to insert these terms into the text, the sentence would read “Sadly, the author never acknowledged the rewards of literary success during her lifetime; public recognition and appreciation of her talent were completely affected.” The semicolon indicates that the idea presented in the second part of the sentence will provide evidence in support of the idea presented in the first part of the sentence. It is reasonable to suggest that the author never “acknowledged,” or took notice of, the rewards of literacy success, but this idea is not logically connected to the idea that the recognition she received was unnatural to those who recognized her talent.

Explanation for Incorrect Answer D :  Choice (D) is incorrect. “To appreciate” means to recognize with gratitude. “Gratuitous” means unearned or unwarranted. If one were to insert these terms into the text, the sentence would read “Sadly, the author never appreciated the rewards of literary success during her lifetime; public recognition and appreciation of her talent were completely gratuitous.” The semicolon indicates that the idea presented in the second part of the sentence will provide evidence in support of the idea presented in the first part of the sentence. It is reasonable to suggest that the author never “appreciated,” or recognized with gratitude, the rewards of literary success and that the recognition and appreciation she received were “gratuitous,” or unearned, but there is no logical connection between these ideas.

Explanation for Incorrect Answer E :  Choice (E) is incorrect. “To pursue” means to seek. “Discredited” means not accepted as true or accurate. If one were to insert these terms into the text, the sentence would read “Sadly, the author never pursued the rewards of literary success during her lifetime; public recognition and appreciation of her talent were completely discredited.” The semicolon indicates that the idea presented in the second part of the sentence will provide evidence in support of the idea presented in the first part of the sentence. Although it is plausible to suggest that the author never “pursued,” or sought, the rewards of literary success, this idea is not logically connected to the idea that the public recognition and appreciation she received was “discredited.” Additionally, it is illogical to suggest that recognition and appreciation of an author’s talent would not be accepted as true.

These two passages discuss the same scholarly book, Strangers from a Different Shore by Asian American historian Ronald Takaki. The first passage is a critique of the book written by another Asian American historian, Li Ling-chi Wang; the second is a defense of the book by its author.

Passage 1

       In terms of its approach, Takaki’s  book is similar to Victor and Bret Nee’s  Longtime Californ’ (1972) because bothLine are media through which diverse Asian5 American voices surface. The major  difference is in how they collected and use  the voices. The Nees identified and 

页码,6/18The Official SAT Online Course

2006-11-12file://E:\新建文件夹\e8.htm

UnRe

gistered

Page 108: SAT practice test 5.pdf

严禁用于商业用途!

selected representative personalities from  different segments of the Chinese10 American community in San Francisco,  whom they laboriously interviewed  themselves over a two-year period, and  placed these lengthy, in-depth interviews  largely verbatim in their book with15 minimal narrative and analysis. From the  analytical standpoint, the book broke no  new ground. But it succeeded in giving us  the sights, sounds, flavors, perspectives,  and feelings of the community that had20 never before been permitted to surface.  For their labor, the book received lavish  reviews by the mainstream press.       Takaki’s approach is virtually  identical with that of the Nees except the25 voices used tend to be fragmentary and  mostly collected from indirect sources. To  begin with, no attempt was made to  distinguish voices and quotes from literary  works from voices of oral histories,30 documents, and newspapers. Unlike in  the Nees’ book, only a very tiny  percentage of the voices, probably less  than 15 percent, are based on personal  interviews by Takaki and of these, most35 are interviews of the author’s relatives  and colleagues. In this respect, the  author depends almost entirely on  available primary and secondary sources,  leaving readers wondering how40 representative they are.       Also, unlike the Nees, Takaki’s  book offers no analytical breakthrough for  our understanding of either Asian  American history or the newly emerging45 communities. Particularly disappointing is  Takaki’s silence on the historic emergence  of “Asian America” in the late 1960s and  early 1970s and his ahistorical approach  to the ensuing decades in Asian American50 history. The rise of ethnic and political  consciousness, the emergence of new  community organizations, and the rise of  Asian American studies are among the  most important developments in the55 history of Asians in the United States, in  my view.       Unfortunately both Takaki and  Roger Daniels [in the latter’s Asian  America: Chinese and Japanese in the

页码,7/18The Official SAT Online Course

2006-11-12file://E:\新建文件夹\e8.htm

UnRe

gistered

Page 109: SAT practice test 5.pdf

严禁用于商业用途!

Passage 2

60 U.S. since 1850] failed to adequately treat  this period of “Asian America” that gave  them the titles for their books, a term that  captures the richness of the newly  emerged Asian American political and65 cultural expressions.       In this respect, book reviewers are  wrong in characterizing Takaki’s book as a  “comprehensive history of 150 years of  Asian experience in the U.S.” It is more70 accurate to describe the book as a history  of Asians in the U.S. up to World War II,  with the final two chapters of anecdotal  events since then tacked on as an  epilogue.

75      Comparing Strangers from a  Different Shore to Victor and Brett Nee’s  Longtime Californ’, Wang points out that I  did not collect the voices myself. However  my study is a very different one from the80 Nees’. Theirs studied only one ethnic  group and only one geographical  community—San Francisco Chinatown.  Mine is comparative and also far more  comprehensive: it studies the Chinese,85 Japanese, Koreans, Filipinos, Asian  Indians, Vietnamese…Thus, given the  tremendous scope of my study, I had to  collect most of the voices from indirect  sources such as the Ethnic Studies Oral90 History Project, and I acknowledged them  through the study….       But I also directly interviewed  people. One of them, Touly Xiong, a  Hmong refugee now living in Wisconsin,95 told me: “We hope you will include our  stories in your book. Americans need to  understand us and what we have gone  through.” He said that he had fought for  the US in Laos and that his brother had100 been killed by North Vietnamese soldiers.   I also interviewed many of my relatives.  One of the meaningful things I discovered  and I hope other Asian American scholars  will learn is that our family histories are105 tied to the histories of our communities.  We, as scholars, are members of  communities, and our aunts, uncles, and  cousins have stories and voices that

页码,8/18The Official SAT Online Course

2006-11-12file://E:\新建文件夹\e8.htm

UnRe

gistered

Page 110: SAT practice test 5.pdf

严禁用于商业用途!

  belong to Asian American history….110      But what about Wang’s claim  that I “failed” to “adequately” treat the  post-World War II period? I wish I had  given more attention to the 1950s, but I  felt that the book was already an115 enormously long one and also that I  wanted to focus more on the post-1965  period. And here it is not accurate to  describe my book as “a history of Asians  in the US up to World War II.” The last120 two chapters (out of a total of twelve  chapters) are devoted to exactly this time  period. As any reader will see, they are  not “chapters of anecdotal events. . .  tacked on as an epilogue,” as Wang125 claims. Rather they are designed to relate  the early period to the more recent history  in a substantive way. They present an  analysis of post-World War II  developments….These certainly are not130 “anecdotal events.”

7 As used in line 1, “approach” most nearly means

ANSWERS AND EXPLANATIONS Explanation for Correct Answer B :  Choice (B) is correct. In context, “approach” most nearly means “technique.” The author of Passage 1 is comparing two books, saying that “both are media through which diverse Asian American voices surface”—clearly a reference to both writers’ technique, or methodology for treating their similar subject matter.

Explanation for Incorrect Answer A :  Choice (A) is incorrect. The author of Passage 1 is clearly comparing two books in their entirety in terms of how they handle their subject matter—in other words, their “technique,” or procedure. The author does not address whether or not the two books begin in the same manner. Their “beginnings” may or may not be similar.

Explanation for Incorrect Answer C :  Choice (C) is incorrect. In comparing two books, the author of Passage 1 is addressing their similar ways of handling their subject matter—in other words, their “technique,” or procedure. The author does not address whether or not the two books share a hypothesis, which in this context would mean a philosophical assumption. The paragraph goes on to compare the books in terms of what they do, not in terms of the assumptions they may or may not share.

Explanation for Incorrect Answer D :  Choice (D) is incorrect. While “approach” as a verb may have some connection to “advance” as a verb, it does not make sense to say that the two books have similar “advances.” In this section, the author of Passage 1 is clearly referring to methods or procedures that the authors of both books share—in other words, their

(A) beginning

(B) technique

(C) hypothesis

(D) advance

(E) style

页码,9/18The Official SAT Online Course

2006-11-12file://E:\新建文件夹\e8.htm

UnRe

gistered

Page 111: SAT practice test 5.pdf

严禁用于商业用途!

“techniques.”

Explanation for Incorrect Answer E :  Choice (E) is incorrect. The author of Passage 1 is clearly speaking here of the two books’ similar “technique,” or methods of handling their subject matter. They may or may not be similar in style, which would more likely refer to literary elements than to scholarly methodology, or “approach.”

8 In context, “surface” (line 20) is closest in meaning to

ANSWERS AND EXPLANATIONS Explanation for Correct Answer D :  Choice (D) is correct. The author is saying here that the Nees’ book provided information that had never before been disclosed about a certain community. It is clear from context (“never before been permitted to”) that this information has been willfully or deliberately kept obscure; in revealing this information, the Nees have allowed it to “emerge,” or come forth from obscurity.

Explanation for Incorrect Answer A :  Choice (A) is incorrect. The author of the passage is speaking here about the information that the Nees’ book has brought to light, or has allowed to “emerge.” In this context, it makes little sense to say that the voices in the book have been permitted to “arrange,” or “come to an agreement.”

Explanation for Incorrect Answer B :  Choice (B) is incorrect. The author of the passage is speaking here about the information that the Nees’ book has brought to light, or has allowed to “emerge.” In this context, it makes little sense to say that the voices in the book have been permitted to “level,” either in the sense of “equalize” or “tear down.”

Explanation for Incorrect Answer C :  Choice (C) is incorrect. While an object that has been submerged might be said to “float” to the surface, the author clearly is not speaking so literally here. Rather, he is speaking of previously obscure voices that have been allowed to “emerge,” or become evident.

Explanation for Incorrect Answer E :  Choice (E) is incorrect. The author of the passage is speaking here about the information that the Nees’ book has brought to light, or has allowed to “emerge.” In this context, it makes little sense to say that the voices in the book have been permitted to “smooth,” or to be become level or flat.

(A) arrange

(B) level

(C) float

(D) emerge

(E) smooth

9 In paragraph 2, the author of Passage 1 implies that a major difference between the Nees’ book and Takaki’s book is that

ANSWERS AND EXPLANATIONS

(A) the Nees’ book draws extensively on interviews conducted by the authors

(B) Takaki’s study focuses on U.S. citizens with roots in only one Asian nation

(C) the Nees make no significant analytical advances

(D) Takaki places great emphasis on the rise of Asian American studies

(E) the Nees’ book uses quotations from literary works and oral histories

页码,10/18The Official SAT Online Course

2006-11-12file://E:\新建文件夹\e8.htm

UnRe

gistered

Page 112: SAT practice test 5.pdf

严禁用于商业用途!

Explanation for Correct Answer A :  Choice (A) is correct. In the second paragraph, the author of Passage 1 contrasts Takaki’s book with the Nees’ book, saying that the former relies on voices that are “mostly collected from indirect sources” and, further, that Takaki, “unlike the Nees,” acquired “only a very tiny percentage” of his sources from direct interviews. Together, these comparisons imply that, in the view of the author, the Nees rely heavily on interviews that they themselves conducted, an area in which the author considers Takaki somewhat deficient.

Explanation for Incorrect Answer B :  Choice (B) is incorrect. In this paragraph, the author does not address the Asian roots of the people interviewed by either Takaki or the Nees; rather, he focuses on what he perceives as the shortage of direct interviews undertaken by Takaki in the preparation of the book.

Explanation for Incorrect Answer C :  Choice (C) is incorrect. Though in the first paragraph the author says that the Nees’ book “broke no new analytical ground,” he does not address this subject in paragraph 2. Further, the author nowhere implies that analytical advances are made by Takaki and not by the Nees, an assumption that underlies option C.

Explanation for Incorrect Answer D :  Choice (D) is incorrect. Elsewhere in the passage, the author implies that Takaki has placed insufficient emphasis on the rise of the field of Asian American studies. The author does not address this topic at all in the second paragraph of the passage.

Explanation for Incorrect Answer E :  Choice (E) is incorrect. While the author does mention “literary works and oral histories” in the second paragraph, he attributes their use to Takaki, not to the Nees. This paragraph is devoted to what the author sees as Takaki’s book’s deficiencies, among them a reluctance to differentiate among quotations from literary works and quotations from oral histories.

10In context, “distinguish” (line 28) most nearly means

ANSWERS AND EXPLANATIONS Explanation for Correct Answer B :  Choice (B) is correct. In this part of the passage, the author is accusing Takaki of neglecting to “distinguish” one kind of literary voice from another kind in his book; he means that Takaki does not “differentiate,” or make a distinction, among quotations from literary works and quotations from documentary sources.

Explanation for Incorrect Answer A :  Choice (A) is incorrect. Although “mark” can be said to mean “distinguish,” in the context of this passage the author is not speaking of physically marking different kinds of quotations in order to keep them distinct; rather, he is speaking of making a distinction among different kinds of quotations in the text of a book.

Explanation for Incorrect Answer C :  Choice (C) is incorrect. In this part of the passage, the author is accusing Takaki of neglecting to “distinguish,” or “differentiate,” among various types of quotations in the text of his book. It does not make logical sense or syntactic sense to say that Takaki fails to “analyze,” or methodically examine, one type of quotation or source “from” another type.

Explanation for Incorrect Answer D : 

(A) mark

(B) differentiate

(C) analyze

(D) judge

(E) discover

页码,11/18The Official SAT Online Course

2006-11-12file://E:\新建文件夹\e8.htm

UnRe

gistered

Page 113: SAT practice test 5.pdf

严禁用于商业用途!

Choice (D) is incorrect. In this part of the passage, the author is accusing Takaki of neglecting to “distinguish,” or “differentiate,” among various types of quotations in the text of his book. Although “judge” can be said to mean “distinguish,” it is clear from context that the author would have preferred that Takaki make some sort of distinction among various types of quotations, not that Takaki “judge,” or evaluate, such types.

Explanation for Incorrect Answer E :  Choice (E) is incorrect. In this part of the passage, the author is accusing Takaki of neglecting to “distinguish,” or “differentiate,” among various types of quotations in the text of his book. It does not make logical sense or syntactic sense to say that Takaki fails to “discover,” or learn about for the first time, one type of quotation or source “from” another type.

11 The author of Passage 1 mentions Roger Daniels primarily as an example of

ANSWERS AND EXPLANATIONS Explanation for Correct Answer D :  Choice (D) is correct. When the author of Passage 1 mentions Roger Daniels in lines 57-65 (“Unfortunately . . . expressions”), he states that Daniels, along with Takaki, “failed to adequately treat this period,” that is, a period during which, in the author’s view, Asian Americans organized themselves into political and cultural groups. The author considers this omission a shortcoming of both Takaki’s and Daniels’ books; in other words, both works “share a weakness.”

Explanation for Incorrect Answer A :  Choice (A) is incorrect. Daniels is mentioned as a scholar whose work, in the opinion of the author, has a shortcoming similar to one found in Takaki’s work. Nowhere in the passage is he identified as writing anything about Takaki’s work.

Explanation for Incorrect Answer B :  Choice (B) is incorrect. Daniels is mentioned as a scholar whose work, in the opinion of the author, has a shortcoming similar to one found in Takaki’s work. Nowhere in the passage is he identified as either someone Takaki consulted or someone whose work Takaki consulted in preparing his own book—the latter is likely, of course, but it is not mentioned in the passage.

Explanation for Incorrect Answer C :  Choice (C) is incorrect. Daniels is mentioned as a scholar whose work, in the opinion of the author, has a shortcoming similar to one found in Takaki’s work. If the author of the passage expresses any opinion at all of Daniels’ work, it is disapproval: “[Both books] failed to adequately treat this period….”

Explanation for Incorrect Answer E :  Choice (E) is incorrect. Daniels is mentioned as a scholar whose work, in the opinion of the author, has a shortcoming similar to one found in Takaki’s work. The two authors’ sharing what the author of the passage considers a shortcoming does not imply that the two writers ever worked together, much less “assisted” one another.

(A) an historian who wrote an essay critical of Takaki’s study

(B) a scholar who was frequently consulted by Takaki

(C) a writer whose research methods the author admires

(D) an author whose book shares a weakness with Takaki’s

(E) someone who assisted Takaki in writing his book

12 In context, “respect” (line 66) is closest in meaning to

(A) regard

(B) appreciation

(C) reference

(D) esteem

页码,12/18The Official SAT Online Course

2006-11-12file://E:\新建文件夹\e8.htm

UnRe

gistered

Page 114: SAT practice test 5.pdf

严禁用于商业用途!

ANSWERS AND EXPLANATIONS Explanation for Correct Answer A :  Choice (A) is correct. In lines 66 and following, the author of Passage 1 is referring back to the preceding paragraph, where two books are compared in terms of a common flaw. “This respect,” then, at the beginning of a new paragraph, refers to something like “Takaki’s neglect of Asian American history after World War II,” with “respect” meaning “regard,” or aspect.

Explanation for Incorrect Answer B :  Choice (B) is incorrect. In lines 66 and following, the author of Passage 1 is referring back to the preceding paragraph, where two books are compared in terms of a common flaw. “This respect,” then, at the beginning of a new paragraph, refers to something like “Takaki’s neglect of Asian American history after World War II.” It makes no sense, then, to say “In this appreciation”—or recognition of value—in this particular context.

Explanation for Incorrect Answer C :  Choice (C) is incorrect. In lines 66 and following, the author of Passage 1 is referring back to the preceding paragraph, where two books are compared in terms of a common flaw. “This respect,” then, at the beginning of a new paragraph, refers to something like “Takaki’s neglect of Asian American history after World War II.” While it might make sense in some contexts to say “In this reference”—or denotation—“critics are wrong,” the author makes it clear here that he means “respect” in the sense of “regard,” or “aspect.”

Explanation for Incorrect Answer D :  Choice (D) is incorrect. In lines 66 and following, the author of Passage 1 is referring back to the preceding paragraph, where two books are compared in terms of a common flaw. “This respect,” then, at the beginning of a new paragraph, refers to something like “Takaki’s neglect of Asian American history after World War II.” It makes no sense, then, to say “In this esteem”—or favorable regard—in this particular context.

Explanation for Incorrect Answer E :  Choice (E) is incorrect. In lines 66 and following, the author of Passage 1 is referring back to the preceding paragraph, where two books are compared in terms of a common flaw. “This respect,” then, at the beginning of a new paragraph, refers to something like “Takaki’s neglect of Asian American history after World War II.” It makes no sense, then, to say “In this recognition”—or favorable notice—in this particular context.

(E) recognition

13

Which of the following best describes the attitude of the author of Passage 1 toward Takaki’s book?

ANSWERS AND EXPLANATIONS Explanation for Correct Answer E :  Choice (E) is correct. The author of Passage 1 takes issue with two major elements of Takaki’s book: its research methodology (paragraph 2); and its treatment of Asian Americans in the U.S. in the 1950’s and 1960’s (paragraphs 3-5). In using such words as “disappointing,” “unfortunately,” “failed to treat,” and “tacked on as an epilogue,” the author betrays dissatisfaction, or discontent, with the book. The passage, however, maintains a scholarly detachment in its appraisal of the book, avoiding harsh language and outright attacks.

(A) disbelieving

(B) confrontational

(C) exasperated

(D) outraged

(E) dissatisfied

页码,13/18The Official SAT Online Course

2006-11-12file://E:\新建文件夹\e8.htm

UnRe

gistered

Page 115: SAT practice test 5.pdf

严禁用于商业用途!

Explanation for Incorrect Answer A :  Choice (A) is incorrect. The author of Passage 1 is clearly not completely satisfied with Takaki’s book, and finds fault with two of its major elements. But nowhere does the author say or imply that Takaki is less than truthful in his reporting or that he is withholding the truth, so the author’s attitude cannot be called “disbelieving.”

Explanation for Incorrect Answer B :  Choice (B) is incorrect. The author of Passage 1 is clearly not completely satisfied with Takaki’s book, and finds fault with two of its major elements. But nowhere does the author sound “confrontational,” or hostile—the author does not, for example, challenge Takaki to defend his sources or his methods, which would be confrontational.

Explanation for Incorrect Answer C :  Choice (C) is incorrect. While the author of the passage is clearly not completely satisfied with Takaki’s book, and finds fault with two of its major elements, he nowhere sounds “exasperated,” or angrily impatient. Throughout the passage, the tone is one of scholarly detachment, avoiding harsh language and outright attacks.

Explanation for Incorrect Answer D :  Choice (D) is incorrect. While the author of the passage is clearly not completely satisfied with Takaki’s book, and finds fault with two of its major elements, he nowhere sounds “outraged,” or grossly offended. Throughout the passage, the tone is one of scholarly detachment, avoiding harsh language and outright attacks.

14The author of Passage 2 asserts that the Nees’ book is inferior to Strangers from a Different Shore in terms of its

ANSWERS AND EXPLANATIONS Explanation for Correct Answer E :  Choice (E) is correct. In the first paragraph of Passage 2, Takaki contrasts Strangers from a Different Shore with Longtime Californ’, pointing out that the former “studied only one ethnic group and only one geographical community,” while the latter is “comparative and also far more comprehensive”; three lines later, he refers to “the tremendous scope of my study.” Thus, Takaki is comparing his book to the Nees’ book in terms of its scope, or range of material covered, and considers his superior.

Explanation for Incorrect Answer A :  Choice (A) is incorrect. The author of Passage 2 does not really mention “purpose” in regard to either book. He does say that Strangers from a Different Shore is “different,” and “more comprehensive,” but he is basically addressing range or scope rather than purpose with these descriptions.

Explanation for Incorrect Answer B :  Choice (B) is incorrect. The passage does not compare the two books in terms of their relative “readability,” or ease or pleasure with which they can be read.

Explanation for Incorrect Answer C :  Choice (C) is incorrect. While the author of Passage 2 does discuss matters of “documentation”—in, for example, lines 86-91—nowhere in the passage does he assert or imply that his methods are superior to those of the Nees, merely that the scope of his work necessitated a different kind of documentation.

Explanation for Incorrect Answer D :  Choice (D) is incorrect. The author of Passage 2 does mention “accuracy” in the last paragraph of the passage, but it is in connection with Li Ling-chi Wang, the

(A) purpose

(B) readability

(C) documentation

(D) accuracy

(E) range

页码,14/18The Official SAT Online Course

2006-11-12file://E:\新建文件夹\e8.htm

UnRe

gistered

Page 116: SAT practice test 5.pdf

严禁用于商业用途!

author of Passage 1, and not with the Nees’ work.

15 The author of Passage 2 mentions the Ethnic Studies Oral History Project (lines 89-90) primarily in order to

ANSWERS AND EXPLANATIONS Explanation for Correct Answer C :  Choice (C) is correct. In this part of the passage, the author is speaking of the “tremendous scope” of his book, which necessitated his using “indirect sources” such as the Ethnic Studies Oral History Project rather than relying solely on direct interviews. He is also defending himself against the charge made by the author of Passage 1: that he did not conduct enough personal interviews. Therefore, he is citing the Ethnic Studies Oral History Project by way of defending his use of it and of other “secondary sources.”

Explanation for Incorrect Answer A :  Choice (A) is incorrect. There is no indication in the passage that the Ethnic Studies Oral History Project either praised or criticized Takaki’s study, only that Takaki himself drew on the Project in his work.

Explanation for Incorrect Answer B :  Choice (B) is incorrect. While it is clear from context that Takaki considers the Ethnic Studies Oral History Project “a valuable resource”—having used it himself—there is no indication that the Nees either used it or did not use it in preparing their study.

Explanation for Incorrect Answer D :  Choice (D) is incorrect. While it can be inferred that Takaki and other scholars draw upon the work of their predecessors, there is no direct mention in the passage of Takaki’s “receiving assistance” nor any acknowledgement of the Ethnic Studies Oral History Project as the work of “earlier scholars” who might have been in a position to provide “assistance.”

Explanation for Incorrect Answer E :  Choice (E) is incorrect. There is no indication in the passage that Takaki was in any way involved with the authorship of the Ethnic Studies Oral History Project, only that he drew on it in documenting Strangers from a Different Shore.

(A) cite an organization that praised his study

(B) highlight a valuable resource overlooked by the Nees

(C) defend the validity of his use of secondary sources

(D) acknowledge the assistance he received from earlier scholars

(E) refer to a study he conducted earlier in his career

16 The author of Passage 2 uses the direct quotation in lines 95-98 most probably in order to

ANSWERS AND EXPLANATIONS Explanation for Correct Answer E :  Choice (E) is correct. The quotation, from a Hmong refugee, Touly Xiong, is provided in part to support the claim, “But I also directly interviewed people.” This assertion is, in turn, made to refute a claim made by the author of Passage 1, paraphrased by Takaki as “Wang points that I did not collect the voices myself” (lines 77-78). So including the direct quotation is intended to “counter,” or

(A) discredit an assertion made in the Nees’ book

(B) demonstrate that his book has gained a wide readership

(C) provide evidence that indirect sources are as valid as direct sources

(D) introduce claims about an ethnic group not discussed in his book

(E) counter a criticism made by the author of Passage 1

页码,15/18The Official SAT Online Course

2006-11-12file://E:\新建文件夹\e8.htm

UnRe

gistered

Page 117: SAT practice test 5.pdf

严禁用于商业用途!

refute, “a criticism made by the author of Passage 1” (that Takaki did not conduct interviews).

Explanation for Incorrect Answer A :  Choice (A) is incorrect. While Takaki states in Passage 2 that the Nees’ book is less ambitious than his own (lines 80-84), he nowhere discredits any of the Nees’ assertions. The quotation in lines 95-98 can be seen as an attempt “to discredit an assertion,” but it is an assertion made in the passage, not the Nees’ book.

Explanation for Incorrect Answer B :  Choice (B) is incorrect. The quotation in lines 95-98 is from a person interviewed by Takaki in the preparation of the latter’s book; there is no indication that the person quoted has read the book, nor is there any indication anywhere in the passage about the scope of Takaki’s audience.

Explanation for Incorrect Answer C :  Choice (C) is incorrect. While in the paragraph preceding this one Takaki defends the use of “indirect sources,” in lines 95-98 he is quoting a person he actually interviewed—in other words, addressing the validity of direct sources.

Explanation for Incorrect Answer D :  Choice (D) is incorrect. While Touly Xiong, the person quoted in lines 95-98, is identified as a Hmong refugee who has left Asia and now lives in the United States, it is obvious from context that Takaki is using the quotation to refer to an ethnic group that is discussed in Strangers from a Different Shore.

17With the parenthetical information in lines 120-121 Takaki intends primarily to

ANSWERS AND EXPLANATIONS Explanation for Correct Answer B :  Choice (B) is correct. In this paragraph, the author is defending his book against Wang’s charge that he “‘failed’ to ‘adequately’ treat the post-World War II period.” In lines 118-119, he quotes Wang’s description of Takaki’s book as “a history of Asians in the US up to World War II,” then insists that “the last two chapters (out of a total of twelve chapters) are devoted to exactly this time period”—that is, the years after World War II. Thus, by saying exactly how much of his book is devoted to this period, Takaki is using the parenthetical information to express how significant this period is to his book as a whole.

Explanation for Incorrect Answer A :  Choice (A) is incorrect. The parenthetical information tells how much of Takaki’s book—two out of twelve chapters—is devoted to a certain topic. Though he does indicate in lines 114-115 that his book is “enormously long,” he is not using the information in parenthesis to illustrate the overall length of the book.

Explanation for Incorrect Answer C :  Choice (C) is incorrect. While the paragraph as a whole suggests that Takaki’s treatment of post-World War II Asian American life comes near the end of his book, the parenthetical information is clearly intended to illustrate how great a proportion of the entire book is taken up with this topic, not where in the book the topic can be found.

Explanation for Incorrect Answer D :  Choice (D) is incorrect. The parenthetical information in no way refers to any other book; rather, Takaki is here referring to how much of his own book—two out of twelve chapters—is concerned with a single topic, Asian American history after World War II.

(A) imply that his book is a relatively lengthy one

(B) illustrate the significance the post-World War II era has in his book

(C) tell readers where his treatment of the post-World War II era can be found

(D) compare his book to a previous book about the post-World War II era

(E) describe the arrangement of chapters in his book

页码,16/18The Official SAT Online Course

2006-11-12file://E:\新建文件夹\e8.htm

UnRe

gistered

Page 118: SAT practice test 5.pdf

严禁用于商业用途!

Explanation for Incorrect Answer E :  Choice (E) is incorrect. While Takaki does say in this paragraph that the chapters of his book that deal with the post-World War II period come at the end of the book, he is clearly using the parenthetical information to demonstrate what proportion of his book is devoted to the topic.

18 The author of Passage 2 uses quotation marks in line 130 most likely in order to

ANSWERS AND EXPLANATIONS Explanation for Correct Answer C :  Choice (C) is correct. In this paragraph, Takaki is refuting Li Lin-chi Wang’s contention that Takaki’s book underemphasizes an important period of Asian American history. In lines 122-124, he quotes Wang's description of the book's last two chapters as "Chapters of anecdotal events...tacked on as an epilogue." Takaki, in lines 127-130, insists that the chapters are more substantive than Wang claims, directly quoting Wang’s judgment in order to disagree with it.

Explanation for Incorrect Answer A :  Choice (A) is incorrect. Quotation marks are more often used to indicate an uncommon expression than a common one; but in this context, it is clear that Takaki is quoting another author’s work, and that he takes issue with it.

Explanation for Incorrect Answer B :  Choice (B) is incorrect. While quotations marks might feasibly be used to highlight an important concept, it is clear in this context that Takaki is quoting another author’s work, and that he takes issue with it.

Explanation for Incorrect Answer D :  Choice (D) is incorrect. Since Takaki is quoting a negative assessment of his work here, it is unlikely that he is doing so in a humorous way. The tone of the paragraph as a whole does not support this interpretation.

Explanation for Incorrect Answer E :  Choice (E) is incorrect. While quotation marks are often used to call attention to an unusual phrase, it is clear from context that Takaki is refuting a judgment with which he disagrees; he does, after all, quote the judgment just above, in lines 122-125.

(A) indicate that he is using a common expression

(B) highlight an important concept

(C) cite a judgment with which he disagrees

(D) emphasize a humorous statement

(E) call attention to an unusual phrase

19 The authors of Passage 1 and Passage 2 would most probably agree with one another about which of the following statements concerning scholarly studies of Asian American communities?

ANSWERS AND EXPLANATIONS Explanation for Correct Answer D : 

(A) Asian American scholars should focus on earlier immigrant groups rather than on those who immigrated later.

(B) Oral histories are less credible than is written documentation of Asian American history.

(C) Interviewing one’s own family members is an effective way to represent the voices of one’s community.

(D) Post-World War II Asian American history is just as important as the history of earlier periods of Asian American history.

(E) A study that focuses solely on San Francisco can treat the history of Asian American immigrants in a sufficiently thorough way.

页码,17/18The Official SAT Online Course

2006-11-12file://E:\新建文件夹\e8.htm

UnRe

gistered

Page 119: SAT practice test 5.pdf

严禁用于商业用途!

Choice (D) is correct. In Passage 1, Wang criticizes Takaki for underplaying the post-World War II era in Strangers from a Different Shore, terming the events of the late 1960’s and early 1970's “among the most important developments in the history of Asians in the United States” (lines 53-55). While Takaki obviously disagrees with Wang’s assessment of the book, he does acknowledge the importance of the period, insisting in the last paragraph of Passage 2 that two out of twelve chapters of his book are dedicated to the period. Thus, it can be inferred that both authors agree about the importance of the post-World War II period in the study of Asian American history.

Explanation for Incorrect Answer A :  Choice (A) is incorrect. As it happens, both passages spend more time discussing relatively recent immigrants, but there is no indication in either passage that the authors consider earlier immigrants more worthy of study then later immigrants, or vice versa.

Explanation for Incorrect Answer B :  Choice (B) is incorrect. Far from considering oral histories less credible than written sources, both authors seem convinced of the importance of oral histories: Wang chides Takaki for failing to distinguish written histories from oral histories and for underusing oral interviews (Passage 1, paragraph 2), and Takaki defends at some length his use of oral histories (Passage 2, paragraph 2).

Explanation for Incorrect Answer C :  Choice (C) is incorrect. While Takaki defends the scholarly usefulness of interviewing family members (“our aunts, uncles, and cousins have stories and voices that belong to Asian American history”), Wang seems unenthusiastic about the practice, critically stating that “most [of Takaki’s interviews] are of the author’s relatives and colleagues.”

Explanation for Incorrect Answer E :  Choice (E) is incorrect. Both passages mention Longtime Californ’ by Victor and Bret Nee as a study that focuses solely on Chinese immigrants in San Francisco, but neither Wang nor Takaki says or implies that the book is thoroughly satisfactory: Wang says that it “broke no new [analytical] ground” (lines 16-17), while Takaki compares his book favorably to the Nees’ in terms of scope (lines 80-86).

    

Back to Score Report  

Copyright © 2006 The College Board. All rights reserved. Privacy Policy Terms of Use Contact Us

 

页码,18/18The Official SAT Online Course

2006-11-12file://E:\新建文件夹\e8.htm

UnRe

gistered

Page 120: SAT practice test 5.pdf

严禁用于商业用途!

Help | Profile | My Organizer | My Bookmarks | Logout

Answers and Explanations

Test Sections

Section 1

Section 2

Section 3

Section 4

Section 5

Section 6

Section 8

Section 9

Section 10

Back to Score Report  

View Answers and Explanations     Online - Practice Test #5

1

Of the labeled points on the cube shown above, which is farthest from point

ANSWERS AND EXPLANATIONS Explanation for Correct Answer C :  Choice (C) is correct. If is the length of each edge of the cube, then points

and are each distance from point The line segments and are

the hypotenuses of right triangles with both legs of length so the distance between points and and that between points and must each be

greater than Furthermore, the line segment between points and is the

hypotenuse of the right triangle with legs and so the distance between

points and must be greater than the distance between points and

Therefore, of the labeled points, point is farthest from point

Explanation for Incorrect Answer A :  Choice (A) is not correct. Point is one of the labeled points that is closest to point The question asks for the labeled point that is farthest from

Explanation for Incorrect Answer B :  Choice (B) is not correct. The line segment between points and is the

hypotenuse of the right triangle with legs and so the distance between

points and must be greater than the distance between points and

Explanation for Incorrect Answer D :  Choice (D) is not correct. The line segment between points and is the

hypotenuse of the right triangle with legs and so the distance between

points and must be greater than the distance between points and

Explanation for Incorrect Answer E :  Choice (E) is not correct. Point is one of the labeled points that is closest to

(A)

(B)

(C)

(D)

(E)

页码,1/14The Official SAT Online Course

2006-11-12file://E:\新建文件夹\e9.htm

UnRe

gistered

Page 121: SAT practice test 5.pdf

严禁用于商业用途!

point The question asks for the labeled point that is farthest from

2

In a scale model of a statue, inch represents inches. If the statue is

inches tall, what is the height of the scale model?

ANSWERS AND EXPLANATIONS Explanation for Correct Answer A : 

Choice (A) is correct. If inch on the model represents inches on the statue,

and if the statue is inches tall, then where is the height, in

inches, of the scale model of the statue. Solving this proportion for gives

Explanation for Incorrect Answer B :  Choice (B) is not correct. If the scale model were inches tall and the statue were

inches tall, then the proportion would hold. Solving this proportion

gives But is not

Explanation for Incorrect Answer C : 

Choice (C) is not correct. If the scale model were inches tall and the statue

were inches tall, then the proportion would hold. Solving this

proportion gives But is not

Explanation for Incorrect Answer D : 

Choice (D) is not correct. If the scale model were inches tall and the statue

were inches tall, then the proportion would hold. Solving this

(A) inches

(B) inches

(C) inches

(D) inches

(E) inches

页码,2/14The Official SAT Online Course

2006-11-12file://E:\新建文件夹\e9.htm

UnRe

gistered

Page 122: SAT practice test 5.pdf

严禁用于商业用途!

proportion gives But is not

Explanation for Incorrect Answer E :  Choice (E) is not correct. If the scale model were inches tall and the statue

were inches tall, then the proportion would hold. Solving this

proportion gives But is not

3

In the figure above, lines and intersect. If what is the value of

ANSWERS AND EXPLANATIONS Explanation for Correct Answer D :  Choice (D) is correct. The angled labeled and the angle consisting of all three

angles labeled are vertical angles, so

Explanation for Incorrect Answer A :  Choice (A) is not correct. This choice, is too small. It could be the result of visually estimating the answer, but the question does not ask for an estimate.

Explanation for Incorrect Answer B :  Choice (B) is not correct. This choice, is too small. It could be the result of visually estimating the answer, but the question does not ask for an estimate.

Explanation for Incorrect Answer C :  Choice (C) is not correct. This choice, is too small. It could be the result of visually estimating the answer, but the question does not ask for an estimate.

Explanation for Incorrect Answer E :  Choice (E) is not correct. This choice, is too large. It could be the result of visually estimating the answer, but the question does not ask for an estimate.

(A)

(B)

(C)

(D)

(E)

页码,3/14The Official SAT Online Course

2006-11-12file://E:\新建文件夹\e9.htm

UnRe

gistered

Page 123: SAT practice test 5.pdf

严禁用于商业用途!

4 If is more than twice what is in terms of

ANSWERS AND EXPLANATIONS Explanation for Correct Answer E :  Choice (E) is correct. The statement “ is more than twice ” can be written

as Subtracting from both sides of this equation gives

now dividing both sides of the resulting equation by yields

Therefore,

Explanation for Incorrect Answer A :  Choice (A) is not correct. If were equal to then would be equal to

which is not equivalent to “ more than twice ”

Explanation for Incorrect Answer B :  Choice (B) is not correct. If were equal to then would be equal to

which is not equivalent to “ more than twice ”

Explanation for Incorrect Answer C : 

Choice (C) is not correct. If were equal to then would be equal to

which is not equivalent to “ more than twice ”

Explanation for Incorrect Answer D : 

Choice (D) is not correct. If were equal to then would be equal to

which is not equivalent to “ more than twice ”

(A)

(B)

(C)

(D)

(E)

5

The scatterplot above shows, for each of students, the amount of time the student spent on homework plotted against the amount of time the student spent on

页码,4/14The Official SAT Online Course

2006-11-12file://E:\新建文件夹\e9.htm

UnRe

gistered

Page 124: SAT practice test 5.pdf

严禁用于商业用途!

after-school activities last week. According to the scatterplot, which of the following statements is true?

ANSWERS AND EXPLANATIONS Explanation for Correct Answer D :  Choice (D) is correct. The number of students who spent hours on homework can be found by counting the number of dots that fall on the vertical line representing hours of homework. There are three such dots, corresponding to

and hours of after-school activities. Therefore, exactly students spent

hours on homework.

Explanation for Incorrect Answer A :  Choice (A) is not correct. It is not true that each of the students spent the same amount of time on homework as on after-school activities. For example, the lowest leftmost dot represents a student who spent hours on homework and hour on after-school activities.

Explanation for Incorrect Answer B :  Choice (B) is not correct. It is not true that each of the students spent more time on homework than on after-school activities. For example, the highest rightmost dot represents a student who spent hours on homework and hours on after-school activities.

Explanation for Incorrect Answer C :  Choice (C) is not correct. It is not true that each of the students spent less time on homework than on after-school activities. For example, the lowest leftmost dot represents a student who spent hours on homework and hour on after-school activities.

Explanation for Incorrect Answer E :  Choice (E) is not correct. It is not true that exactly students spent hours on

after-school activities. The number of students who spent hours on after-school activities can be found by counting the number of dots on the horizontal line that represents hours of after-school activities. There are only two such dots, one

corresponding to hours of homework and one corresponding to hours of homework.

(A) Each of the students spent the same amount of time on homework as on after-school activities.

(B) Each of the students spent more time on homework than on after-school activities.

(C) Each of the students spent less time on homework than on after-school activities.

(D) Exactly students spent hours on homework.

(E) Exactly students spent hours on after-school activities.

6If the function is defined by what is the value of

ANSWERS AND EXPLANATIONS Explanation for Correct Answer B : 

Choice (B) is correct. If then

(A)

(B)

(C)

(D)

(E)

页码,5/14The Official SAT Online Course

2006-11-12file://E:\新建文件夹\e9.htm

UnRe

gistered

Page 125: SAT practice test 5.pdf

严禁用于商业用途!

Explanation for Incorrect Answer A : 

Choice (A) is not correct. not Therefore,

not

Explanation for Incorrect Answer C : 

Choice (C) is not correct. The value of cannot be because

Explanation for Incorrect Answer D :  Choice (D) is not correct. This incorrect answer could be the result of incorrectly

computing the sign of

Explanation for Incorrect Answer E : 

Choice (E) is not correct. not Therefore,

not

7

If the areas of the two rectangles in the figure above are equal, which of the following could be the coordinates of point

ANSWERS AND EXPLANATIONS Explanation for Correct Answer D :  Choice (D) is correct. Since is in Quadrant IV and the two rectangles in the figure are of equal area, it follows that the -coordinate of is positive, the -coordinate of is negative, and the absolute value of the product of the -coordinate of and the -coordinate of is Of the given choices, only

satisfies these three conditions.

Explanation for Incorrect Answer A :  Choice (A) is not correct. Since is in Quadrant IV, its -coordinate must be

positive. Therefore, cannot be the coordinates of

Explanation for Incorrect Answer B : 

(A)

(B)

(C)

(D)

(E)

页码,6/14The Official SAT Online Course

2006-11-12file://E:\新建文件夹\e9.htm

UnRe

gistered

Page 126: SAT practice test 5.pdf

严禁用于商业用途!

Choice (B) is not correct. Since is in Quadrant IV, its -coordinate must be

positive and its -coordinate must be negative. Therefore, cannot be the

coordinates of

Explanation for Incorrect Answer C :  Choice (C) is not correct. The upper rectangle in the figure has area If the

coordinates of point were then the lower rectangle would be of area

However, the question states that the two rectangles must be of equal area.

Explanation for Incorrect Answer E :  Choice (E) is not correct. Since is in Quadrant IV, its -coordinate must be

negative. Therefore, cannot be the coordinates of

8 l The first person ordered a salad. l The second person did not order a salad. l The third person ordered a hamburger. l The fourth person ordered the same thing as the first person. l The fifth person ordered the same thing as the second person.

A lunch stand has three choices: hamburger, hot dog, or salad. Five people from an office ordered one choice each from the lunch stand. The statements above are about what these five people ordered. If is the number of people who ordered a hamburger, which of the following statements is true?

ANSWERS AND EXPLANATIONS Explanation for Correct Answer E :  Choice (E) is correct. The first person ordered a salad. The second ordered either a hamburger or a hot dog. The third ordered a hamburger, and the fourth ordered a salad. The fifth person ordered either a hamburger or a hot dog to match the order of the second person. There are only two possibilities for the number of hamburgers ordered, and they depend on whether the second person ordered a hamburger or a hot dog. If the second person ordered a hot dog, the orders were salad, hot dog, hamburger, salad, hot dog. If the second person ordered a hamburger, the orders were salad, hamburger, hamburger, salad, hamburger. In the first case, the number of people who ordered a hamburger was In the second case, the number of people who ordered a hamburger was These are the only two possible cases, so the statement “ must be or ” must be true.

Explanation for Incorrect Answer A :  Choice (A) is not correct. The number of people who ordered a hamburger may be

but this number could also be Therefore, the statement “ must be ” is

not true.

Explanation for Incorrect Answer B :  Choice (B) is not correct. The number of people who ordered a hamburger could not be

Explanation for Incorrect Answer C :  Choice (C) is not correct. The number of people who ordered a hamburger may be

but this number could also be Therefore, the statement “ must be ”

is not true.

Explanation for Incorrect Answer D :  Choice (D) is not correct. The number of people who ordered a hamburger could be

but this number could not be

(A) must be

(B) must be

(C) must be

(D) must be or

(E) must be or

页码,7/14The Official SAT Online Course

2006-11-12file://E:\新建文件夹\e9.htm

UnRe

gistered

Page 127: SAT practice test 5.pdf

严禁用于商业用途!

9 If which of the following could be the value of

ANSWERS AND EXPLANATIONS Explanation for Correct Answer E : 

Choice (E) is correct. If then Thus, the possible values for

are and Therefore, of the choices given, only could be the value of

Explanation for Incorrect Answer A : 

Choice (A) is not correct. If then However,

not

Explanation for Incorrect Answer B : 

Choice (B) is not correct. If then

However, not

Explanation for Incorrect Answer C : 

Choice (C) is not correct. If then which is

undefined. However, is defined and equal to

Explanation for Incorrect Answer D : 

Choice (D) is not correct. If then

However, not

(A)

(B)

(C)

(D)

(E)

10The first term of a sequence is and each term after the first is more than the previous term. Which of the following is an expression for the term of the

sequence for any positive integer

(A)

(B)

(C)

页码,8/14The Official SAT Online Course

2006-11-12file://E:\新建文件夹\e9.htm

UnRe

gistered

Page 128: SAT practice test 5.pdf

严禁用于商业用途!

ANSWERS AND EXPLANATIONS Explanation for Correct Answer B :  Choice (B) is correct. From the given description of the sequence, the term is

the term is the term is

and for any positive integer the term is This last formula can

also be written as

Another way to see this is to look at the terms of the sequence. The first five terms are and Only the formula in choice (B) yields these terms for

and

Explanation for Incorrect Answer A :  Choice (A) is not correct. If for any positive integer the term of the

sequence were equal to then the first term of the sequence would be

But the first term of the sequence is

Explanation for Incorrect Answer C :  Choice (C) is not correct. If for any positive integer the term of the

sequence were equal to then the first term of the sequence would be

But the first term of the sequence is

Explanation for Incorrect Answer D :  Choice (D) is not correct. If for any positive integer the term of the

sequence were equal to then the first term of the sequence would be

But the first term of the sequence is

Explanation for Incorrect Answer E :  Choice (E) is not correct. If for any positive integer the term of the

sequence were equal to then the first term of the sequence would be

But the first term of the sequence is

(D)

(E)

11

In the figure above, and are all squares. If a point is chosen

at random from what is the probability that the point will be from one of the shaded regions?

(A)

(B)

(C)

页码,9/14The Official SAT Online Course

2006-11-12file://E:\新建文件夹\e9.htm

UnRe

gistered

Page 129: SAT practice test 5.pdf

严禁用于商业用途!

ANSWERS AND EXPLANATIONS Explanation for Correct Answer D :  Choice (D) is correct. The probability that a point chosen at random from will be from one of the shaded regions is equal to the ratio of the total area of the shaded regions to the area of The area of is equal to

and the area of the shaded region is

Since is a square, it follows that Since is also a

square, it follows that so Thus, the area of is

equal to Similarly, the area of the second shaded region, is

equal to Therefore, the probability that the chosen point

will be from one of the shaded regions is

Explanation for Incorrect Answer A :  Choice (A) is not correct. This is the probability that the point chosen at random from will be from the shaded region but the point could also be

from the shaded region

Explanation for Incorrect Answer B : 

Choice (B) is not correct. This choice is equal to but that is not equal to the

probability that a point chosen at random from will be from one of the shaded regions.

Explanation for Incorrect Answer C :  Choice (C) is not correct. This is the ratio of to but that is not equal to

the probability that a point chosen at random from will be from one of the shaded regions.

Explanation for Incorrect Answer E : 

Choice (E) is not correct. is the probability that a point chosen at random from

will be from one of the unshaded regions, but the question asks for the probability that the point will be from one of the shaded regions.

(D)

(E)

12If which of the following must be true?

ANSWERS AND EXPLANATIONS Explanation for Correct Answer B : 

Choice (B) is correct. Expanding the left-hand side of gives

Subtracting from both sides yields now

multiplying both sides by gives

(A)

(B)

(C)

(D)

(E)

页码,10/14The Official SAT Online Course

2006-11-12file://E:\新建文件夹\e9.htm

UnRe

gistered

Page 130: SAT practice test 5.pdf

严禁用于商业用途!

Explanation for Incorrect Answer A :  Choice (A) is not correct. The value of must be not

Explanation for Incorrect Answer C : 

Choice (C) is not correct. The value of does not have to be if the

equation holds given any value for

Explanation for Incorrect Answer D :  Choice (D) is not correct. It is the value of that must be not the value of

Explanation for Incorrect Answer E :  Choice (E) is not correct. It is the value of that must be not the value of

13 If what is the value of

ANSWERS AND EXPLANATIONS Explanation for Correct Answer A : 

Choice (A) is correct. Since it follows that Thus,

Therefore,

Explanation for Incorrect Answer B : 

Choice (B) is not correct. If were equal to then would be

equal to But equals not and so

Explanation for Incorrect Answer C : 

Choice (C) is not correct. The value of is not equal to the sum of

the constant terms and

Explanation for Incorrect Answer D : 

Choice (D) is not correct. implies that Therefore, each factor

in the product is positive, and so the product cannot be equal to

Explanation for Incorrect Answer E :  Choice (E) is not correct. The value of can be determined from the condition

Therefore, the value of the product can also be

determined.

(A)

(B)

(C)

(D)

(E) It cannot be determined from the information given.

14What is the ratio of the diameter of a circle to half the circumference of the circle?

(A)

(B)

(C)

(D)

页码,11/14The Official SAT Online Course

2006-11-12file://E:\新建文件夹\e9.htm

UnRe

gistered

Page 131: SAT practice test 5.pdf

严禁用于商业用途!

ANSWERS AND EXPLANATIONS Explanation for Correct Answer C :  Choice (C) is correct. A circle with diameter has circumference Thus, half

the circumference of the circle is Therefore, the ratio of the diameter to half

the circumference is which is equal to or

Explanation for Incorrect Answer A :  Choice (A) is not correct. is the ratio of the radius of the circle to the circumference of the circle.

Explanation for Incorrect Answer B :  Choice (B) is not correct. is the ratio of the diameter of the circle to the entire circumference, but the question asks for the ratio of the diameter to half the circumference.

Explanation for Incorrect Answer D :  Choice (D) is not correct. is the ratio of the circumference of the circle to the diameter of the circle.

Explanation for Incorrect Answer E :  Choice (E) is not correct. is the ratio of the circumference of the circle to the radius of the circle.

(E)

15A carton contains boxes of paper plates, and each box contains plates. If the carton costs dollars, what is the cost per paper plate, in dollars, when the plates are bought by the carton?

ANSWERS AND EXPLANATIONS Explanation for Correct Answer B :  Choice (B) is correct. Each carton contains boxes of paper plates, and each box

contains plates, so each carton contains plates. Since each carton costs

dollars and contains plates, the cost per plate is dollars.

Explanation for Incorrect Answer A :  Choice (A) is not correct. Each carton contains boxes of paper plates, and each

box contains plates, so each carton contains plates. Since each carton costs

dollars and contains plates, the cost per plate is dollars, not

dollars.

(A)

(B)

(C)

(D)

(E)

页码,12/14The Official SAT Online Course

2006-11-12file://E:\新建文件夹\e9.htm

UnRe

gistered

Page 132: SAT practice test 5.pdf

严禁用于商业用途!

Explanation for Incorrect Answer C : 

Choice (C) is not correct. This choice, is the number of plates per dollar, not

the number of dollars per plate.

Explanation for Incorrect Answer D :  Choice (D) is not correct. Each carton contains boxes of paper plates, and each

box contains plates, so each carton contains plates. Since each carton costs

dollars and contains plates, the cost per plate is dollars, not

Explanation for Incorrect Answer E :  Choice (E) is not correct. Each carton contains boxes of paper plates, and each

box contains plates, so each carton contains plates. Since each carton costs

dollars and contains plates, the cost per plate is dollars, not

16

Triangles and in the figure above are equilateral. What is the ratio of

to

ANSWERS AND EXPLANATIONS Explanation for Correct Answer B : 

Choice (B) is correct. Since and it follows that is the

perpendicular bisector of Let the point where and intersect, as

shown in the figure below.

Then and are all triangles. Thus,

(A) to

(B) to

(C) to

(D) to

(E) to

页码,13/14The Official SAT Online Course

2006-11-12file://E:\新建文件夹\e9.htm

UnRe

gistered

Page 133: SAT practice test 5.pdf

严禁用于商业用途!

the ratio of to is to Since it follows that

and Therefore,

Explanation for Incorrect Answer A : 

Choice (A) is not correct. Let the point where and intersect. If the

ratio of to were to then the ratio of to would also be

to But then, by the Pythagorean Theorem, the ratio of to would

be to and so the ratio of to would be to But it is given

that is equilateral, so this cannot be true.

Explanation for Incorrect Answer C : 

Choice (C) is not correct. Let the point where and intersect. Then

is a triangle with having measure and

having measure Thus, so Therefore, the ratio of

to cannot be to

Explanation for Incorrect Answer D : 

Choice (D) is not correct. Let the point where and intersect. The ratio

of to is to but the ratio of to is to

Explanation for Incorrect Answer E : 

Choice (E) is not correct. Let the point where and intersect. If the

ratio of to were to then the ratio of to would also be

to But then, by the Pythagorean Theorem, the ratio of to would

be to and so the ratio of to would be to But it is

given that is equilateral, so this cannot be true.

    

Back to Score Report  

Copyright © 2006 The College Board. All rights reserved. Privacy Policy Terms of Use Contact Us

 

页码,14/14The Official SAT Online Course

2006-11-12file://E:\新建文件夹\e9.htm

UnRe

gistered

Page 134: SAT practice test 5.pdf

严禁用于商业用途!

Help | Profile | My Organizer | My Bookmarks | Logout

Answers and Explanations

Test Sections

Section 1

Section 2

Section 3

Section 4

Section 5

Section 6

Section 8

Section 9

Section 10

Back to Score Report  

View Answers and Explanations     Online - Practice Test #5

1 The first 10,000 United States patents, they were known as the X-patents, were burned in a fire in 1836.

ANSWERS AND EXPLANATIONS Explanation for Correct Answer E :  Choice (E) is correct. It avoids the comma splice of the original by replacing the independent clause “they were known as the X-patents” with the participial phrase (“known as the X-patents”) that describes the subject of the independent clause, “patents.”

Explanation for Incorrect Answer A :  Choice (A) involves a comma splice. This sentence contains two independent clauses joined only by a comma: “The first 10,000 United States patents were burned in a fire in 1836” and “they were known as the X-patents.” However, adding a coordinating conjunction after the first comma or replacing the first comma with a semicolon will not correct the problem because one independent clause comes between the subject (“patents”) and verb (“were burned”) of the other independent clause. The error can be corrected by turning the interrupting clause into a subordinate participial phrase, “known as the X-patents,” that modifies the subject (“patents”) of the other independent clause.

Explanation for Incorrect Answer B :  Choice (B) creates a pronoun error. This revision corrects the comma-splice error of the original by turning one independent clause (“they were known as the X-patents”) into a dependent clause introduced by “which,” but the revision creates a pronoun error. The pronoun “they” has no antecedent to which it can logically refer.

Explanation for Incorrect Answer C :  Choice (C) creates a pronoun error. This revision corrects the comma-splice error of the original by turning one independent clause (“they were known as the X-patents”) into a dependent clause introduced by “which,” but the revision creates a pronoun error. The pronoun “they” has no antecedent to which it can logically refer.

Explanation for Incorrect Answer D :  Choice (D) involves faulty logic. Although this revision corrects the comma-splice error of the original by turning one independent clause (“they were known as the X-patents”) into a subordinate infinitive phrase (“to be known”), it also creates faulty logic. The infinitive phrase “to be known” illogically suggests that the action will occur at some point in the future rather than now, in the present.

(A) they were known

(B) which they knew

(C) which they know

(D) to be known

(E) known

2 Winston knew that if he practiced often enough he would one day be able to play the piano as well as his brother’s playing.

his brother’s playing

页码,1/11The Official SAT Online Course

2006-11-12file://E:\新建文件夹\e10.htm

UnRe

gistered

Page 135: SAT practice test 5.pdf

严禁用于商业用途!

ANSWERS AND EXPLANATIONS Explanation for Correct Answer D :  Choice (D) is correct. It avoids the illogical comparison and mixed construction of the original by replacing the gerund phrase “his brother’s playing” with the clause “his brother could.” This change also untangles the mixed construction of the phrase “as well as” by turning “as well” into an adverb modifying “to play” and “as his brother could play” into an adverb clause modifying the adverb “well.”

Explanation for Incorrect Answer A :  Choice (A) involves faulty logic. The sentence is meant to compare the way that Winston would one day be able to play the piano with the way that his brother already could play, but instead it illogically suggests that Winston could play the piano as well as “his brother’s playing” could.

Explanation for Incorrect Answer B :  Choice (B) results in faulty logic. The pronoun “that” refers to the noun “piano,” so this construction illogically suggests that Winston could play the piano as well as his brother’s piano could play.

Explanation for Incorrect Answer C :  Choice (C) results in faulty logic. With the phrase “his brother’s” serving as the object of the preposition “as,” the construction illogically suggests that Winston could play the piano as well as his brother’s piano could play.

Explanation for Incorrect Answer E :  Choice (E) results in faulty logic. With the phrase “what his brother did” serving as the object of the preposition “as,” the construction illogically suggests that Winston could play the piano as well as something that his brother did could play.

(A)

(B) that of his brother

(C) his brother’s

(D) his brother could

(E) what his brother did

3In both his longer and his shorter works of fiction, Gabriel García Márquez achieves the rare feat to be accessible to the common reader while satisfying the most demanding of sophisticated critics.

ANSWERS AND EXPLANATIONS Explanation for Correct Answer C :  Choice (C) is correct. It avoids the improper phrasing of the original by replacing the infinitive phrase “to be” with an idiomatic participial phrase (“of being”) to modify the noun “feat.”

Explanation for Incorrect Answer A :  Choice (A) involves improper phrasing. The preposition “of being,” instead of the infinitive “to be,” is the idiomatic construction to modify the noun “feat.”

Explanation for Incorrect Answer B : 

(A) to be

(B) for being

(C) of being

(D) that he is

(E) that they are

页码,2/11The Official SAT Online Course

2006-11-12file://E:\新建文件夹\e10.htm

UnRe

gistered

Page 136: SAT practice test 5.pdf

严禁用于商业用途!

Choice (B) involves improper phrasing. The preposition “of,” instead of the preposition “for,” is the idiomatic preposition to follow “feat.”

Explanation for Incorrect Answer D :  Choice (D) involves an awkward construction. Although a dependent clause introduced by the relative pronoun “that” can modify a noun, the construction is not idiomatic in this sentence. The prepositional phrase “of being” is the idiomatic construction to modify the noun “feat.”

Explanation for Incorrect Answer E :  Choice (E) involves an awkward construction and a pronoun error. Although a dependent clause introduced by the relative pronoun “that” can modify a noun, the construction is not idiomatic in this sentence. The prepositional phrase “of being” is the idiomatic construction to modify the noun “feat.” Also, in the context of this sentence, the pronoun that serves as the subject of the dependent clause (“they”) should refer to the singular subject of the main clause, “Marquez,” not to the object of the preposition, the plural noun “works.”

4 In 1977, Jann Wenner, the founder of Rolling Stone, moved the magazine’s offices from San Francisco to New York City, there he began developing a slicker, more commercial style of magazine and began cultivating relationships with major advertisers.

ANSWERS AND EXPLANATIONS Explanation for Correct Answer C :  Choice (C) is correct. It avoids the comma-splice error of the original by using the relative pronoun “where” to introduce a subordinate clause that describes what happened when the magazine moved its offices to New York City.

Explanation for Incorrect Answer A :  Choice (A) creates a comma splice. The sentence presents two complete ideas, but the ideas are joined using only a comma. Two independent clauses should never be joined by only a comma. This problem can be fixed by using the relative pronoun “where” to introduce a dependent clause that describes what happened when the magazine moved its offices to New York City.

Explanation for Incorrect Answer B :  Choice (B) creates a comma splice. The sentence presents two complete ideas, but the ideas are joined using only a comma. Two independent clauses should never be joined by only a comma. This problem can be fixed by using the relative pronoun “where” to introduce a dependent clause that describes what happened when the magazine moved its offices to New York City.

Explanation for Incorrect Answer D :  Choice (D) creates a comma splice. The sentence presents two complete ideas, but the ideas are joined using only a comma. Two independent clauses should never be joined by only a comma. This problem can be fixed by using the relative pronoun “where” to introduce a dependent clause that describes what happened when the magazine moved its offices to New York City.

Explanation for Incorrect Answer E :  Choice (E) involves awkward phrasing and ambiguous pronoun reference. While the pronoun “which” is used correctly to introduce a dependent clause, the first part of the sentence (“In 1977, Jann Wenner, the founder of Rolling Stone, moved the magazine’s offices from San Francisco to New York City”) would have to be recast so that the pronoun “when” would unambiguously refer to a time (“1977”) rather than to a place (“New York City”).

(A) there

(B) then

(C) where

(D) so

(E) which is when

页码,3/11The Official SAT Online Course

2006-11-12file://E:\新建文件夹\e10.htm

UnRe

gistered

Page 137: SAT practice test 5.pdf

严禁用于商业用途!

5 Covering about 120 square miles, the New York City borough of Queens is almost as large as if you combine Manhattan, the Bronx, and Staten Island.

ANSWERS AND EXPLANATIONS Explanation for Correct Answer C :  Choice (C) is correct. It avoids the awkward phrasing and unnecessary shift in person of the original by replacing the subordinate clause introduced by “if” with a noun phrase, “Manhattan, the Bronx, and Staten Island combined,” which makes the comparison logical by comparing a noun (“Queens”) with other nouns. This revision also avoids the unnecessary shift to the second person by deleting “you.”

Explanation for Incorrect Answer A :  Choice (A) involves awkward phrasing and an unnecessary shift in person. In the comparative construction “as … as,” the first “as” is an adverb that modifies the adjective (“large”) that follows it, and the second “as” is a preposition, which requires an object. An object of a preposition must be a noun. The clause “if … Island” acts as an adverb, not a noun, so it cannot serve as the object of a preposition. In addition, the use of the second-person pronoun “you” is an unnecessary shift in person.

Explanation for Incorrect Answer B :  Choice (B) involves awkward phrasing. In the comparative construction “as … as,” the first “as” is an adverb that modifies the adjective (“large”) that follows it, and the second “as” is a preposition, which requires an object. An object of a preposition must be a noun. The clause “when … combined” acts as an adverb, not a noun, so it cannot serve as the object of a preposition.

Explanation for Incorrect Answer D :  Choice (D) involves awkward phrasing. In the comparative construction “as … as,” the first “as” is an adverb that modifies the adjective (“large”) that follows it, and the second “as” is a preposition, which requires an object. An object of a preposition must be a noun. The clause “if … Island” acts as an adverb, not a noun, so it cannot serve as the object of a preposition.

Explanation for Incorrect Answer E :  Choice (E) makes an illogical comparison. In the comparative construction “as … as,” the first “as” is an adverb that modifies the adjective (“large”) that follows it, and the second “as” is a preposition, which requires an object. An object of a preposition must be a noun. Although the gerund “combining” is a noun, it results in an illogical comparison: Queens, a borough, is compared to “combining,” not to the other boroughs.

(A) if you combine Manhattan, the Bronx, and Staten Island

(B) when Manhattan, the Bronx, and Staten Island are combined

(C) Manhattan, the Bronx, and Staten Island combined

(D) if Manhattan were to combine with the Bronx and Staten Island

(E) combining Manhattan, the Bronx, and Staten Island

6While cotton is still one of Georgia’s chief cash crops, the number of acres devoted to its cultivation grows smaller every year.

ANSWERS AND EXPLANATIONS Explanation for Correct Answer A :  Choice (A) is correct. It avoids the errors of the other options by using concise phrasing (“its cultivation”) in place of awkward wording, by removing

(A) devoted to its cultivation grows

(B) it devotes to the cultivation of it grow

(C) they devoted to its cultivation can grow

(D) devoted to its cultivation growing

(E) they devoted to the cultivation of it are growing

页码,4/11The Official SAT Online Course

2006-11-12file://E:\新建文件夹\e10.htm

UnRe

gistered

Page 138: SAT practice test 5.pdf

严禁用于商业用途!

unnecessary pronouns (“it” and “they”), and by providing a singular verb “grows” to agree with the singular subject “the number.”

Explanation for Incorrect Answer B :  Choice (B) involves awkward phrasing and subject-verb disagreement. The awkward phrase “it devotes to the cultivation of it” can be reduced to “devoted to its cultivation.” The plural verb “grow” does not agree with the singular subject “the number.” The singular verb “grows” should be used instead.

Explanation for Incorrect Answer C :  Choice (C) involves illogical pronoun use and an error in verb tense. There is nothing in the sentence to which the plural pronoun “they” can logically refer, and the use of the past tense (“devoted”) is not consistent with the use of the present tense elsewhere in the sentence (“is”).

Explanation for Incorrect Answer D :  Choice (D) results in a sentence fragment. As is, the sentence contains two dependent clauses. To fix this problem, a main verb is needed (“the number . . . is growing”).

Explanation for Incorrect Answer E :  Choice (E) involves illogical pronoun use, awkward phrasing, and subject-verb disagreement. There is nothing in the sentence to which the plural pronoun “they” can logically refer, the awkward phrase “devoted to the cultivation of it” can be reduced to “devoted to its cultivation,” and the plural verb “are” does not agree in number with the singular subject “the number.”

7 Unlike flying squirrels, which may leave their young in a tree cavity while foraging, the babies of flying lemurs are usually carried with them.

ANSWERS AND EXPLANATIONS Explanation for Correct Answer D :  Choice (D) is correct. It avoids the illogical comparison of the original by replacing the subject of the main clause (“babies”) with “flying lemurs” so that “flying squirrels” and “flying lemurs” are compared.

Explanation for Incorrect Answer A :  Choice (A) involves an illogical comparison. Because the subject of the independent clause is “babies,” adult “flying squirrels” are illogically compared to “babies” rather than to “flying squirrels.”

Explanation for Incorrect Answer B :  Choice (B) involves an illogical comparison. Because the subject of the independent clause, “babies,” is modified by the singular possessive “flying lemur’s,” adult “flying squirrels” are illogically compared to “babies” rather than to “flying lemurs.”

Explanation for Incorrect Answer C :  Choice (C) creates an unparallel comparison and a verb-tense error. The singular subject of the independent clause, “flying lemur,” is compared to the plural noun “flying squirrels.” Also, the simple present tense (“carries”), not the present progressive tense (“is carrying”), should be used in statements of general truth such as this one.

Explanation for Incorrect Answer E :  Choice (E) involves an illogical comparison. Because the subject of the independent clause, “babies,” is modified by the plural possessive “flying lemurs’,” adult

(A) the babies of flying lemurs are usually carried

(B) the flying lemur’s babies are usually carried

(C) the flying lemur is usually carrying its babies

(D) flying lemurs usually carry their babies

(E) flying lemurs’ babies are usually carried

页码,5/11The Official SAT Online Course

2006-11-12file://E:\新建文件夹\e10.htm

UnRe

gistered

Page 139: SAT practice test 5.pdf

严禁用于商业用途!

“flying squirrels” are illogically compared to “babies” rather than to “flying lemurs.”

8 Born Charles Hardin Holley, Buddy Holly recorded some of the most distinctive and influential songs in rock-and-roll music, which includes such classics as “That'll Be the Day,” “Rave On,” and “Peggy Sue.”

ANSWERS AND EXPLANATIONS Explanation for Correct Answer D :  Choice (D) is correct. Its avoids the verb-tense error of the original. The tense of the verb in the dependent clause should be consistent with the past tense of the main verb “recorded.” Replacing the dependent clause (“which … ‘Peggy Sue’”) with a participial phrase (“including … ‘Peggy Sue’”) to modify the noun “songs” fixes this inconsistency in verb tenses.

Explanation for Incorrect Answer A :  Choice (A) involves a verb-tense error. The tense of the verb in a dependent clause should be consistent with the tense of the verb of the main clause. The present tense of the verb “includes” is inconsistent with the past tense verb “recorded.”

Explanation for Incorrect Answer B :  Choice (B) creates a comma splice. The two independent clauses “Born Charles Hardin Holley, Buddy Holly recorded some of the most distinctive and influential songs in rock-and-roll music” and “they include such classics as ‘That'll Be the Day,’ ‘Rave On,’ and ‘Peggy Sue’” are joined only by a comma rather than by a semicolon or a comma and a coordinating conjunction.

Explanation for Incorrect Answer C :  Choice (C) involves faulty logic and unclear modification. The adverb “also” used to introduce the participial phrase (“also … ‘Peggy Sue’”) suggests that the songs named in the participial phrase are in addition to other songs already named, but no other songs have been named. Also, the noun that this participial phrase modifies is unclear. It could modify either “songs” or “music” in the main clause.

Explanation for Incorrect Answer E :  Choice (E) creates a comma splice. The two independent clauses “Born Charles Hardin Holley, Buddy Holly recorded some of the most distinctive and influential songs in rock-and-roll music” and “these include such classics as ‘That'll Be the Day,’ ‘Rave On,’ and ‘Peggy Sue’” are joined only by a comma rather than by a semicolon or a comma and a coordinating conjunction.

(A) which includes

(B) they include

(C) also including

(D) including

(E) these include

9 Chipmunks do not technically hibernate, but they do remain in their dens or burrows during cold periods.

ANSWERS AND EXPLANATIONS Explanation for Correct Answer A : 

(A) hibernate, but they do remain

(B) hibernate, but remaining

(C) hibernate, but they remained

(D) hibernate, they remain

(E) hibernate, remaining

页码,6/11The Official SAT Online Course

2006-11-12file://E:\新建文件夹\e10.htm

UnRe

gistered

Page 140: SAT practice test 5.pdf

严禁用于商业用途!

Choice (A) is correct. The two independent clauses “Chipmunks do not technically hibernate” and “they do remain in their dens or burrows during cold periods” are correctly connected with a comma and coordinating conjunction (“, but”), and the conjunction “but” clearly expresses the contrast between the ideas of the two clauses. Also, the two clauses are parallel in structure, and their verbs are consistent in tense.

Explanation for Incorrect Answer B :  Choice (B) involves a lack of parallelism. The two grammatical units connected by a coordinating conjunction (“but”) should be parallel, but in this construction, an independent clause (“Chipmunks do not technically hibernate”) is connected with a coordinating conjunction to a participial phrase (“remaining in their dens or burrows during cold periods”).

Explanation for Incorrect Answer C :  Choice (C) involves an improper verb tense. The two independent clauses in this sentence (“Chipmunks … hibernate” and “they …periods”) are statements of general truth. Statements of general truth should be expressed in the present tense. The past-tense verb of the second clause, “remained,” should be the present-tense verb “remain.”

Explanation for Incorrect Answer D :  Choice (D) creates a comma splice and involves faulty logic. The two independent clauses “Chipmunks do not technically hibernate” and “they remain in their dens or burrows during cold periods” are connected only by a comma rather than by a semicolon or a comma and a coordinating conjunction. Also, without the conjunction “but” to express the contrast between the ideas of the two clauses, the two clauses seem to contradict each other. The first clause states that chipmunks do not hibernate, and the second clause suggests that they do.

Explanation for Incorrect Answer E :  Choice (E) creates faulty logic. Although this sentence is grammatically correct, it is not logical. The independent clause “Chipmunks do not technically hibernate” and the participial phrase “remaining in their dens or burrows during cold periods' contradict other." This faulty logic can be corrected by connecting the two ideas with a contrasting conjunction, as in the original.

10Cryonics — the technique used to store human bodies at extremely low temperatures with the hope of one day reviving them — are being performed today, but their technology is still in its infancy.

ANSWERS AND EXPLANATIONS Explanation for Correct Answer E :  Choice (E) is correct. It avoids the errors in subject–verb agreement and pronoun–antecedent agreement of the original. It replaces the plural verb “are being performed” with the singular verb “is being performed” to agree with the singular subject “Cryonics.” This revision also replaces the plural pronoun “their,” for which there is no logical antecedent.

Explanation for Incorrect Answer A :  Choice (A) involves errors in subject–verb agreement and pronoun–antecedent agreement. “Cryonics” is a singular noun and requires the singular verb “is being performed,” not the plural verb “are being performed.” Also the pronoun “their” can not logically refer to anything in the sentence.

Explanation for Incorrect Answer B :  Choice (B) involves subject–verb disagreement and imprecise word choice.

(A) are being performed today, but their technology

(B) are being performed today, while the technology

(C) which is being performed today, using technology that

(D) is being performed today, but their technology

(E) is being performed today, but the technology

页码,7/11The Official SAT Online Course

2006-11-12file://E:\新建文件夹\e10.htm

UnRe

gistered

Page 141: SAT practice test 5.pdf

严禁用于商业用途!

“Cryonics” is a singular noun and requires the singular verb “is being performed,” not the plural verb “are being performed.” Also, the use of the subordinate conjunction “while” to introduce the clause “while the technology is still in its infancy” does not clearly express the contrast between the idea expressed in the dependent clause and the idea of the main clause (“Cryonics … is being performed today”).

Explanation for Incorrect Answer C :  Choice (C) creates a sentence fragment. Although the subject–verb and pronoun–antecedent agreement errors are corrected, this revision creates a sentence fragment. The use of “which” turns the verb phrase “is being performed today” into a subordinate clause, which cannot stand alone. Also the participle “using” turns all that follows into a subordinate phrase, which cannot stand alone. Therefore, the subject “Cryonics” has no main verb to complete the sentence.

Explanation for Incorrect Answer D :  Choice (D) involves an error in pronoun–antecedent agreement. There is nothing in the sentence to which the plural pronoun “their” can logically refer.

11 In science fiction stories, robots are generally depicted as working in the service of humanity, often escaping the control of their human masters and doing them harm.

ANSWERS AND EXPLANATIONS Explanation for Correct Answer C :  Choice (C) is correct. It avoids the errors of the other options by precisely expressing the contrast between the idea of the main clause (“robots … humanity”) and the idea of the subordinate phrase (“often … harm”).

Explanation for Incorrect Answer A :  Choice (A) inappropriately uses a participial phrase (“often … doing”) that does not precisely express the contrast between the two ideas in the sentence (“robots … humanity” and “often … harm”).

Explanation for Incorrect Answer B :  Choice (B) involves imprecise phrasing and faulty logic. The participial phrase “often being controlled by their human masters” does not clearly express the contrast between the idea of the main clause (“robots … humanity”) and the idea of this subordinate clause. Also, the subordinate clause “which does them harm” modifies “escaping,” and the pronoun “them” refers to “robots.” This clause illogically suggests that the robots cause themselves harm by escaping human control, instead of suggesting that the robots harm their human masters when they escape human control.

Explanation for Incorrect Answer D :  Choice (D) involves errors in verb tense and word choice and faulty logic. The past tense of the verb “escaped” is inconsistent with the present tense of the main verb “are … depicted.” Also, the adjective “controlling,” used to modify “human masters,” illogically changes the meaning from humans in general to only those humans who control robots. In addition, the pronouns “it” and “them” in the independent clause “and it does them harm” illogically suggests that the robots cause themselves harm by escaping human control, instead of suggesting that the robots harm their human masters when they escape human control.

Explanation for Incorrect Answer E :  Choice (E) involves improper word choice and faulty logic. The adjective “controlling,” used to modify “human masters,” illogically changes the meaning from humans in general to only those humans who control robots. In addition, the subordinate clause “which does them harm” modifies “escaping,” and the pronoun “them” refers to “robots.” This clause illogically suggests that

(A) often escaping the control of their human masters and doing

(B) often escaping being controlled by their human masters, which does

(C) but they often escape the control of their human masters and do

(D) but they often escaped their controlling human masters, and it does

(E) but often escaping their controlling human masters, which does

页码,8/11The Official SAT Online Course

2006-11-12file://E:\新建文件夹\e10.htm

UnRe

gistered

Page 142: SAT practice test 5.pdf

严禁用于商业用途!

the robots cause themselves harm by escaping human control, instead of suggesting that the robots harm their human masters when they escape human control.

12 The first Indian woman to win the Booker Prize, Arundhati Roy’s debut novel, The God of Small Things, received the honor in 1997.

ANSWERS AND EXPLANATIONS Explanation for Correct Answer C :  Choice (C) is correct. It avoids the apposition error of the original by replacing the subject of the main clause, “novel,” with the subject “Arundhati Roy” so that the introductory appositive, “The first Indian woman to receive the Booker Prize,” properly renames the author, not the novel.

Explanation for Incorrect Answer A :  Choice (A) involves an error in apposition. In this sentence, the introductory phrase, “The first Indian woman to receive the Booker Prize,” illogically renames “novel,” the subject of the main clause, rather than the author, Arundhati Roy.

Explanation for Incorrect Answer B :  Choice (B) creates a sentence fragment. Neither the infinitive “to be” nor the participle “receiving” can serve as a main verb for the subject “Arundhati Roy.” A main verb is needed to complete the action of the sentence.

Explanation for Incorrect Answer D :  Choice (D) involves faulty logic and faulty modification. The participial phrase “winning her the first such honor” modifies the subject (“Booker Prize”) of the main clause. It does not make sense to say that the prize won the prize. In addition, the prepositional phrases “for an Indian woman” and “in 1977” combine to illogically suggest that Roy won the prize “for an Indian woman in 1977,” not for her own novel, The God of Small Things.

Explanation for Incorrect Answer E :  Choice (E) creates faulty logic. In its simplest subject–verb–direct object pattern, this sentence states “Arundhati Roy … made … her,” which illogically suggests that the author made herself.

(A) The first Indian woman to win the Booker Prize, Arundhati Roy’s debut novel, The God of Small Things, received the honor in 1997.

(B) Arundhati Roy, the first Indian woman to win the Booker Prize for her debut novel The God of Small Things, receiving the honor in 1997.

(C) The first Indian woman to receive the Booker Prize, Arundhati Roy won the honor in 1997 for her debut novel, The God of Small Things.

(D)The Booker Prize was given to Arundhati Roy’s debut novel The God of Small Things, winning her the first such honor for an Indian woman in 1997.

(E) Arundhati Roy, winning the Booker Prize in 1997 for her debut novel The God of Small Things, made her the first Indian woman to do so.

13 Pilots at the airline, angered at the prospect of seeing their pension plans replaced with less generous versions, vowed to use legal means to fight it.

ANSWERS AND EXPLANATIONS Explanation for Correct Answer D :  Choice (D) is correct. It avoids the pronoun error of the original by replacing the pronoun “it,” which has no clear antecedent, with the noun phrase “such a

(A) it

(B) them

(C) this

(D) such a move

(E) that from happening

页码,9/11The Official SAT Online Course

2006-11-12file://E:\新建文件夹\e10.htm

UnRe

gistered

Page 143: SAT practice test 5.pdf

严禁用于商业用途!

move.”

Explanation for Incorrect Answer A :  Choice (A) involves a pronoun error. The pronoun “it” has no antecedent to which it clearly refers. Because the pronoun is singular, “it” can refer to either of the two singular nouns (“prospect” or “airline”).

Explanation for Incorrect Answer B :  Choice (B) involves a pronoun error. The plural pronoun “them” has no antecedent to which it clearly refers. Because it is plural, “them” could refer to any of the plural nouns in the first part of the sentence: “pilots,” “plans,” or “versions.”

Explanation for Incorrect Answer C :  Choice (C) involves a pronoun error. The pronoun “this” has no antecedent to which it clearly refers. Because the pronoun is singular, “this” can refer to either of the two singular nouns (“prospect” or “airline”).

Explanation for Incorrect Answer E :  Choice (E) involves a pronoun error and awkward phrasing. The pronoun “that” has no antecedent to which it clearly refers. Because the pronoun is singular, “that” can refer to either of the two singular nouns (“prospect” or “airline”). In addition to its pronoun problem, the phrase “fight that from happening” is not idiomatic. The idiomatic phrase is “keep that from happening,” but this correction still does not correct the pronoun problem.

14This basic document is stating the liberties guaranteed to the English people, the Magna Carta, signed in 1215 by England’s King John, proclaims rights that have formed the foundation of the constitutions of every English-speaking nation.

ANSWERS AND EXPLANATIONS Explanation for Correct Answer D :  Choice (D) is correct. It avoids the comma-splice error of the other options by turning the first independent clause, “This basic document is stating the liberties” into an appositive. An appositive is a subordinate noun phrase that renames a noun. In this revision, “A basic document” is the appositive that renames “Magna Carta,” and the dependent clause “that states the liberties” modifies “a basic document.”

Explanation for Incorrect Answer A :  Choice (A) involves a comma splice. This long sentence contains two independent clauses (“This basic document … the English people” and “the Magna Carta, signed … nation”) joined by only a comma. The error can be corrected by turning the first independent clause into an appositive that renames the subject of the second independent clause, “the Magna Carta.” In addition, the progressive tense of the verb “is stating” should be changed to the simple present “states” to be consistent with the present tense of the independent clause (“proclaims”).

Explanation for Incorrect Answer B :  Choice (B) involves a comma splice. This long sentence contains two independent clauses (“This basic document … the English people” and “the Magna Carta, signed … nation”) joined by only a comma. The error can be corrected by turning the first independent clause into an appositive that renames the subject of the second independent clause, “the Magna Carta.”

Explanation for Incorrect Answer C :  Choice (C) involves a comma splice. This long sentence contains two independent

(A) This basic document is stating

(B) This basic document states

(C) A basic document, it states

(D) A basic document that states

(E) A basic document, it stated

页码,10/11The Official SAT Online Course

2006-11-12file://E:\新建文件夹\e10.htm

UnRe

gistered

Page 144: SAT practice test 5.pdf

严禁用于商业用途!

clauses (“A basic document, it … the English people” and “the Magna Carta, signed … nation”) joined by only a comma. The error can be corrected by turning the first independent clause into an appositive that renames the subject of the second independent clause, “the Magna Carta.”

Explanation for Incorrect Answer E :  Choice (E) involves a comma splice. This long sentence contains two independent clauses (“A basic document, it … the English people” and “the Magna Carta, signed … nation”) joined by only a comma. The error can be corrected by turning the first independent clause into an appositive that renames the subject of the second independent clause, “the Magna Carta.”

  

Back to Score Report  

Copyright © 2006 The College Board. All rights reserved. Privacy Policy Terms of Use Contact Us

 

页码,11/11The Official SAT Online Course

2006-11-12file://E:\新建文件夹\e10.htm

UnRe

gistered